Combined Sets!!

Ace your homework & exams now with Quizwiz!

Queen Electric is structured around five primary business divisions: Queen Power, Queen Oil & Gas, Queen Energy Connections, Queen Transportation, and Queen Digital. These business areas are subdivided further into either product or geographic divisions. What kind of structure has Queen Electric adopted? A. divisional B. hollow C. matrix D. functional E. horizontal

A

Roscoe Products produce widgets. Using a(n) __________, they are able to achieve economies of scale by specializing the application of labor to specific and standardized functions. A. traditional design B. open design C. horizontal structure D. organic structure E. closed design

A

A(n) ________ organization is composed of "a group of people working together to collectively enhance their capacities to create results that they truly care about." A. information B. complex C. network D. learning

D

Government, church, and military organizations are based on a (n) _______ structure. They have many layers of managers and red tape. A. horizontal B. open C. boundaryless D. traditional E. virtual

D

O'Connell Research is a small organization with four partners, each responsible for one primary function. The organization structure that is best for small companies is A. divisional. B. hollow. C. matrix. D. functional. E. horizontal

D

Which of the following is not an organizational subsystem of an organization? A. managerial B. technical C. structural D. environmental E. psychosocial

D

More on Espoused values

generally established by the founder of a new or small company and by the top management team in a larger organization. represent aspirations that are explicitly communicated to employees, managers hope that those values will directly influence employee behavior, but this does not always happen

Realistic job previews

mitigate unrealistic expectations formed during this phase Higher performance Lower quit rates

More on Basic Underlying Assumptions

represent deep-seated beliefs that employees have about their company and thus constitute the core of organizational culture. highly resistant to change More companies have basic underlying assumptions about sustainability.

More on Enacted values

represent the values that employees ascribe to an organization based on their observations of what occurs on a daily basis. It is important for managers to reduce gaps between espoused and enacted values because they can significantly influence employee attitudes and organizational performance.

Four important characteristics of organizational culture

(1) it is a shared concept; (2) it is learned over time; (3) it influences our behavior at work;(4) it impacts outcomes at multiple levels.

Organizational culture operates on three levels

(1) observable artifacts; (2) espoused values; and (3) basic underlying assumptions.

Jane works in an organization where quality and efficiency are highly valued. This organization's culture is likely A.hierarchy. B.adhocracy. C.goal-driven. D.clan. E.market.

.....

Which level of organizational culture is the hardest to change? A.artifacts B.transactional C.enacted values D.espoused values E.basic underlying assumptions

........

Five elements drive organizational culture

1) the founder's values; (2) the industry and business environment; (3) the national culture; (4) the organization's vision and strategies; and (5) the behavior of leaders.

Four truths about culture change

1.Leaders are the architects and developers of organizational change. 2.Changing culture starts with one of the three levels of organizational culture: artifacts, espoused values, basic underlying assumptions. 3.Consider how closely the current change aligns with the organization's vision and strategic plan. 4.Use a structured approach when implementing culture change.

The human body is an example of a(n) ________ because it requires life-sustaining oxygen, nutrients, and water from the environment. A. open system B. closed system C. learning system D. hierarchy E. flat organization

A

The three-phase model of organizational socialization

Anticipatory socialization Encounter Change and acquisition

A horizontal approach to organizational design tends to focus on A. specific customers. B. work processes. C. functional expertise. D. efficiency over effectiveness. E. use of information technology

B

Which of the following is an open organizational design? A. matrix B. modular C. horizontal D. functional E. divisional

B

Adam is a new product development manager for Hello.com, an Internet match-making service. He has only two employees reporting directly to him. From what you have read about organization charts, what do you know for sure? A. His subordinates enjoy greater autonomy than others. B. His subordinates take part in the decision-making process. C. He has a narrow span of control. D. His subordinates are not closely supervised. E. He is encountering low administrative costs

C

Narrow spans of control are most likely in companies with a(n) A. clan culture. B. market culture. C. hierarchical culture. D. adhocracy culture. E. network structure

C

ABC Company has divided its employees into departments of manufacturing, marketing, and finance. The company is said to have a(n) _____ structure. A. entrepreneurial B. project C. hollow D. modular E. functional

E

Aligned goals start with the A. hierarchy of authority. B. organizational structure. C. organizational culture. D. organizational climate. E. strategic plan.

E

Johnson & Co. has divided its employees according to the locations of the customers that they tend to. Thus, it has teams that cater to the urban areas, semi-urban areas, and so on. Johnson & Co. is said to have a ________ structure. A. functional B. hollow C. modular D. virtual E. divisional

E

Mary Flowers, the founder of MedTran, an offsite transcription service, asks transcribers to discuss their potential problems at weekly staff meetings. ___________ is the process that a learning organization uses to share information among its people. A. Information acquisition B. Information interpretation C. Knowledge integration D. Organizational memory E. Information distribution

E

Simentec Inc., a tech support company, has over 150 employees working in five countries. The company allows workers to work from anywhere, and virtual meetings are the primary means of communication across the company. Simentec is a _________ structure—one whose members are geographically apart and work using information technology. It appears to customers as a single, unified organization with a real physical location. A. hollow B. modular C. matrix D. horizontal E. virtual

E

organizational socialization

The process by which a person learns the values, norms, and required behaviors which permit them to participate as a member of an organization

Sustainability

a company's ability to make a profit without sacrificing the resources of its people, the community, and the planet

Vision

a long-term goal that describes what an organization wants to become

Organizational structure and internal processes then affect

a variety of group and social processes, which impact employees' work attitudes and behaviors and a variety of organizational outcomes.

When we rank an individual value in order of their_____, we obtain the person's value system. a. Intensity b. Content c. Context d. Flexibility

a. Intensity

Which of the following tends to have a disproportionate (higher than it should be) amount of influence on employee selection decisions? a. Interviews b. Written Tests c. Performance-Simulation Tests d. Work-Sampling Tests

a. Interviews

Some of the functions of organizational culture are: a. Organizational identity, collective commitment, and social system stability b. Clan, market, adhocracy, hierarchy c. Stability and flexibility anticipatory encounter and acquisition d. Inboarding and outboarding

a. Organizational identity, collective commitment, and social system stability

Three levels of organizational culture are: a. Strategy, mission, and values b. External environmental and internal environment c. Observable artifacts, espoused values, and basic underlying assumptions d. Espoused values and enacted values e. Identity, commitment, and stability

c. Observable artifacts, espoused values, and basic underlying assumptions

Which of the following is the least predictive set of criteria used to evaluate employees? a. Task outcomes b. Behaviors c. Personality d. Traits

c. Personality

Applicants describe how they handled specific problems and situations in previous jobs in a(n)______ interview. a. Performance-simulation b. Problem solving c. Audition d. Behavioral structured

d. Behavior structured

When managers tend to give individuals the same ratings on several different dimensions based on their overall view of the person, this is called.. a. Leniency Error b. Fundamental Attribution Error c. Central Tendency error d. Halo Error

d. Halo Error

John Holland's personality job-fit theory presents six personality types. Which of the following is one of these six types? a. Analytic b. Imaginative c. Practical d. Investigative

d. Investigative

Giving Negative Feedback... a. Should be in the form of a "sandwich":--good, bad, good. b. Is to be avoided because it is demoralizing c. Is more motivating when given in a group setting d. Should be kept short.

d. Should be kept short.

Espoused values

the explicitly stated values and norms that are preferred by an organization

Organizational culture

the set of shared, taken-for-granted implicit assumptions that a group holds and that determines how it perceives, thinks about, and reacts to its various environments.

The Meyers Motor Company to¬day is very different from the same company of 1965, yet many essential characteristics remain so that "Meyers is still Meyers." This suggests that organizations have ____________, the means to retain and transmit information from past to future members of the social system. A. organizational memory B. information acquisition C. information distribution D. information interpretation E. knowledge integration

A

Outcomes Associated with Organizational Culture

Organizational culture is related to measures of organizational effectiveness. Employees are more satisfied and committed to organizations with clan cultures. Innovation and quality can be increased by building characteristics associated with clan, adhocracy, and market cultures. Financial performance is not strongly related to organizational culture. Market cultures tend to have more positive organizational outcomes.

Levels of decisions

1) Strategic decisions: made to set the course of an organization (CEOs and Boards of Directors) 2) Tactical decisions: decisions about how things will get done (Managers) 3) Operational decisions: decisions employees make each day to make the organization function (employees throughout the organization)

8 Political Tactics

1. Attacking or blaming others. 2. Using information as a political tool. 3. Creating a favorable image. (Also known as impression management.) 4. Developing a base of support. 5. Praising others (ingratiation). 6. Forming power coalitions with strong allies. 7. Associating with influential people. 8. Creating obligations (reciprocity).

3 Levels of Political Action

1. Individual 2. Coalition 3. Network

_______ is the number of people reporting directly to a given manager. A. Span of control B. Span of authority C. Chain of command D. Unity of command E. Unity of control

A

The manager of the Conquest Casino notices that the security employees at the casino are treating customers impersonally. He sets a goal that they should make better eye contact and treat customers in a friendlier manner. This is a _______ goal. A. Behavioral B. Objective C. Task D. Project E. Procedural

A. Behavioral

Anna blinks her eyes whenever she looks up at the sun. This is an example of __________. A. Respondent behavior B. A learned reaction C. Behavioral contingency D. Operant behavior E. Law of effect

A. Respondent behavior

___ is based on a simple premise: Rightly or wrongly, people infer causes for their own and others' behavior.

Attribution theory

Which of the following is not a layer of diversity

Attributional

Melissa's manager set a goal that she should be able to produce 16 widgets per hour. This is a(n) ________ goal. A. Behavioral B. Objective C. Task D. Project E. Procedural

B. Objective

Affirmative action plans

Can negatively affect women and minorities

The two general categories of motivation theories are:

Content and process

___________ is achieved through formulation and enforcement of policies, rules, and regulations. A. Hierarchy of authority B. The strategic plan C. Aligned goals D. Division of labor E. Coordination of effort

E

The characteristic of PsyCap that consists of having self-confidence is

Efficacy

___ are complex, relatively brief responses aimed at a particular target

Emotions

Subcultures often form around

Functional or occupational groups or work roles Divisions or departments Geographical areas Products, markets, technology Levels of management

A person in the ______ role will encourage all group members to participate.

Gatekeeper

Which of the following is not an appropriate measure of goal achievement?

Lack of interpersonal conflict

___ intelligence will be most closely related to a performance on tests like the Scholastic Aptitude Test and the Graduate Management Admissions Test.

Logical-mathematical

Exchange (hard)

Making express or implied promises and trading favors

Dr. Riley, a professor in the economics department, finds that if he keeps his door open, people tend to stop in to say hello. This interruption is distracting. So now Dr. Riley keeps his office door open only a crack during his office hours, asking visitors to knock before entering. This small change allows him to grade exams more quickly and accurately. This is an example of his increasing his:

Mindfulness

Sales quotas and piecework are best measured using _______ goals.

Objective

Formal groups perform two basic functions: __________ and __________.

Organization;indivdual

The ______ keeps the group headed toward its stated goals.

Orienter

Angela works for a company that does not provide good health benefits or career opportunities. Her ___ is most likely to be negative.

POS (Perceived Organizational Support)

_______ is the popular term for linking monetary incentives with results or accomplishments.

Pay for performance Explanation Pay for performance is the popular term for monetary incentives linking at least some portion of the one's pay directly to results or accomplishments.

The two basic types of goals are:

Performance and Learning

___, with which you can build goodwill, include personality, teamwork and leadership.

Personal Attributes

Tom seems to want to control others and often manipulates people for his own gratification. He exhibits the need for:

Personal power

Mari has been out of work for eight months and is worried about providing for her family. She takes a job at a fast-food restaurant, even though the position is a step down from her previous job. According to Maslow's theory, Mari is motivated by ____ needs.

Physiological

In Maslow's needs hierarchy theory, ___ is the lowest level and ___ is the highest level

Physiological; self- actualization

Prosocial behaviors are:

Positive acts performed without expecting anything in return

Other Personality Traits

Positive affective people experience positive moods more frequently. Negative affective people look at the glass as half empty and experience anxiety and nervousness.

Participate Management Benefits

Predicted to increase motivation because it helps employees fulfill three basic needs: (1) autonomy, (2) meaningfulness of work, and (3) interpersonal contact.

___ is associated with entrepreneurialism.

Proactivity

_____ is the set of processes and managerial behaviors that involve defining, monitoring, measuring, evaluating, and providing consequences for performance expectations.

Professional management

Marion, a shift supervisor, is keeping a performance diary for each of there employees. This mechanism can ___ performance appraisals.

Reduce Bias in

Personal Appeals (soft)

Referring to friendship and loyalty when making a request

Person-Organization Fit

Refers to the degree to which a person's values, personality, goals and other characteristics match those of the organization

What kind of a response is a manager making when he or she chooses a satisfactory, but not ideal, solution?

Resolving

The process by which a firm attempts to identify job applicants with the necessary knowledge, skills, abilities, and other characteristics that will help the firm achieve its goals is known as ________.

Selection

Other Personality Traits Pt. 2

Self Monitoring refers to the extent to which a person is capable of monitoring his or her actions and appearance in social situations.

___ is knowing who you are and what you want

Self-awareness

Trevor is an overachiever. This brother calls him ambitious and power hunger. Trevor exemplifies the ___ dimension.

Self-enhancement

___ is your general belief about your own self-worth

Self-esteem

Passive

Shawn is a member of the student ethics committee. In meetings, he sometimes appears to be paying attention, but at other times does not appear to be tracking the conversations. He rarely contributes to the discussions. He is exhibiting the __________ style of listening.

Person factors and ___ are the two broad categories of OB concepts and theories.

Situation Factors

Problem solving and critical thinking are ___ because they use logic and reasoning to develop and evaluate options.

Soft skills

Halim hires a new employee who best meets the characteristics he is looking for in the ideal employee. He is

Solving the Problem

Coalition

Temporary groupings of people who actively pursue a single issue.

Self Effacement Bias

Tendency for people to underestimate their performance, undervalue capabilities and see events in a way that puts them in a more negative light

Self Perception Bias

Tendency to overestimate our performance and capabilities and see ourselves in a more positive light then others do.

Commitment

The best possible influence outcome is __________ because the target person's intrinsic motivation energizes good performance.

self-interest.

The critical aspect of organizational politics is the emphasis on:

face-to-face exchanges.

The richest form of communication is:

According to ___, the typical perspective held by managers is that employees dislike work, must be monitored, and can be motivated only with rewards and punishment ("carrots and sticks")

Theory X

Prosocial behaviors are: According to _________, the typical perspective held by managers is that employees dislike work, must be monitored, and can be motivated only with rewards and punishment ("carrots and sticks").

Theory X

At Cornet Company, employees and teams had higher performance, greater job satisfaction, and increased organizational commitment when their managers showed ___ behaviors.

Theory Y

use a spell-checker.

To manage e-mail effectively, an employee should:

Brenda has two exams and a research paper scheduled within the next 10 days. She creates a study plan in which she studies an hour a day for each exam and devotes an addition 90 minutes to her research paper. On Tuesday and Wednesday, Brenda is invited to go out with her friends. Instead, she stays at home and sticks with her study plan because she knows this will help her learn more effectively.

True

In equity theory, you compare your outputs and inputs to those of another person

True

False Consensus Error

We overestimate how similar we are to others

starting with the most controversial elements.

When involved in a crucial conversation, a manager should avoid:

Positive legitimate, expert, and referent

_________ power tend to produce commitment.

Negative legitimate, coercive, and reward

__________ power tend to produce compliance.

1. Which of the following is not a process theory of motivation? a. Acquired needs b. Equity c. Justice d. Expectancy e. Goal-setting

a. Acquired needs

1. Folding arms, crossing hands, and crossing legs communicate: a. defensiveness. b. immediacy. c. liking. d. happiness. e. boredom.

a. defensiveness.

1. An individual's belief that a particular degree of effort will be followed by a particular level of performance is called a(an): a. expectancy. b. expectation. c. instrumentality. d. valence. e. value.

a. expectancy.

1. The first step in the TED five-step protocol for effective presentations is: a. frame your story. b. plan your delivery. c. select your medium. d. develop your stage presence. e. control your stress.

a. frame your story.

Values represent beliefs that influence behaviors ________; attitudes relate to behavior ________.

across all situations; toward specific targets

The president of American Systems announces that all employees will be going on a trip to San Francisco. Some employees like the idea and some don't. According to the _________ component of attitudes, these evaluations reflect the feelings of the employees.

affective Explanation The affective component of an attitude contains the feelings or emotions one has about a given object or situation.

Positive practices that escalate positive outcomes because of their associations with positive emotions and social capital are having a(n) __________ effect.

amplifying

Ryan is looking to buy a used car. He seems to be most concerned with the color and style of the car, and uses those criteria as a basis for evaluating the value of the car, rather than considering how well the engine or the transmission is maintained. This is an example of

anchoring bias. Explanation Anchoring bias occurs when decision makers are influenced by the first information they receive about a decision, even if it is irrelevant.

I will perceive negative inequity when: another's total rewards are greater than rewards I receive. another's ratio of outputs to inputs is equal to my ratio of my outputs to my inputs. another's total input is less than my total input. another's ratio of outputs to inputs exceeds my ratio of my outputs to my inputs. another's ratio of outputs to inputs is less than my ratio of my outputs to my inputs.

another's ratio of outputs to inputs exceeds my ratio of my outputs to inputs

Negative emotions

are limiting. Explanation Negative emotions spur you to act in quite narrow or specific ways, thereby limiting you.

Negative emotions:

are limiting`

Which of these would you not see from an inspiring transformational leader?

authoritarian leadership Explanation Figure 13.5 provides a sketch of how transformational leaders rely on these four key leader behaviors (second column from left): inspirational motivation, idealized influence, individualized consideration, and intellectual stimulation.

1. The age group that is most likely to utilize social media is: a. 13-18. b. 19-29. c. 30-49. d. 50-64. e. 65+.

b. 19-29.

The concept of ______ can be applied to appraisals to increase the perception that employees are treated fairly. a. Selective evaluation b. Due process c. Multiple raters d. Selective retention e. Counterproductivity

b. Due process

________ ranking requires the evaluator to place employees into a particular classification, such as top one-fifth or second one-fifth. a. Individual b. Group Order c. Paired d. Fractional e. Percentile

b. Group Order

1. Brian has a Master's degree in electrical engineering and brings 10 years of experience to his current job. He is known for putting in long hours of hard work on his job. He receives a salary of $75,000, medical and dental coverage, and two weeks of paid vacation every year. From Brian's perspective, which of the following is an input in the individual-organization exchange relationship? a. His salary of $75,000 b. His Master's degree in electrical engineering c. Medical coverage d. Dental coverage e. Two weeks of paid vacation

b. His Master's degree in electrical engineering

1. What kind of goals is best for jobs with clear and readily measured outcomes? a. Behavioral b. Objective c. Task d. Project e. Reinforcement

b. Objective

1. In the TED five-step protocol for effective presentations, TED stands for: a. Technology, Education, Delivery. b. Technology, Education, Design. c. Teaching, Education, Development. d. Technique, Effort, Delivery. e. Technique, Education, Development.

b. Technology, Education, Design.

1. The exchange of information between a sender and a receiver, and the inference of meaning between the individuals involved, is known as: a. feedback. b. communication. c. encoding. d. decoding. e. channeling.

b. communication.

1. Chuck is concerned with what he considers to be an unfair situation at work. Although he put in 10 hours of overtime last week, he received the same pay as a co-worker who didn't put in any overtime. This week he decided to take an extra hour for lunch every day. Using the terminology of equity theory, Chuck has decided to ____. a. increase his efficiency b. decrease his inputs c. increase his outcomes d. increase his inputs e. decrease his outcomes

b. decrease his inputs

1. The characteristic of PsyCap that consists of having confidence is: a. hope. b. efficacy. c. effectiveness. d. health. e. resilience.

b. efficacy.

1. Ted was dissatisfied with his job. He said that the company policy, supervision, and working conditions were responsible for his dissatisfaction. According to Frederick Herzberg's theory, these extrinsic factors that create job dissatisfaction are called: a. motivators. b. hygiene factors. c. affiliation factors. d. wellness factors. e. achievement factors.

b. hygiene factors.

1. One of the inputs of positive OB is: a. amplification. b. mindfulness. c. flourishing. d. buffering. e. positive deviance.

b. mindfulness.

1. A theory that attempts to describe how various person and environmental factors affect motivation is known as a: a. content theory of motivation. b. process theory of motivation. c. extrinsic theory of motivation. d. intrinsic theory of motivation. e. integrated theory of motivation.

b. process theory of motivation.

When positive practices reduce the impact of negative events or stressors, a(n) _________ effect has occurred.

buffering Explanation Organizations can provide decision-making discretion, share information, minimize incivility, and provide feedback. Feedback can be both motivational and instructional, and therefore can include negative feedback.

When an appraiser rates employees based on items on a continuum with the points reflecting actual behave on a given job, this type of evaluation is called ______. a. Critical intent b. Group order ranking c. Bars d. Graphic ratings scale

c. BARS (BARS = Behavior Anchored Rating Scale)

5. ________ has been found to be more important than other traits in the success of CEOs of private equity companies, as well as other types of jobs a. Extroversion b. Agreeableness c. Conscientiousness d. Emotional stability

c. Conscientiousness

Which dimension of the big five model is a measure of reliability a. Extroversion b. Agreeableness c. Conscientiousness d. Emotional stability

c. Conscientiousness

1. Which of the following is not an example of a crucial conversation? a. A negative performance review b. Ending a relationship c. Conveying routine feedback d. Confronting a teammate who is frequently late to meetings e. Critiquing a colleague's report

c. Conveying routine feedback

1. __________ occurs when receivers receive a message. a. Feedback b. Encoding c. Decoding d. Understanding e. Noise

c. Decoding

The ______ phase of socialization involves the new recruit seeing what the organization is truly like. a. Anticipatory b. Normative c. Encounter d. Maintenance e. Change and acquisition

c. Encounter

1. In order to foster employee positivity, employers should do all but one of the following. Which one? a. Allow employees to make decisions. b. Share information with employees. c. Give employees only positive feedback. d. Make the well-being of employees a priority. e. Discourage uncivil behavior.

c. Give employees only positive feedback.

With reference to the Hofstede's framework, a class or caste system that discourages upward mobility is more likely to exist in a nation that score ______. a. High on individualism b. Low in masculinity c. High on power-distance d. High on uncertainty avoidance

c. High on power-distance

1. _______ involves putting more variety into a worker's job by combining specialized tasks of comparable difficulty. a. Job rotation b. Job enrichment c. Job enlargement d. Scientific management e. Vertical loading

c. Job enlargement

1. How an individual perceives the movement from performance to outcome is called: a. an expectancy. b. an expectation. c. instrumentality. d. a valence. e. a value.

c. instrumentality.

1. Touching another person communicates: a. defensiveness. b. immediacy. c. liking. d. happiness. e. boredom.

c. liking.

1. The _______ effect is the attraction of all living systems toward that which is life-giving and away from that which is life-depleting. a. buffering b. attentional c. positivity d. amplifying e. deviant

c. positivity

1. The type of goals that is best for jobs that are dynamic but in which nearer-term activities and milestones can be defined is: a. behavioral. b. objective. c. task. d. learning. e. reinforcement.

c. task.

1. Extrinsic rewards come from ________ and intrinsic rewards come from ________. a. the organization; coworkers b. the organization; the environment c. the environment; oneself d. the environment; society e. oneself; the organization

c. the environment; oneself

1. The most effective schedules of reinforcement are: a. continuous. b. Fixed interval and fixed ratio. c. variable ratio and variable interval. d. fixed interval and variable interval. e. fixed ratio and variable ratio.

c. variable ratio and variable interval.

By doing attitude surveys of all staff, Cue Manufacturing shows management that moral is quite low and, as a result, the risk to safety is quite high. Management knows they have to take action. They decide to modify employee shift hours and breaks. Once employees and managers begin to see how these actions are benefiting them, the company and those around them, they will begin to participate more fully in the transition. This occurs during the ________ stage of the Lewin model.

change Explanation The changing stage entails providing employees with new information, new behavioral models, new processes or procedures, new equipment, new technology, or new ways of getting the job done.

In the ________ phase of organizational socialization, employees master important tasks and roles and adjust to their work group's values and norms.

change and acquisition

XYZ, Inc., makes consumer products such as shampoo, conditioner, deodorant, and toothpaste. Recently, market research conducted at a university revealed that consumer tastes have changed, and consumers now want their products to come in pastel colors. Currently, XYZ's products all follow a deep blue packaging scheme. What source of change is this?

changing customer preferences Explanation This change is customer and market changes. Customers are increasingly sophisticated and demand the companies with whom they do business to deliver higher value products and services.

Which of the following is not a characteristic of a servant-leader?

charisma Explanation Servant-leadership characteristics are listening, empathy, healing, awareness, persuasion, conceptualization, foresight, stewardship, commitment to the growth of people, and building community.

Carla tends to take long-term view and rely on discussions with others to acquire information. She has a high tolerance for ambiguity and is oriented toward people and social concerns when making decisions. Which decision-making style does Carla possess?

conceptual Explanation People with a conceptual style have a high tolerance for ambiguity and tend to focus on the people or social aspects of a work situation. They take a broad perspective on problem solving and like to consider many options and future possibilities. Conceptual types adopt a long-term view and rely on intuition and discussions with others to acquire information. They also are willing to take risks and are good at finding creative solutions to problems. On the downside, however, a conceptual style can foster an idealistic and indecisive approach to decision making.

The styles of conflict handling are differentiated along two dimensions:

concern for others and concern for self. Explanation Figure 10.5 shows that five of the most common styles of conflict handling are distinguished based on relative concern for other (x-axis) and concern for self (y-axis).

Ethan observes that his subordinate, Nate, is performing and acting like the other members of the unit. He is observing

consensus. Explanation Consensus compares an individual's behavior with that of his or her peers. There is high consensus when one acts like the rest of the group and low consensus when one acts differently.

Samuel values his tradition and conformity. Not surprisingly, family time is very important to him. His employer, Stillman & Sons, expects him to work on holidays. Stillman & Sons and Samuel do not share the values of

conservation. Explanation See Figure 2.2. Samuel and his company do not appear to value conformity, tradition, and security (which comprise conservation) equally.

Josef, a manager, reviews Arturo's performance over the past six months and notes that during February he was late to work five times. He was not late during any other months. Josef is observing

consistency. Explanation Consistency judges if the individual's performance on a given task is consistent over time. Low consistency is undesirable for obvious reasons, and implies that a person is unable to perform a certain task at some standard level. High consistency implies that a person performs a certain task the same way, with little or no variation over time.

In the leniency perceptual error, a person

consistently evaluated people in an extremely positive fashion

Using ______ is a way of getting others to participate in planning, making decisions, and changes.

consultation Explanation Consultation is getting others to participate in planning, making decisions, and changes.

People who are high on openness to experience are likely to demonstrate

curiosity and broad-mindedness

1. Successful performance that dramatically exceeds the norm in a positive direction is known as: a. organizational citizenship behavior. b. organizational commitment. c. social deviance. d. positive deviance. e. positive amplification.

d. positive deviance.

1. Unlearned reflexes are: a. operant behavior. b. consequential behavior. c. reward behavior. d. respondent behavior. e. contingent behavior.

d. respondent behavior.

Instrumental values

deal with views on acceptable modes of conduct, such as being honest and ethical, and being ambitious.

Programmed decisions

decisions that occur frequently enough that we develop an automated response to them

Jim is head of the marketing department and must decide how to allocate a fixed amount of funds. There are two top projects that Jim believes hold the most promise. Recently, the majority of funds have been allocated to Project A, but Jim wonders if it is time to provide Project B with a larger share of the money. Jim decides to assign a particular individual, Clara, to thoroughly criticize the proposal for Project B, looking for every possible flaw. Which technique is Jim using to help him make a better decision?

devil's advocacy Explanation Jim is using devil's advocacy to help him make a better decision. Devil's advocacy involves assigning someone the role of critic.

Under ________, intent is irrelevant.

disparate impact

Queen Electric is structured around five primary business divisions: Queen Power, Queen Oil & Gas, Queen Energy Connections, Queen Transportation, and Queen Digital. These business areas are subdivided further into either product or geographic divisions. What kind of structure has Queen Electric adopted?

divisional Explanation In a divisional structure, employees are segregated into organization groups based on similar products or services, customers or clients, or geographic regions.

2. The age group that is least likely to utilize social media is: a. 13-18. b. 19-29. c. 30-49. d. 50-64. e. 65+.

e. 65+.

Perception is the process that

enables us to interpret and understand our surroundings

Organizations with market cultures are described as

externally focused and valuing stability and control

Ted was dissatisfied with his job. He said that the company policy, supervision, and working conditions were responsible for his dissatisfaction. According to Frederick Herzberg's theory, these extrinsic factors that create job dissatisfaction are called: affiliation factors. hygiene factors. wellness factors. motivators. achievement factors.

hygiene factors

Which of the following is NOT an outcome likely to result from positive emotions?

loss of focus

The theory of motivation that states that motivation is a function of five basic needs that are met sequentially is: herzberg's motivator-hygiene. extrinsic and intrinsic motivation. acquired needs. maslow's need hierarchy. theory X and Theory Y.

maslow's need hierarchy

The _______ for Mortimer, Inc. is "We build the world's best mousetraps."

mission statement Explanation Mission statements represent the reason organizations exist.

Escalation of commitment

occurs when individuals continue on a failing course of action after information reveals it may be a poor path to follow

Teams with high levels of ______ and ______ deal with task conflict better than those without these characteristics.

openness; emotional stability Explanation Team research shows that teams with members who possess high levels of openness and emotional stability deal with task conflict better than those without these characteristics.

In McClelland's acquired needs theory,

we learn needs as we live our lives

Power Outcomes

• Expert and referent power had a generally positive impact. • Reward and legitimate power had a slightly positive impact. • Coercive power had a slightly negative impact.

3 Possible Influence Outcomes

1. Commitment: Your friend enthusiastically agrees and will demonstrate initiative and persistence while completing the assignment. 2. Compliance: Your friend grudgingly complies and will need prodding to satisfy minimum requirements. 3. Resistance: Your friend will say no, make excuses, stall, or put up an argument.

Greater delegation was associated with the following factors:

1. Employee was competent. 2. Employee shared manager's task objectives. 3. Manager had a long-standing and positive relationship with employee. 4. The lower-level person also was a supervisor.

Political maneuvering is triggered primarily by uncertainty. Five common sources of uncertainty within organizations are:

1. Unclear objectives. 2. Vague performance measures. 3. Ill-defined decision processes. 4. Strong individual or group competition.76 5. Any type of change.

___ is a process that involves an individual comparing perceptions of his or her of performance with those of managers, subordinates and peers.

360-degree feedback

________ is a process that involves an individual comparing perceptions of his or her of performance with those of managers, subordinates, and peers.

360-degree feedback Explanation In 360-degree feedback individuals compare perceptions of their own performance with behaviorally specific performance information from their manager, subordinates, and peers.

A fabric design company is known for the designs it develops, owing to its creative talent and knowledge of the market. This firm uses partner organizations to take orders, ship products, and manage employee benefits. The company has a _____ structure. A. hollow B. modular C. matrix D. horizontal E. virtual

A

According to Kaplan and Norton, a(n) ________ is a "visual representation of a company's critical objectives and the crucial relationships among them that drive organizational performance." A. strategy map B. external process C. corporate plan D. resource acquisition E. goal accomplishment

A

Martha wants her house fully cleaned on Monday. She takes the dusting, Marvin vacuums, Kit does the bathrooms, and Tina cleans the kitchen. ___________ occurs when the common goal is pursued by individuals performing separate but related tasks. A. Division of labor B. Hierarchy of authority C. Unity of command D. Span of control E. Traditional structure

A

Mosco's is a ____________ in which management has largely succeeded in breaking down barriers among internal levels, job functions, and departments, as well as reducing external barriers between the organization and those with whom it does business. A. boundaryless organization B. learning organization C. matrix structure D. clan culture E. divisional structure

A

Sunshine Crackers introduces Wheat Flats,a cracker with more fiber, fewer calories, and great flavor. Wheat Flats is a great success. This is an example of __________ innovation. A. product B. process C. resource D. culture E. capital

A

The ____________ translates an organization's vision and strategy into a comprehensive set of performance measures that provides the framework for a strategic measurement and management system. A. balanced scorecard B. internal processes C. resource acquisition D. strategy map E. organization chart

A

While the rest of the academia is suffering from labor shortages, WhatsUp University averages five applicants for every position opening. With this ratio of applicants to openings, the university can pick and choose from among the top candidates. The organization is known to excel at meeting expectations. WhatsUp University is successful, most likely, because of what balanced scorecard perspective? A. internal processes B. customer C. external processes D. goal accomplishment E. financial

A

________ is the creation of something new that makes money and finds a pathway to the consumer. A. Innovation B. Invention C. Creativity D.Integration E. Inspiration

A

__________ rely on a vertical hierarchy and attempt to define clear departmental boundaries and reporting relationships. A. Traditional structures B. Horizontal designs C. Open designs D. Boundaryless organizations E. Learning organizations

A

__________ typically have a more open communication and contribution style. For example, at Google, the environment is unpredictable, but because of the freedom afforded the employees and management, it runs smoothly. A. Organic organizations B. Closed organizations C. Market cultures D. Virtual structures E. Functional organizations

A

Simple

A decision environment in which the context is stable, with clear cause-and-effect relationships, is categorized as:

Consensus

A decision making rule that groups may use when the goal is to gail support for an idea or plan of action. This decision making rules is inclusive, participatory, cooperative and democratic

Delphi technique

A group process that utilizes written responses to a series of questionnaires instead of physically bringing individuals together to make decisions

Reward Power

A manager has reward power to the extent that he or she obtains compliance by promising or granting rewards. On-the-job behavior shaping, for example, relies heavily on reward power.

resistance.

A person who is passive-aggressive in reaction to another's influence attempt is exhibiting:

Rational decision-making model

A series of steps that decision makers should consider if their goals is to maximize their outcome and make the best choice: 1) Identify problem 2) Establish decision criteria 3) Weight decision criteria 4) Generate alternatives 5) Evaluate alternatives 6) Choose best decision and 7) Implement decision

Nominal group technique (NGT)

A technique designed to help with group decision making by ensuring that members participate fully

Groupthink

A tendency to avoid critical evaluation of ideas the group favors.

According to _____, leaders must learn to manipulate or influence the leadership situation in order to create a "match" between their leadership style and the amount of control within the situation at hand. A) Fiedler's contingency model B) path-goal theory C) trait theory D) LMX model E) expectancy theory

A) Fiedler's contingency model Feedback: Learning Objective: 14-02 Topic: Situational Theories Blooms: Remember AACSB: Analytic Level of Difficulty: Easy Page: 370 Explanation: Fiedler's contingency model suggests that leaders must learn to manipulate or influence the leadership situation in order to create a "match" between their leadership style and the amount of control within the situation at hand.

Followers want organizational leaders to create which of the following emotional responses in others? A) Significance B) Dominance C) Self-confidence D) Consideration E) Intuition

A) Significance Feedback: Learning Objective: 14-07 Topic: Additional Perspectives on Leadership Blooms: Remember AACSB: Analytic Level of Difficulty: Easy Page: 390 Explanation: Followers want organizational leaders to create feelings of significance (what one does at work is important and meaningful).

Which of the following is not a benefit of open structures? A. high control of outsourced operations B. focus on what they do best C. work well when trusted and suitable partners are available D. high control of outsourced operations E. work well when efficiency is very important F. work well when trusted and suitable partners are available G. potentially better access to specialists H. work well when efficiency is very important I. potentially better access to specialists

A,D

The type of goal most relevant for knowledge work is: A. Behavioral B. Objective C. Task D. Project E. Reinforcement

A. Behavioral

What kind of goals can be used in most jobs? A. Behavioral B. Objective C. Task D. Project E. Reinforcement

A. Behavioral

_________ is removing consequences from an undesirable behavior. A. Extinction B. Punishment C. Negative reinforcement D. Respondent behavior E. Operant behavior

A. Extinction

Carolyn is a very likable employee. Recently, her boss rated her at the highest level of all areas of performance, even though in some areas her performance did not warrant this judgment. This is an example of a ___________ perceptual error. A. Halo B. Leniency C. Central tendency D. Recency E. Contrast

A. Halo

Forming an overall impression about a person or object and then having that impression bias ratings about that person or object is known as the ________ perceptual error. A. Halo B. Leniency C. Central tendency D. Recency E. Contrast effects

A. Halo

In the organizing framework A. Inputs lead to processes, which lead to outcomes B. Inputs lead directly to outcomes C. Outcomes never affect inputs D. Outcomes never affect processes E. Inputs are unrelated to processes

A. Inputs lead to processes, which lead to outcomes

A head chef of a restaurant always encouraged his subordinates to be creative in the culinary art. He did not believe in using prescribed recipes while cooking. He allowed them to create unique recipes and use different ingredients to prepare dishes. This implied that the head chef established _________ goals. A. Learning B. Procedural C. Performance D. Equity E. Project

A. Learning

Nora wants to be a buyer for marstons stores. She knows that they are looking for outgoing social people. She purposely tries to present herself as an extravert on the personality test that she takes even though she is shy and retiring. What is the likely result of this? A. Nora might get the job, but find she is unhappy in the role B. Nora will not get the job. It is impossible to "fake" your personality on a test C. Nora will get the job. The test is simply pro forma and no one looks at the results D. The test will show that Nora is unstable E. The test will flag Nora as someone who is not truthful and should not be hired by the company

A. Nora might not get the job, but find she is unhappy in the role

Behavior that is learned when one acts on the environment to produce desired consequences is known as: A. Operant behavior B. Consequential behavior C. Reward behavior D. Respondent behavior E. Contingent behavior

A. Operant behavior

When Eileen sees that the sun is shining brightly, she puts on sunglasses. This is: A. Operant behavior B. Consequential behavior C. Reward behavior D. Respondent behavior E. Contingent behavior

A. Operant behavior

Matt completed his assignments on time as his teacher had said that she would allow extra play time to children who submit their assignments on or before time. Which of the following made Matt complete his assignments on time? A. Positive reinforcement B. Negative reinforcement C. Respondent behavior D. Unlearned reflexes E. Intermittent reinforcement

A. Positive reinforcement

The process of strengthening a behavior by contingently presenting something pleasing is known as: A. Positive reinforcement B. Negative reinforcement C. Extinction D. Punishment E. Respondent behavior

A. Positive reinforcement

To minimize halo perceptual error, a manager should: A. Record examples of positive and negative performance throughout the year B. Be fair and realistic in evaluations C. Define an accurate profile with both high and low points D. Evaluate employees against a predetermined standard E. Provide only negative feedback

A. Record examples of positive and negative performance throughout the year

The steps of implementing a goal-setting program are, in order: A. Set the goal, promote goal commitment, provide support and feedback, and create action plan B. Set the goal, provide support and feedback, promote goal commitment, and create the action plan C. Set the goal, provide support and feedback, create an action plan, and promote goal achievement D. Set the goal, create the action plan, promote goal commitment, and provide support and feedback E. Set the goal, create the action plan, provide support and feedback, and promote goal commitment

A. Set the goal, promote goal commitment, provide support and feedback, and create action plan

Papa Bill's, a local pizza chain, instituted a new compensation policy to increase the on-time delivery of its pizzas. Drivers were paid an extra $1.00 for each on-time delivery, payable on the next pay period. However, a review of the policy showed that traffic tickets for reckless driving among delivery drivers had increased by 50 percent since the new policy was introduced. Which of the following best explains why this reward system did not perform as expected? A. The organization was unintentionally rewarding counterproductive behavior B. Employees viewed the $1.00 incentive as an entitlement C. The delay was too long between the performance and the reward D. The organization placed too much emphasis on monetary rewards E. The reward lacked an "appreciation effect"

A. The organization was unintentionally rewarding counterproductive behavior

When monitoring performance, a manager should pay attention: A. To both progress toward goal achievement and the ultimate level of goal achievement B. Only to progress toward goal achievement C. Only to ultimate goal achievement D. Primarily to subordinates' reactions to the goal E. Primarily to the "bottom line"

A. To both progress toward goal achievement and the ultimate level of goal achievement

resistance.

ABC Company purchases new safety glasses for its machine operators. Unfortunately, the equipment makes the operators very uncomfortable on hot days. Davey removes the equipment when the foreman is not around. This is an example of:

Social Power

Ability to get things done with human, informational, and material resources.

Which of the following is not one of the most frequently experienced positive emotions?

Accomplishment

The components of attitudes include

Affective, cognitive, and behavioral

___ is an artificial intervention aimed at giving management a chance to correct an imbalance, injustice, mistake, or outright discrimination that occurred in the past

Affirmative Action

Which one of the following is not a difference between affirmative action and diversity management?

Affirmative action only refers to mandatory programs.

Coercion

Alexa, a manager, explains to Harvey that if he is late to work one more time in the next six weeks he will be suspended without pay for a day. Alexa is exercising _______ power.

Consultation

Alexis, a customer service supervisor at ABC Airlines, typically seeks the full participation of her team members when planning, making important decisions, and implementing changes. Which one of the nine generic influence tactics is she using?

Referent Power

Also called charisma, referent power comes into play when one's personality becomes the reason for compliance. Role models have referent power over those who identify closely with them.

________ are the phases of organizational socialization.

Anticipatory, encounter, and change and acquisition

________ is an alternative dispute resolution technique in which disagreeing parties agree ahead of time to accept the decision of a neutral party in a formal courtlike setting, often complete with evidence and witnesses.

Arbitration Explanation Arbitration is an alternative dispute resolution technique in which disputing parties agree ahead of time to accept the decision of a neutral arbitrator in a formal courtlike setting, often complete with evidence and witnesses.

E-mail

As HR Director at a large manufacturing firm, it is Henry's responsibility to ensure that employees are properly compensated according to a biweekly time schedule. Many of Henry's employees chose to have their pay checks automatically deposited into their designated bank accounts. However, it is still necessary to inform these workers that their pay has been properly deposited. Which of the following communication media should Henry choose to convey this information?

faking,

As a result, the rankings of the candidates who take the test may be affected by their ability to fake desired qualities. Some experts believe that this is a serious problem. In fact, it is estimated that 20-50% of all job applicants give fake responses in personality tests.It is even possible that the ability to fake is related to a personality trait that increases success at work, such as self monitoring. This issue raises potential questions regarding whether personality tests are the most effective way of measuring candidate personality. Scores are not only distorted because of some candidates faking better than others. Therefore, using self-report measures of performance may not be the best way of measuring someone's personality, whereas asking our former colleagues or supervisors may reveal more accurate answers.

Management at the Relaxing Pines resort chain places great emphasis on customer satisfaction and loyalty. They emphasize quiet, luxurious rooms and personalized service. In fact, bonuses are based primarily on customer evaluations. What perspective does this reflect?: A. financial B. customer C. internal processes D. goal accomplishment E. organizational culture

B

Many new companies and small businesses opt for a(n) __________ because they have a limited amount of employees, A. traditional structure B. horizontal structure C. open design D. boundaryless organization E. learning organization

B

One of the problems that Featherstock Products faces is the coordination and communication across silos. Featherstock uses a ________ organization. A. matrix B. functional C. hollow D. learning E. modular

B

The ________ organization structure is best for situations where the nature of product modules and design interfaces can be specified. A. hollow B. modular C. matrix D. horizontal E. virtual

B

Tsunamet Cosmetics is highly responsive to its customers. This is one of the advantages of _________ organizations. A. functional B. horizontal C. divisional D. hollow E. virtual

B

Wayneslock, when producing the Steamer Cabin Craft luxury yacht, assigned subcontractors to complete specific subassemblies and deliver them to Wayneslock Corporate for final assembly. This approach resulted in a leaner and simpler assembly line and lower inventory. Wayneslock is an example of a __________ structure. A. hollow B. modular C. matrix D. horizontal E. virtual

B

When a business doesn't have control of all the environmental forces, it is called __________ design. For example, Acme Medical relies on predictions and contingencies to cope with unexpected input. Last year, an influenza epidemic affected suppliers, personnel, and even customers, causing lost production and lost profit. A. traditional B. open C. horizontal D. functional E. divisional

B

Which of the following cannot be viewed in an organizational chart? A. hierarchy of authority B. organizational culture C. division of labor D. span of control E. line and staff positions

B

Which of the following is not one of the common denominators of organizations? A. coordination of effort B. profit motive C. hierarchy of authority D. division of labor E. aligned goals

B

Which of the following statements about mechanistic organizations is true? A. Mechanistic organizations generally have open structures. B. Mechanistic organizations generally are expected to produce reliability and consistency in internal processes. C. Mechanistic organizations result in lower efficiency, quality, and timeliness. D. Mechanistic organizations are more responsive to customer feedback. E. Mechanistic organizations are inappropriate in industries where mistakes and errors can be catastrophic

B

Which of the following statements about span of control is true? A. Larger organizations tend to have wider spans of control. B. Costs tend to be higher in organizations with narrow spans. C. Complex tasks tend to require a narrower span of control. D. Narrow spans of control are mostly found in clan cultures. E. Senior-level executives tend to have wider spans of control

B

XYZ Corporation set a number of objectives for the quarter, including increasing productivity by 10 percent and reducing turnover by 20 percent. As a whole, the organization accomplished both these objectives. XYZ Corporation is successful according to which balanced scorecard perspective? A. financial B. internal processes C. customer D. learning and growth E. strategic constituencies satisfaction

B

________ is the idea that organizations tend to be more effective when they are structured to fit the demands of the situation. A. Virtualness B. Contingency approach C. The closed systems theory D. A mechanistic approach E. A cultural fit process

B

_____ model is based on the assumption that leaders develop unique one-to-one relationships with each of the people reporting to them. A) Path-goal B) Leader-member exchange C) Laissez-faire D) Fiedler's contingency E) Behavioral styles

B) Leader-member exchange Feedback: Learning Objective: 14-05 Topic: Additional Perspectives on Leadership Blooms: Remember AACSB: Analytic Level of Difficulty: Easy Page: 383 Explanation: The leader-member exchange (LMX) model is based on the assumption that leaders develop unique one-to-one relationships with each of the people reporting to them.

_____ leaders can produce significant organizational change and results as this form of leadership fosters higher levels of employee engagement, trust, and loyalty from followers than any other leadership. A) Transactional B) Transformational C) Laissez-faire D) Implicit E) Explicit

B) Transformational Feedback: Learning Objective: 14-04 Topic:The Full-range Model of Leadership: From Laissez-faire to Transformational Leadership Blooms: Remember AACSB: Analytic Level of Difficulty: Easy Page: 378 Explanation: Transformational leaders can produce significant organizational change and results because this form of leadership fosters higher levels of employee engagement, trust, and loyalty from followers than does transactional leadership.

Researchers at Ohio State University determined that there are only two independent dimensions of leader behavior: _____ and: A) compliance; authority. B) consideration; initiating structure. C) assertiveness; task competence. D) biophysical traits; personal traits. E) emotional intelligence; supportiveness.

B) consideration; initiating structure. Feedback: Learning Objective: 14-01 Topic: Trait and Behavioral Theories of Leadership Blooms: Remember AACSB: Analytic Level of Difficulty: Easy Page: 368 Explanation: Researchers at Ohio State University determined that there are two independent dimensions of leader behavior: consideration and initiating structure, which yielded four behavioral styles of leadership.

Jarvis thinks that a good leader should be assertive without being aggressive. He also assumes that people who are willing to help always make better leaders than those who do not. He believes in the: A) servant-leadership. B) implicit theory of leadership. C) behavioral theory of leadership. D) "great man" theory of leadership. E) trait theory of leadership.

B) implicit theory of leadership. Feedback: Learning Objective: 14-01 Topic: Trait and Behavioral Theories of Leadership Blooms: Apply AACSB: Analytic Level of Difficulty: Medium Page: 365 Explanation: Implicit leadership theory is based on the idea that people have beliefs about how leaders should behave and what they should do for their followers.

A(n) _____ is a mental representation of the traits and behaviors that people believe are possessed by leaders. A) leader trait B) leadership prototype C) initiating structure D) contingency factor E) consideration

B) leadership prototype Feedback: Learning Objective: 14-01 Topic: Trait and Behavioral Theories of Leadership Blooms: Remember AACSB: Analytic Level of Difficulty: Easy Page: 365 Explanation: A leadership prototype is a mental representation of the traits and behaviors that people believe are possessed by leaders.

The _____ theory was originally proposed by Robert House in the 1970s. A) trait B) path-goal C) behavioral D) laissez-faire E) contingency

B) path-goal Feedback: Learning Objective: 14-03 Topic: Situational Theories Blooms: Remember AACSB: Analytic Level of Difficulty: Easy Page: 372 Explanation: Path-goal theory was originally proposed by Robert House in the 1970s.

The term "_____" was coined by Robert Greenleaf in 1970 and is more a philosophy of managing than a testable theory. A) shared leadership B) servant-leadership C) transformational D) transactional E) implicit

B) servant-leadership Feedback: Learning Objective: 14-06 Topic: Additional Perspectives on Leadership Blooms: Remember AACSB: Analytic Level of Difficulty: Easy Page: 386 Explanation: The term servant-leadership was coined by Robert Greenleaf in 1970 and is more a philosophy of managing than a testable theory.

The "law of effect" states that: A. Whatever can go wrong, will go wrong B. Behavior with favorable consequences tends to be repeated C. People are motivated by punishment D. Garbage in, garbage out E. Leadership affects how people perform

B. Behavior with favorable consequences tends to be repeated

Dr. Davis, an accounting professor, was concerned that his classes were not engaging his students as much as he would like. He asked his boss, the department chair, and two of his colleagues to sit in on one of his classes this term, to give him comments and suggestions for improvement. He also passed out forms to his students at the end of the term, asking the same. He was __________. A. Providing positive reinforcement for himself B. Conducting a 360 degree feedback process C. Developing an action plan D. Creating a contrast effect E. Conducting research

B. Conducting a 360 degree feedback process

Kelley hypothesized that people make casual attributions by observing A. External and internal factors and perceptions B. Consensus, distinctiveness, and consistency C. Effort, consistency, and ability D. Implicit and explicit factors E. Stereotypes and implicit cognition

B. Consensus, distinctiveness, and consistency

Organizations typically rely on __________ schedules. A. Continuous B. Fixed interval and fixed ratio C. Variable ratio and variable interval D. Fixed interval and variable interval E. Fixed ratio and variable interval

B. Fixed interval and fixed ratio

Which of the following is not a type of appropriate measure of goal achievement? A. Timeliness B. Lack of interpersonal conflict C. Quantity D. Quality E. Financial measures

B. Lack of interpersonal conflict

CRF schedules are particularly useful when an employee is: A. Missing quotas B. Learning a new skill C. Frequently absent from work D. Performing far above expectations for his or her job E. Feeling entitled to receiving a reward

B. Learning a new skill

Dr. Slotsky is a well-liked professor. Last term, no one received less than a B in any of his courses. Dr. Slotsky may be suffering from _________ perceptual error. A. Halo B. Leniency C. Central tendency D. Recency E. Contrast effects

B. Leniency

Emily is part of a team working on a project. Veronica's role on this is to update and present status reports to management. Emily knows that Veronica's reports do not show the delays that have occurred, but she doesn't say anything because she does not want upper management more involved. Which of the following is emily exhibiting? A. Ill-conceived goals B. Motivated blindness C. Indirect blindness D. The slippery slope E. Overvaluing outcomes

B. Motivated blindness

The sales department of a mutual fund firm set goals for the next fiscal year to sell a certain number of mutual funds each month. The firm only set targets and not anything about how they will meet this target. These goals are ________ goals. A. Behavioral B. Objective C. Task D. Project E. Procedural

B. Objective

What kind of goals is best for jobs with clear and readily measured outcomes? A. Behavioral B. Objective C. Task D. Project E. Reinforcement

B. Objective

Mason is highly satisfied with his job. He receives consistently strong evaluations and volunteers to train new employees. Mason exhibits positive job performance and A. Community involvement B. Organizational citizenship behaviors C. Genetic components D. Withdrawal cognitions E. Personal motivation

B. Organizational citizenship behaviors

Which of the following is not something a manager should do when giving feedback? A. Provide feedback as soon as possible B. Provide feedback about an aspect of a job that is beyond the individual's control C. Be honest D. Keep feedback relevant by relating it to existing goals E. Provide specific, descriptive feedback

B. Provide feedback about an aspect of a job that is beyond the individual's control

The general criteria used for distributing rewards are: A. Extrinsic and intrinsic B. Results, behaviors and actions, and nonperformance considerations C. Positive reinforcement, negative reinforcement, punishment, and extinction D. Attraction, motivation, and retention E. Financial and nonfinancial

B. Results, behaviors and actions, and nonperformance considerations

Kelly is a successful attorney specializing in corporate law. While it has nothing to do with her field of employment, Kelly has always been interested in the history of religion. Using what little free time she has, she decided to take a course in comparative religion at the local university. She thinks that the course would be interesting and enjoyable. Kelly is not pursuing a degree in the subject, nor is she taking the class for a grade. What can you say about Kelly's motivation for taking the class? A. She has a high need for power. B. She is intrinsically motivated. C. She has a high need for affiliation. D. She is not serious about her career. E. She sees the potential that a degree in religion studies can help her advance in her job.

B. She is intrinsically motivated.

The generation described as workaholic, idealistic, competitive, and materialistic is

Baby Boomers

Legitimating tactics (hard)

Basing a request on one's authority or right, organizational rules or policies, or express, or implied support form superiors.

________ goals are most relevant for knowledge work.

Behavior

Dawn is at the playground with her four-year-old daughter. The father of another child is yelling and cursing at him. Dawn decides to tell the father to stop the verbal abuse or leave the playground, or she will call the authorities. Dawn's behavior reflects the ___ component of her attitude.

Behavioral

Dawn is at the playground with her four-year-old daughter. The father of another child is yelling and cursing at him. Dawn decides to tell the father to stop the verbal abuse or leave the playground, or she will call the authorities. Dawn's behavior reflects the ________ component of her attitude.

Behavioral

___ goals are most relevant for knowledge work.

Behavioral

The existence of implicit cognition leads people to make ___ decisions without realizing they are doing so.

Biased

________ is a stress-reduction technique that uses electronic monitors to train people to detect muscular tension.

Biofeedback Explanation Cognitive restructuring is a stress-reduction technique in which irrational or maladaptive thoughts are identified and replaced with those that are rational or logical with help of a counselor. See Table 16.4.

Len is the top swimmer on his school's team. Star athletes are likely to be high in ___ intelligence

Bodily-kinesthetic

According to research, the relationship between job performance and job satisfaction is best summarized by which of the following statements?

Both variables indirectly influence each other

A(n) _______ structure groups people according to the type of work they perform. A. divisional B. matrix C. functional D. horizontal E. open

C

A(n) _________ can hamper growth because the flow of information stays within the system and has no chance to interact with or build on knowledge from the outer environment. A. organic structure B. learning organization C. closed system D. divisional structure E. modular structure

C

In XYZ Aerospace, project engineers report both to a project manager for the particular work they are performing as well as the manager of engineering. This is a ________ organization. A. divisional B. hollow C. matrix D. functional E. horizontal

C

In a learning organization, the _________ subprocess leads to shared understanding among individuals and groups. A. information acquisition B. information interpretation C. knowledge integration D. organizational memory E. information distribution

C

One of the pros of _________ organizations is a lower cost of entry and overhead. A. matrix B. functional C. hollow D. learning E. divisional

C

Pierre is a foreman in a plant. He has 25 line workers who report to him on the night shift. From what you have read about organization charts, what do you know for sure? A. His subordinates enjoy less autonomy than others. B. His subordinates take part in the decision-making process. C. He has a wide span of control. D. His subordinates are closely supervised. E. He is encountering high administrative costs

C

Straight Line Co. faces heavy price competition and is under pressure to cut costs. Management has decided to farm out some processes to outside companies, but they are concerned that this choice might result in loss of in-house capacity to innovate. Straight Line is using a _____ structure. A. matrix B. functional C. hollow D. learning E. divisional

C

The _______ structure enables Jerome Financial to upgrade and promote excellence around core capabilities, so they can be very focused on developing functional expertise in specific disciplines, such as marketing, money management, and lending. A. divisional B. hollow C. matrix D. functional E. horizontal

C

Using __________, Sensotix emphasizes using the chain of command to make sure the right people do the right things at the right time. A. coordination of effort B. the span of control C. the hierarchy of authority D. a division of labor E. aligned goals

C

When organizations need stronger horizontal alignment or cooperation in order to meet their goals, a ________ structure is appropriate. A. divisional B. hollow C. matrix D. functional E. horizontal

C

________ organizational design organizes people around the workflow. A. Traditional B. Open C. Horizontal D. Mechanistic E. Closed

C

Federico loves to spend his free time helping coach a youth soccer team. He not only volunteers his time, but also actively fundraises to support the team with equipment and uniforms. This reflects the ____ value in Schwart's model A. Universalism B. Self-direction C. Benevolence D. Security E. Stimulation

C. Benevolence

Avoiding extreme judgments and rating people and objects as average or neutral is known as the _______ perceptual error. A. Halo B. Leniency C. Central tendency D. Recency E. Contrast effects

C. Central tendency

Janelle filled out a peer evaluation rating form for her teammates in her OB class. She rated everyone as a "3" or "4" on the 6-point scale. This is an example of ___________ perceptual error. A. Halo B. Leniency C. Central tendency D. Recency E. Contrast effects

C. Central tendency

The idea that a manager should do what the situation requires is captured by the term: A. Integrative framework B. Interactional perspective C. Contingency approach D. Performance management E. Performance appraisal

C. Contingency approach

If every instance of a target behavior is reinforced, a _________ schedule is in effect. A. Positive B. Negative C. Continuous D. Intermittent E. Variable

C. Continuous

Which of the following is not included in the monitoring step of performance management? A. Measuring performance B. Verifying progress C. Deciding how to measure performance D. Verifying achievement of goals E. Tracking performance

C. Deciding how to measure performance

To minimize central tendency bias, a manager should: A. Record examples of positive and negative performance throughout the year B. Be fair and realistic in evaluations C. Define an accurate profile with both high and low points D. Evaluate employees against a predetermined standard E. Provide only negative feedback

C. Define an accurate profile with both high and low points

The first step in effective performance management is: A. Providing consequences B. Reviewing performance C. Defining performance D. Evaluating performance E. Monitoring performance

C. Defining performance

Information about performance shared with those in a position to improve the situation is known as: A. Coaching B. Instruction C. Feedback D. Monitoring E. Measurement

C. Feedback

Which of the following is not a tip for increasing goal commitment and success? A. Reward yourself for progress, not just for ultimate goal achievement B. Break large goals down into sub-goals C. Keep your goals to yourself D. Write goals down E. List the benefits of achieving the goal

C. Keep your goals to yourself

The popular term for linking monetary incentives with results or accomplishments is: A. Pay for rewards B. Pay for play C. Pay for performance D. Pay for incentives E. Pay as you go

C. Pay for performance

Barry set himself a goal of bowling at least one game above 250 (out of 300 possible) before the end of the season. This is a ________ goal. A. Learning B. Procedural C. Performance D. Equity E. Project

C. Performance

The set of processes and managerial behaviors that involve defining, monitoring, measuring, evaluating, and providing consequences for performance expectations is known as: A. Performance appraisal B. Performance review C. Performance management D. Goal setting E. Coaching

C. Performance management

Which forms of consequence strengthen a desired behavior? A. Positive reinforcement and extinction B. Positive reinforcement and punishment C. Positive reinforcement and negative reinforcement D. Extinction and punishment E. Extinction and negative reinforcement

C. Positive reinforcement and negative reinforcement

Barney was late to work one morning, so he drove over the speed limit through a school zone. As a result, he got pulled over by a police officer and received a ticket. This scenario is an instance of __________. A. Positive reinforcement B. Negative reinforcement C. Punishment D. Extinction E. Shaping

C. Punishment

Janell is having an argument with her father. He believes that focusing on her computer knowledge (hard skills) is the most important of janell's college experience. Janell disagrees, arguing that ____ is/are the more critical and will help her in the hiring process. A. Problem solving B. Self-awareness C. Soft skills D. An ethical perspective E. Business knowledge

C. Soft skills

The type of goals that is best for jobs that are dynamic but in which nearer-term activities and milestones can be defined is: A. Behavioral B. Objective C. Task D. Learning E. Reinforcement

C. Task

Extrinsic rewards come from ________ and intrinsic rewards come from ________. A. The organization; coworkers B. The organization; the environment C. The environment; oneself D. The environment; society E. Oneself; the organization

C. The environment; oneself

The most effective schedules of reinforcement are: A. Continuous B. Fixed interval and fixed ratio C. Variable ratio and variable interval D. Fixed interval and variable interval E. Fixed ratio and variable ratio

C. Variable ratio and variable interval

Ingratiation

Carl needs a day off from his work. But he is apprehensive that David, his manager, will not grant the leave. Carl knows that David wants to shed a few kilos and has been hitting the gym for the past month. Before asking for the leave, Carl compliments David on his build and physique. It comes as no surprise to Carl that David grants him the leave! What kind of an influence tactic did Carl use?

___ are suspected or inferred causes of behavior

Causal Attributions

Chapter 1 Assessment

Chapter 1 Assessment

Chapter 10 Assessment

Chapter 10 Assessment

Chapter 11 Assessment

Chapter 11 Assessment

Chapter 12 Assessment

Chapter 12 Assessment

Chapter 13 Assessment

Chapter 13 Assessment

Chapter 14 Assessment

Chapter 14 Assessment

Chapter 15 Assessment

Chapter 15 Assessment

Chapter 16 Assessment

Chapter 16 Assessment

Chapter 2 Assessment

Chapter 2 Assessment

Chapter 3 Assessment

Chapter 3 Assessment

Chapter 4 Assessment

Chapter 4 Assessment

Chapter 5 Assessment

Chapter 5 Assessment

Chapter 6 Assessment

Chapter 6 Assessment

Chapter 7 Assessment

Chapter 7 Assessment

Chapter 8 Assessment

Chapter 8 Assessment

Chapter 9 Assessment

Chapter 9 Assessment

Which of the following is not a best practice for becoming more productive?

Check social media every two hours

Which of the following is not a best practice for becoming more productive?

Check social media every two hours. Explanation You should actually trade off social and gaming time. Whatever amount of time you spend playing video games or connecting on social media, try reallocating some of it to clearing e-mails or tackling other items on your to-do list.

______ is employees' shared perceptions of policies, practices, and procedures of an organization.

Climate Explanation Climate represents employees' shared perceptions of policies, practices, and procedures.

Tonya believes that cell phone use—even in public places like trains and restaurants—is acceptable because it helps people manage their busy lives. Tonya's belief reflects the ________ component of her attitude.

Cognitive

___ is the psychological discomfort a person experiences when simultaneously holding two or more conflicting cognitions.

Cognitive Dissonance

Which of the following is not one of the predominant models of the causes of job satisfaction?

Cognitive dissonance

an individual's ability to effectively use communication behaviors in a given context.

Communication competence is:

share credit.

Compared to men, women are more likely to:

The ________ framework provides a practical way for managers to understand, measure, and change organizational culture.

Competing values

Dr. Davis, an accounting professor, is concerned that he was not engaging his students as much as he would like. He asks his boss, the department chair, and two of his colleagues to sit in on one of his classes, to give him comments and suggestions for improvement. He also passes out student evaluation forms. Dr. Davis is:

Conducting a 360 degree feedback process

___ is the personality trait with the strongest positive relationship with job performance.

Conscientiousness

Samuel values his tradition and conformity. Not surprisingly, family time is very important to him. His employer, Stillman & Songs, expects him to work on holidays. Stillman & Sons and Sauel do not share the values of

Conservation

_____ is defined by order, self-restriction, preservation of the past, and resistance to change.

Conservation Explanation Conservation is defined by order, self-restriction, preservation of the past, and resistance to change (i.e., security, conformity, tradition).

"What are the different needs that activate motivation's direction, intensity, and persistence?" This question reflects which theories of motivation?

Content

"what are the different needs that activate motivation's direction, intensity, and persistence?" This statement reflects which theories of motivation

Content

________ is the idea that a manager should do what the situation requires.

Contingency approach

________ is the idea that a manager should do what the situation requires.

Contingency approach Explanation The contingency approach is to do what the situation requires, rather than applying a one-size-fits-all approach and simply doing what has always been done.

When a manager evaluates employees against a predetermined standard, he or she is minimizing _______ bias.

Contrast

When a manager evaluates employees against a predetermined standard, he or she is minimizing _______ bias.

Contrast effect

Morgan, who has been a salesperson for six months, receives a review that she feels is unfair. Her manager just completed Samuel's review yesterday. Samuel has been a top salesperson for four years. It's likely that her manager has committed a _______ perceptual error.

Contrast effects

The team is generating a lot of ideas on how to approach the management case study problem. Everyone seems to have a plan. Joe stands up and says, "I will make a list on the board of all of our ideas." Joe is acting as:

Coordinator

________ provides broad and useful ways to describe personality in terms of our individual differences in self-efficacy, self-esteem, locus of control, and emotional stability.

Core self-evaluation

People's with high ___ see themselves as capable and effective

Core self-evaluations

___ provides broad and useful ways to describe personality in terms of our individual differences in self-efficacy, self-esteem, locus of control, and emotional stability

Core-self evaluation

Mario is setting up an after-school program for inner city youth. The population is a mix of white, African-American and Latino, with Latinos comprising the majority of participants. Which of the following would help the group overcome stereotypes?

Create project opportunities using a mix of populations with equal status. Explanation Managers/leaders should create opportunities for diverse people to meet and work together in cooperative groups of equal status. Social scientists believe positive interpersonal contact among mixed groups is the best way to reduce stereotypes because it provides people with more accurate data about the characteristics of others.

Creative decisions

Creativity is the generation of new, imaginative ideas. It is the first step in the innovation process, but not the same thing. Step 1) Problem recognition 2) Immersion 3) Incubation 4) Illumination 5) Verification and application

_________ teams are created with members from different disciplines within an organization, such as finance, operations, and R&D.

Cross-functional Explanation Cross-functional teams are created with members from different disciplines within an organization, such as finance, operations, and R&D.

People who are high on openness to experience are likely to demonstrate

Curiosity and broad-mindedness

Happy Helpers provides caregiver services to their local community. The company's financial growth is dependent on getting more revenue from existing customers. To do this, management decides to A. merge with a caregiver group across the country. B. sell the business to a nationwide nursing service. C. invest in a pharmaceutical company. D. supply personal care services (such as hair styling, shopping, etc...) to its clients. E. handle Happy Helpers' computer services internally, rather than farming it out to a tech company

D

The ________ perspective focuses on providing employees with the capabilities, resources, and work environment they need to achieve their goals. A. goal accomplishment B. internal processes C. financial D. customer E. learning and growth

D

Which of the following is not a benefit of horizontal structures? A. faster and more efficient new product development B. encourage knowledge sharing C. greater creativity D. greater opportunities for functional specialization E. rapid communication and reduction of cycle time

D

________ is not a benefit of horizontal structures. A. Faster and more efficient new product development B. Encourage knowledge sharing C. Greater creativity D. Greater opportunities for functional specialization E. Rapid communication and reduction of cycle time

D

________ occur(s) when organizations divide the work in pursuing the common goal among groups and individuals performing separate but related tasks. A. Coordination of effort B. The chain of command C. The hierarchy of authority D. Division of labor E. Aligned goals

D

A process that involves an individual comparing perceptions of his or her of performance with those of managers, subordinates, and peers is called: A. Performance monitoring B. Performance appraisal C. Goal monitoring D. 360-degree feedback E. Self-assessment

D. 360-degree feedback

In the leniency perceptual error, a person: A. Forms an overall impression, and then uses that impression to bias ratings B. Avoids all extreme judgments and rates everyone as average or neutral C. Evaluates people by comparing them with characteristics of recently observed people D. Consistently evaluates people in an extremely positive fashion E. Relies on most recent information

D. Consistently evaluates people in an extremely positive fashion

To minimize contrast effects bias, a manager should: A. Record examples of positive and negative performance throughout the year B. Be fair and realistic in evaluations C. Define an accurate profile with both high and low points D. Evaluate employees against a predetermined standard E. Provide only negative feedback

D. Evaluate employees against a predetermined standard

Allen used to make suggestions to his boss about how the department's performance could be improved. Although his boss always said they were great ideas, they were never acted upon. Allen then stopped making suggestions. This is an example of: A. Punishment B. Negative reinforcement C. Positive reinforcement D. Extinction E. The law of effect

D. Extinction

Polly worked as a bartender. A few people used to wave money at her so that she would serve them first. She found this to be rude and, hence, either ignored them or always served them last. People came to realize that she served only the people who waited politely, and much of the rude behavior stopped. Polly used the model of __________. A. Positive reinforcement B. Negative reinforcement C. Variable ratio reinforcement D. Extinction E. Shaping

D. Extinction

Which forms of consequences weaken an undesirable behavior? A. Negative reinforcement and extinction B. Negative reinforcement and punishment C. Positive reinforcement and negative reinforcement D. Extinction and punishment E. Extinction and negative reinforcement

D. Extinction and punishment

Josh used to run into heavy traffic every day while driving to work. One morning, he left his home earlier than usual and did not run into heavy traffic. Again the next morning, he left earlier and avoided heavy traffic. His behavior of leaving earlier enabled him to avoid heavy traffic and reach the office on time. Which of the following processes did Josh engage in? A. Positive reinforcement B. Extinction C. Punishment D. Negative reinforcement E. Intermittent reinforcement

D. Negative reinforcement

Alfredo set a goal of completing the literature review portion of his research paper by Sunday night. This is a ________ goal. A. Behavioral B. Knowledge C. Equity D. Project E. Procedural

D. Project

Unlearned reflexes are: A. Operant behavior B. Consequential behavior C. Reward behavior D. Respondent behavior E. Contingent behavior

D. Respondent behavior

In the context of goal setting, SMART stands for: A. Specific, measurable, attainable, realistic, time-bound B. Specific, measurable, aligned, results-oriented, time-bound C. Sequential, measurable, aligned, results-oriented, time-bound D. Specific, measurable, attainable, results-oriented, time-bound E. Sequential, measurable, attainable, realistic, time-bound

D. Specific, measurable, attainable, results-oriented, time-bound

Dr. Beswick enjoys teaching college courses. He likes the salary he receives as well as the ability to travel in summers. However, he also finds the job challenging and enjoyable. This would imply that he is working for: A. Intrinsic rewards B. Extrinsic rewards C. Alternative rewards D. Total rewards E. Justice

D. Total rewards

Noise

Dana is trying to talk to a client from Russia, but his Russian accent is so thick that she can hardly understand what he is saying. In this situation, Dana's communication with her client is hampered due to __________.

Instrumental Value

Deal with views on acceptable modes of conduct, such as being honest and ethical and being ambitious

Dr. Beswick was writing questions for a test, but found herself listening to reggae music coming from an adjoining office, and thinking about her upcoming trip to the Caribbean. She is experiencing:

Deficit

______ is the first step on the three-step journey (or approach) of the Organizing Framework.

Defining the problem Explanation Our applied approach to problem solving proposes three activities or steps: Step 1: Define the problem. Step 2: Identify the OB concepts or theories to use to solve the problem. Step 3: Make recommendations and (if appropriate) take action.

Person- Job Fit

Degree to which a person's skill, knowledge, abilities and other characteristics match the job demands

5. Explain why delegation is the highest form of empowerment, and discuss the connections among delegation, trust, and personal initiative.

Delegation gives employees more than a participatory role in decision making. It allows them to make their own work-related decisions. Managers tend to delegate to employees they trust. Employees can get managers to trust them by demonstrating personal initiative (going beyond formal job requirements and being self-starters).

_________ are levers for changing organizational culture, addressing all three levels of culture.

Deliberate role modeling, training, and coaching Explanation Table 14.1 indicates that the deliberate role modeling, training, and coaching mechanism addresses all three levels of culture.

Pressure (hard)

Demanding compliance or using intimidation or threats

Which of the following statements about leadership effectiveness is false?

Demographic characteristics such as gender have no effect on leadership behaviors. Explanation Research that has demonstrated that gender is a factor in leadership behaviors includes differences in task and social leadership, leader style, and assertiveness.

Legitimate

Dewey, a manager at the Busy Bee Restaurant, gives Alice her schedule for the next week. Alice knows she must follow that schedule. What kind of power is Dewey exercising?

Decision trees

Diagrams in which answers to yes or no questions lead decision makers to address additional questions until they reach the end of the tree

________ occurs when a facially neutral employment practice disproportionately excludes a protected group from employment opportunities.

Disparate impact

Peter Quill (aka Star Lord) and Rocket (aka 89P13) both applied for a job as a trainee pilot with NovaCorps. Despite both having similar profiles, Peter was hired but Rocket was not, and NovaCorps continued to look for other pilots. In this case, Rocket was a possible victim of

Disparate treatment

Jonathan decides that rather than hire an employee to replace someone who left, he will eliminate the position. Jonathan is

Dissolving the problem

Distinctiveness

Does this person behave the same way across different situations

Consistency

Does this person behave the same way in different occasions in the same situation

Consensus

Does this person behave the same way?

Which of the following statements is true of the strategic implementation stage of the strategic management process?

During this stage, an organization follows through on a strategy chosen in the strategy formulation stage

A major pharmaceutical firm is developing a new medical drug for the market. Stockholders are pressing for an early release date, as the drug is expected to generate significant profits. Unions representing production workers are pressing for wage increases, which would reduce the resources available for development and marketing, thus delaying the release date. Consumer activists are pressing for increased testing which would both increase costs and delay the release date. Which balanced scorecard perspective is most applicable in this situation? A. resource allocation B. internal processes C. learning and growth D. customer E. financial

E

ABC International has just passed $250 million in sales, and its president decides to adopt a different organizational structure in order to better service its customers. The structure looks like this: • Commercial division. Focuses on all commercial customers and has its own product development and production employees. • Retail division. Focuses on all retail customers in the United States, and has its own product development and production employees. • International division. Focuses on all retail customers outside of the United States. It shares product development and production facilities with the retail division. ABC International is now using a ____________ structure. A. hollow B. matrix C. functional D. mechanistic E. divisional

E

Deliver-IT dispatches over 5,400 trucks from 165 service centers on a daily basis. The company picks up deliveries, consolidates them, and then delivers them to customers. This company: • delivers 81 percent of packages on time. • has an impressive driver safety record. • uses driver mentors and trainers. • links pay to safety measure. This reflects the ___________ perspective. A. resource acquisition B. goal accomplishment C. organizational culture D. strategic constituencies satisfaction E. internal processes

E

Managers should consider certain key factors when making decisions about how best to design an organization. These are A. strategy and goals, culture, climate, and human resources. B. strategy and goals, technology, size, and culture. C. culture, climate, human resources, and leadership. D. size, technology, and environmental change. E. strategy and goals, size, technology, and human resources.

E

The Peter's Group originated four years ago, and since then, the company has grown each year. Management has focused on making sure that the organization's strategic goals cascade down through the organization so that the employees can work together in pursuit of common ends. Peter's Group is using ________ in their organization. A. coordination of effort B. the chain of command C. the hierarchy of authority D. division of labor E. aligned goals

E

According to three separate meta-analyses and a series of studies conducted by consultants across the United States, which of the following statements about leadership traits in men and women is true? A) Men scored more than women in peer ratings when rated on a number of effectiveness criteria. B) Men are more assertive than women. C) Women use a more autocratic and directive style than men. D) Men use a more democratic or participative style than women. E) Men and women were seen as displaying more task and social leadership, respectively.

E) Men and women were seen as displaying more task and social leadership, respectively. Feedback: Learning Objective: 14-01 Topic: Trait and Behavioral Theories of Leadership Blooms: Understand AACSB: Analytic Level of Difficulty: Medium Page: 367 Explanation: Three separate meta-analyses and a series of studies conducted by consultants across the United States uncovered the following differences: (1) Men and women were seen as displaying more task and social leadership, respectively; (2) women used a more democratic or participative style than men, and men used a more autocratic and directive style than women; (3) men and women were equally assertive; and (4) women executives, when rated by their peers, managers, and direct reports, scored higher than their male counterparts on a variety of effectiveness criteria.

Which of the following is not a part of effective performance management? A. Reviewing performance B. Monitoring performance C. Providing consequences D. Defining performance E. All of the above are parts of effective performance management

E. All of the above are parts of effective performance management

_______ is the personality trait with the strongest positive relationship with job performance A. Extraversion B. Emotional stability C. Proactivity D. Agreeableness E. Conscientiousness

E. Conscientiousness

Barbara is training her new puppy to sit on command. Every time the puppy responds correctly, he receives a treat. Barbara is using a __________ reinforcement schedule. A. Fixed reward B. Variable ratio C. Fixed interval D. Variable interval E. Continuous

E. Continuous

Dr. Hogan carefully selected the final examinations written by her best students to read first. They all received excellent grades. After these, she read the rest of the exams and graded most of them average or worse. This is an example of _________ perceptual error. A. Halo B. Leniency C. Central tendency D. Recency E. Contrast effects

E. Contrast effects

Evaluating people or objects by comparing them with characteristics of other recently observed people or objects is known as the ________ perceptual error. A. Halo B. Leniency C. Central tendency D. Recency E. Contrast effects

E. Contrast effects

The process of comparing performance at some point in time to a previously established expectation or goal is called: A. Monitoring B. Measuring C. Giving feedback D. Coaching E. Evaluating

E. Evaluating

Francois endures constant criticism at his job because of where he is from. His ability to endure the crisiticism is due to ________ intelligence A. Musical B. Naturalist C. Linguistic D. Interpersonal E. Intrapersonal

E. Intrapersonal

Jill is a sales representative for a large pharmaceutical corporation. According to the current compensation system, if Jill meets her target for the quarterly sales projections, she will receive a bonus equivalent to 7% of her annual salary. Jill is working hard to meet the sales goal so that she can use the bonus money as a down payment on a new car for herself. Which of the following is true regarding Jill's motivation? A. It comes from a sense of competence. B. It comes from a sense of progress. C. It is a consequence of job redesign. D. It is intrinsically motivated. E. It is extrinsically motivated.

E. It is extrinsically motivated.

The two basic types of goals are: A. Learning and behavioral B. Behavioral and performance C. Performance and objective D. Project and objective E. Performance and learning

E. Performance and learning

Negative reinforcement is: A. Strengthening a behavior by contingently presenting something pleasing B. Weakening a behavior by ignoring it C. Weakening a behavior through contingently presenting something displeasing D. Weakening behavior by contingently withdrawing something positive E. Strengthening a behavior by contingently withdrawing something displeasing

E. Strengthening a behavior by contingently withdrawing something displeasing

Philip got a mail from his supervisor with compliments and praises for his work over the past few months. This instance is an example of a __________ schedule of reinforcement. A. Continuous B. Fixed ratio C. Variable ratio D. Fixed interval E. Variable interval

E. Variable interval

What research tells us

Effective onboarding programs result in increased retention, productivity, and rates of task completion for new hires. Many organizations use socialization tactics to reinforce a culture that promotes ethical behavior. Managers need to help new hires integrate with the culture to overcome stress associated with a new environment. Support for the stage model is mixed, different techniques are appropriate for different people at different times. Managers should pay attention to the socialization of diverse employees

The characteristic of PsyCap that consists of having self-confidence is:

Efficacy

_____ is the characteristic of PsyCap that consists of having confidence.

Efficacy Explanation The characteristic of PsyCap that consists of having confidence to take on and put in the necessary effort to succeed at challenging tasks is efficacy.

Active

Eileen is on a five-person team in her business strategy class. The team is responsible for participating in a 10-week simulation of a firm in the airline industry. Eileen contributes frequently in discussions in the twice-weekly team meetings. She is exhibiting the ______ style of listening.

Betty plans on applying Herzberg's theory to motivate employees. As such, the first step should be to

Eliminate sources of dissatisfaction

Which of the following is NOT a suggestion as to improving communications between generations?

Embrace the most current technology. Explanation Helpful suggestions for improving communications between generations were offered by trainer Dana Brownlee in an interview with Forbes: Match degree of formality with the culture. Use a variety of communication tools. Avoid stereotypes. Be aware of different values and motives. Be willing to learn and teach.

Which of the following is not one of the basic dimensions of the Big Five personality dimensions?

Emotional Intelligence

___ is the ability to monitor one's own and other's feelings and emotions, to discriminate among them, and to use this information to guide one's thinking and actions.

Emotional Intelligence

________ are complex, relatively brief responses aimed at a particular target.

Emotions

___ is the harnessing of organization members' selves to their work roles.

Employee engagement

Which of the following is NOT an element of Positive Organizational Behavior?

Employees are self-absorbed and anxious

Phase 2: Encounter

Employees come to learn what the organization is really like. Organizations use onboarding programs.

Phase 3: Change and acquisition

Employees master important tasks and roles and adjust to their group's values and norms.

Which of the following statements is true? -PERMA elements are positively related to good health, but are not related to work-related outcomes such as organizational commitment and career satisfaction. -Well-being is a single, unique concept that is related to happiness. -Employees' level of flourishing is related to organizational outcomes such as productivity and financial performance. -Positive emotions "happen to people"; they cannot be pursued proactively. -PERMA should be pursued as a means to obtain a better career.

Employees' level of flourishing is related to organizational outcomes such as productivity and financial performance. Explanation Flourishing is positively related to outcomes at both the group/team level-team performance-and the organizational level-overall productivity and financial performance.

_________ is(are) the efforts to enhance employee performance, well-being, and positive attitudes.

Empowerment Explanation Empowerment is defined as efforts to enhance employee performance, well-being, and positive attitudes.

4. Define the term empowerment, and explain why it is a matter of degree.

Empowerment involves sharing varying degrees of power and decision-making authority with lower-level employees to tap their full potential. Empowerment is not an either-or, all-or-nothing proposition. It can range from merely consulting with employees, to having them actively participate in making decisions, to granting them decision-making authority through delegation.

In which stage of the organizational socialization process would an onboarding program be provided?

Encounter

Which of the following is not a task role?

Encourager

Terminal Values

End states people desire in life, such as leading a prosperous life and a world at peace

Maria focuses on community volunteerism, while Anna focuses on getting a promotion at work. ___ are represented by Maria's and Anna's focuses.

Endpoints of one of the dimensions of values

Meghan is a member of a group that raises money for a homeless shelter. She feels that the group spends too much time arguing the merits of different fundraisers and needs to make a decision. She calls for a vote at today's meeting. Meghan has taken on the role of:

EnergizeR

Which of the following agencies is responsible for enforcing Title VII of the Civil Rights Act and the Americans with Disabilities Act?

Equal Employment Opportunity Commission

Peter offers his manager some suggestions for a new holiday display. His manager shrugs his shoulders and tells Peter that the display is all set. Then Sarah approaches the manager with an idea for the display, and the manager tells her that its a great idea. ___ theory is the model of motivation that explains how peter is striving for fairness and justice.

Equity

The idea underlying the ____ model is that satisfaction is tied to how fairly an individual is treated at work

Equity

Waco Company monitors employees' perceptions of fairness by quarterly surveys and occasional focus groups. Management is following the ___ model.

Equity

________ values are the explicitly stated values and norms that are preferred by an organization.

Espoused

The need for prestige and reputation is part of the need for

Esteem

The need for prestige and reputation is part of the need for:

Esteem

Legitimating tactics

Ethan tried to convince his subordinates to accept a new policy by showing them that the policy is consistent with the organizational rules and policies. He also showed them that the people above him had already supported and sanctioned the policy. What influence tactic was Ethan using?

___ highlight the fact that choosing among available options is not always a choice between right and wrong.

Ethical dilemmas

Rachel gets her midterm grades and finds that she has a 2.4 in OB. She expected a better grade point average to date. Rachel is ___ her performance.

Evaluating

Which of the following statements is true? -Evidence-based decision making should never be used in making decisions. -Evidence-based decision making should never be used in informing decisions. -Evidence-based decision making should never be used in supporting decisions. -Evidence-based decision making should always be used in supporting decisions. -Evidence-based decision making should sometimes be used in supporting decisions.

Evidence-based decision making should sometimes be used in supporting decisions. Explanation Evidence is used in three different ways: to make a decision, to inform a decision, and to support a decision. Evidence is used to make a decision whenever the decision follows directly from the evidence. Evidence is used to inform a decision whenever the decision process combines hard, objective facts with qualitative inputs, such as intuition or bargaining with stakeholders. These two uses of evidence are clearly positive and should be encouraged.

__________ theory holds that people are motivated to behave in ways that produce desired combinations of expected outcomes.

Expectancy Explanation Expectancy theory holds that people are motivated to behave in ways that produce desired combinations of expected outcomes.

______ is not a content theory of motivation.

Expectancy theory Explanation Content theories include: McGregor's Theory X and Theory Y; Maslow's need hierarchy theory; acquired needs theory; self-determination theory; and Herzberg's motivator-hygiene theory. Expectancy theory is a process theory of motivation.

65% to 95%

Experts estimate that about ________ of every conversation is interpreted through nonverbal communication.

Jake made a mistake on the manufacturing line, and it ended up costing the company a significant contract. When asked why this had occurred, he blamed the line manager for poor quality checking. Jake is expressing

External locus of control

Lenore has a fight with her best friend, Samantha, and doesn't want to speak to her. She unfriends her on social media. By doing this, Lenore is using ___, thereby removing consequences from an undesirable behavior.

Extinction

People who are outgoing, talkative, sociable, and assertive are likely to score high on

Extraversion

Mary will receive a promotion and a raise if she completes a difficult assignment. This is an example of:

Extrinsic motivation

Expectancy theory is a model of motivation that explains how people strive for fairness

False

7. Distinguish between favorable and unfavorable impression management tactics.

Favorable upward impression management can be job-focused (manipulating information about one's job performance), supervisor-focused (praising or doing favors for the boss), or self-focused (being polite and nice). Unfavorable upward impression management tactics include decreasing performance, not working to potential, withdrawing, displaying a bad attitude, and broadcasting one's limitations.

___ is information about performance shared with those in a position to improve the situation.

Feedback

Which of the following statements about emotions is false?

Felt emotions are always displayed. Explanation Researchers draw a distinction between felt and displayed emotions.

commitment.

Fiona used an influence tactic on Gerald and got him to enthusiastically agree to her proposal. Gerald demonstrated initiative and persistence while completing the assignment. This is an example of:

2. Name at least three "soft" and two "hard" influence tactics, and summarize the practical lessons from influence research.

Five soft influence tactics are: -rational persuasion-inspirational appeals -consultation -ingratiation -personal appeals. They are more friendly and less coercive. The four hard influence tactics: -exchange -coalition tactics -pressure -legitimating tactics According to research, soft tactics are better for generating commitment and are perceived as more fair than hard tactics. Ingratiation—making the boss feel good through compliments and being helpful—can slightly improve performance appraisal results and make the boss like you a lot more. Influence through domination is a poor strategy for both men and women. Influence is a complicated and situational process that needs to be undertaken with care, especially across cultures.

Which of the following actions does not foster employee positivity?

Focus only on providing positive feedback. Explanation Organizations can provide decision-making discretion, share information, minimize incivility, and provide feedback. Feedback can be both motivational and instructional, and therefore can include negative feedback.

defensiveness.

Folding arms, crossing hands, and crossing legs communicate:

Phil is part of a newly formed work group. He has been introduced to all the group members, but he still feels like he cannot trust them. The group has not yet chosen a leader, and Phil feels unsure about his exact role within the group. According to Tuckman's five-stage model of group development, which of the following stages is Phil's group currently in?

Forming

3. Identify and briefly describe French and Raven's five bases of power, and discuss the responsible use of power.

French and Raven's five bases of power are: -reward power (rewarding compliance) -coercive power (punishing noncompliance) -legitimate power (relying on formal authority) -expert power (providing needed information) -referent power (relying on personal attraction) Responsible and ethical managers strive to use socialized power (primary concern is for others) rather than personalized power (primary concern for self). Research found higher organizational commitment among employees with bosses who used uplifting power than among those with power-hungry bosses who relied on dominating power.

Coalition Tactics (hard)

Getting other to support your effort to persuade someone

Consolation (soft)

Getting others to participate in planning, making decisions, and changes

Impression Management

Getting others to see us in a certain manner.

coalition tactics.

Getting others to support your efforts to persuade someone is called:

Ingratiation (soft)

Getting someone in a good mood prior to making a request; being friendly, helpful, and using praise or flattery

The term___ identifies an invisible but absolute barrier that prevents women from advancing to higher-level positions.

Glass ceiling

Which of the following statements about goals and goal setting is not true?

Goals should be set at a level just beyond reachable "stretch" goals. Explanation People must have the ability and resources needed to achieve the goal, and they need to be committed to the goal. If these conditions are not met, goal setting does not lead to higher performance.

Personal Initiative

Going beyond formal job requirements and being an active self-starter.

Three months ago, XYZ Corporation changed the way the typing pool is organized from a self-managed team to a more traditional structure in which employees receive their assignments from a direct supervisor. Since then, three of the seven employees have left the organization. ___ is the OB perspective applied here

Group

_______ is a mode of thinking that people engage in when they are deeply involved in a cohesive in-group, when members' strivings for unanimity override their motivation to realistically appraise alternative courses of action.

Groupthink Explanation Groupthink is a mode of thinking that people engage in when they are deeply involved in a cohesive in-group, when members' strivings for unanimity override their motivation to realistically appraise alternative courses of action.

Carolyn is a very likable employee. Recently, her boss rated her at the highest level in all areas of performance, even though in some areas her performance did not warrant this judgment. This is an example of a ___________ perceptual error.

Halo

encoding

Harry is thinking about meeting Connie. He writes a text message to Connie, inviting her for a drink at the local bar. This is an example of __________.

Adam is a new product development manager for Hello.com, an Internet match-making service. He has only two employees reporting directly to him. From what you have read about organization charts, what do you know for sure?

He has a narrow span of control. Explanation Span of control refers to the number of people reporting directly to a given manager. Spans of control range from narrow to wide. For example, the president in Figure 15.2 has a narrow span of control of two.

Pierre is a foreman in a plant. He has 25 line workers who report to him on the night shift. From what you have read about organization charts, what do you know for sure?

He has a wide span of control. Explanation Span of control refers to the number of people reporting directly to a given manager. Spans of control can range from narrow to wide. For example, the president in Figure 15.2 has a narrow span of control of two. Narrow spans of control tend to create "taller" organizations or those that are more hierarchical. In contrast, a wide span of control leads to a "flat" organization. The executive administrative director in Figure 15.2 has a wider span of control of five. Historically, spans of 7 to 10 people were considered best.

A(n) ________ culture develops reliable internal processes, extensive measurement, and a variety of control mechanisms.

Hierarchy

_______ is reflected in this statement: "I feel I am a person of worth, as good as other people."

High self-esteem Explanation "I feel I am a person of worth, as good as other people," is a positive statement reflecting high self-esteem.

The acronyym HERO shows people with high levels of positive psychological capital tend to display:

Hope, efficacy, resilience, optimism

________ organizational design organizes people around the workflow.

Horizontal Explanation Organizations defined by a horizontal approach work hard to flatten hierarchy and organize people around specific segments of the workflow.

resistance.

Hugh tried his best to convince Irene about a new business proposal, but Irene presented some good counterarguments and stalled the proposal. This is an example of:

Ted was dissatisfied with his job. He said that the company policy, supervision, and working conditions were responsible for his dissatisfaction. According to Frederick Herzberg's theory, ________ are the extrinsic factors that create job dissatisfaction

Hygiene Factors

Of the approaches to job design, ___ is the most recently evolved and attempts to merge earlier perspectives.

I-deals

actively try to help the sender convey his or her message.

If a person wants to be an effective listener, he or she should:

compliance.

If the second party grudgingly agrees to the first party's proposal and needs a lot of prodding to satisfy the minimum requirements, the second party is said to be exhibiting:

___ are goals and incentives set to promote a desired behavior, but instead encourage a negative one.

Ill-conceived goals

_______ are goals and incentives set to promote a desired behavior, but instead encourage a negative one.

Ill-conceived goals Explanation Table 1.2 states that ill-conceived goals occur when we set goals and incentives to promote a desired behavior, but they encourage a negative one.

_________ is(are) an example of social and political pressures for change.

Impact of climate change Explanation Social and political pressures are created by social and political events. For example, widespread concern about the impact of climate change and rising energy costs have been important forces for change in almost every industry around the world.

Which of the following statements about impression management is false?

Impression management is generally damaging to careers. Explanation A moderate amount of upward impression management is a necessity for the average employee today. For example, ingratiation can slightly improve your performance appraisal results and make your boss like you significantly more. Engage in too little impression management and busy managers are liable to overlook some of your valuable contributions when they make job assignment, pay, and promotion decisions. Too much, and you run the risk of being branded a "schmoozer," a "phony," and other unflattering things by your coworkers. Consider, for instance, that noticeable flattery and ingratiation can backfire by embarrassing the target person and damaging your credibility.

create trust and mutual obligation.

In an in-group exchange, a leader is attempting to:

create more formality in expectations and rewards.

In an out-group exchange, a leader is attempting to:

Cause groups to interpret silence as assent

In groupthink, the illusion of unanimity is likely to:

Reward

In his annual performance review meeting with Eileen, who has been working at the company for two years, Ruben suggests that if she keeps up her good work she is likely to be offered a promotion within the next year. Ruben is exercising his _______ power.

Evidence forms the basis of the decision

In making a decision,

Delegation

In power distribution, the manager adopts the _____ style.

males are expected to hide their emotions.

In regard to differences in how men and women communicate, in general:

Confirmation

In the _________ bias, a decision maker subconsciously decides something even before investigating, and then seeks information that supports the decision.

Which of the following statements is true of the administrative linkage level between the human resource management function and the strategic management function?

In this level, the human resource management department is completely divorced from any component of the strategic management process.

Deep-level characteristics of diversity

Include attitudes, opinions and values

Pat is applying for anew job. Which of the following will help her online "image"?

Include information on her volunteer activities

Which of the following is a tip for more effective PowerPoint presentations?

Include questions on slides. Explanation Table 9.8 identifies 10 tips for more effective PowerPoint presentations. One of which is ask questions. Questions engage the audience or pique their interest, so use them!

___ is(are) when we hold others less accountable for unethical behavior that is carried out through third parties.

Indirect blindness

_____ is(are) when we hold others less accountable for unethical behavior that is carried out through third parties.

Indirect blindness Explanation Table 1.2 states that indirect blindness occurs when we hold others less accountable for unethical behavior when it's carried out through third parties.

Alexa quit her job because she felt it was boring. This would be diagnosed as a(n) ___ level problem

Individual

Reward

Individuals or organizations have _______ power if they can obtain compliance by promising or delivering positive reinforcement.

Have a tendency to avoid conflict

Individuals who are behavioral in decision making style:

______ is(are) conscious efforts to affect and change a specific behavior in others.

Influence tactics Explanation Influence tactics are conscious efforts to affect and change a specific behavior in others.

Mary Flowers, the founder of MedTran, an offsite transcription service, asks transcribers to discuss their potential problems at weekly staff meetings. ___________ is the process that a learning organization uses to share information among its people.

Information distribution Explanation Information distribution pertains to the processes or systems that people, groups, or organizational units use to share information among themselves.

In the Organizing Framework,

Inputs lead to processes, which lead to outcomes.

___ is how an individual perceives the movement from performance to outcome

Instrumentality

______ refers to the amount of effort that we invest in an activity.

Intensity Explanation Intensity represents the amount of effort being invested in the activity.

_______________ are the key links between _____________ in Ajzen's Theory of Planned Behavior.

Intentions; attitudes and planned behavior

___________ are the key links between ___________ in Ajzen's Theory of Planned Behavior.

Intentions; attitudes and planned behavior Explanation Ajzen developed and refined a model focusing on intentions as the key link between attitudes and planned behavior. See Figure 2.3.

"Different people may perceive similar situations in different ways and similar people may perceive different situations in the same way." This represents the ___ perspective.

Interactional

Group Decision Support Systems (GDSS)

Interactive computer based systems that are able to combine communication and decision technologies to help groups make better decisions

___ intelligence is the potential to understand and regulate oneself.

Intrapersonal

________ intelligence is the potential to understand and regulate oneself.

Intrapersonal

Eileen signed up for a photography class after having seen some spectacular wildlife art pictures done by a local artist. She is most likely experiencing:

Intrest

___ results when an individual gains satisfaction and self-praise from an assignment.

Intrinsic motivation

_____________ results when an individual gains satisfaction and self-praise from an assignment.

Intrinsic motivation Explanation Intrinsic motivation occurs when an individual is "turned on to one's work because of the positive internal feelings that are generated by doing well, rather than being dependent on external factors (such as incentive pay or compliments from the boss) for the motivation to work effectively."

At Peter James Consulting, managers are in the process of submitting their budget cuts for the next fiscal year. In evaluating alternatives, managers should consider all of the following except

Is this the choice that I, personally, would like to make? Explanation Managers need to evaluate alternatives in terms of costs and quality, ethicality, feasibility, and will it remove the causes and solve the problem.

Which of the following statements is true of the SWOT analysis?

It gives the strategic planning team of an organization all the information it needs to generate a number of strategic alternatives.

Level 2: Espoused Values

It is important to distinguish between values that are espoused versus those that are enacted.

Which of the following is true of one-way linkage?

It precludes the company from considering human resource issues while formulating the strategic plan

Expert

Jeff has been a project assistant for XYZ Corporation for five years. Recently he was asked to train Harriet, a new employee. He suggests to her that she use an Excel worksheet to lay out the projects she is currently working on, and she complies. He is using _______ power.

Jim is the daytime supervisor for an automobile assembly line. He supervises 45 workers who perform routine jobs that require minimal training. Which of the following statements would indicate that Jim is following the transactional model of leadership?

Jim sets goals, monitors progress toward goal achievement, and rewards and punishes people for their level of goal accomplishment. Explanation Transactional leadership focuses on clarifying employees' role and task requirements and providing followers with positive and negative rewards contingent on performance.

helper.

Joaquin is usually the first one to volunteer at the weekly team meetings to take on additional tasks like taking notes for the meeting. Joaquin would probably be categorized as a(n):

___ is defined as the physical and cognitive changes individuals make in the task or relational boundaries of their work.

Job crafting

A firm trained its customer service representatives who handled telephone inquiries to handle walk-in customers as well. The firm engaged in

Job enlargement. Explanation Job enlargement involves putting more variety into a worker's job by combining specialized tasks of comparable difficulty. Here, the firm introduced variety into the jobs of customer service representatives by training them to handle walk-in customers as well.

Which of the following is not a tip for increasing goal commitment and success?

Keep your goals to yourslef

LMX (leader member exchange theory)

Kevin manages a group of eight project engineers, all of whom have worked for the company for over five years. Almost every day, Kevin and Joe, Carlos, and Kim, who are three of his subordinates, go to the cafeteria and play bridge while eating lunch. Joe, Carlos, and Kim receive very good performance reviews and are typically given the most challenging projects. What theory best explains this situation?

Kevin manages a group of eight project engineers, all of whom have worked for the company for over five years. Almost every day, Kevin and Joe, Carlos, and Kim, who are three of his subordinates, go to the cafeteria and play bridge while eating lunch. Joe, Carlos, and Kim receive very good performance reviews and are typically given the most challenging projects. What theory best explains this situation? -Transformational leadership -Laissez-faire leadership -Transactional leadership -LMX -House's path-goal theory

LMX

the leader's relationships with followers.

Leader-member exchange theory focuses on:

_________ is a set of culturally learned signals by which we communicate what we mean and interpret others' meanings and evaluate one another as people.

Linguistic style Explanation Linguistic style refers to a person's characteristic speaking pattern.

Vivek and Warren are working on an assignment for their marketing class. Warren finished his part of the assignment; Vivek doesn't. Vivek apologizes because he took an extra shift at work and didn't give the assignment for the priority it needed. He promises Warren he will stay home tonight and finish his work. what personality characteristic does this show?

Locus of Control

__________ is the use of manipulation, a cynical view of human nature, and a moral code that puts results over principles.

Machiavellianism Explanation Machiavellianism entails the use of manipulation, a cynical view of human nature, and a moral code that puts results over principles.

In some business situations, you will make recommendations to solve a problem. Which of the following actions can help improve the quality of your recommendations?

Make sure that your recommendations address the causes of the problem. Explanation If you map recommendations onto the causes of the problem, you can be more confident that as you remedy the causes, you will solve or at least ease the underlying problem.

becoming defensive.

Malcolm's boss just came into his office, obviously upset, and said, "I'm tired of your being late all the time on these projects. You need to clean up your act!" Malcolm is likeliest to respond by:

1. Explain the concept of mutuality of interest.

Managers are constantly challenged to foster mutuality of interest (a win-win situation) between individual and organizational interests.

Involved

Manash pays attention in his management class, where the professor has entertaining, interesting visual aids. At times, he makes good eye contact with the professor, and occasionally raises his hand to ask questions. On other days, he is quiet and his eyes are on his notebook. He is exhibiting the ________ style of listening.

Marcy left a large Fortune 500 company to go to a small start-up technology firm. Earlier in her career, Marcy wanted a high salary and corporate position, but now she wants something different. She wants to be part of something that will grow and she enjoys the flexibility a small firm offers. In addition, she is involved in decision making, so she almost feels like it's her own company. These differing needs represent steps in

Maslow's need hierarchy

___ states that motivation is a function of five basic needs that are met sequentially

Maslow's need hierarchy theory

Which of the following statements about mechanistic organizations is true?

Mechanistic organizations generally are expected to produce reliability and consistency in internal processes. Explanation The orderliness of mechanistic organizations is expected to produce reliability and consistency in internal processes, thereby resulting in higher efficiency, quality, and timeliness.

Dr. Riley, a professor in the economics department, finds that if he keeps his door open, people tend to stop in to say hello. This interruption is distracting. So now Dr. Riley keeps his office door open only a crack during his office hours, asking visitors to knock before entering. This small change allows him to grade exams more quickly and accurately. This is an example of his increasing his:

Mindfullness

_____ is a key input to positivity from person factors.

Mindfulness Explanation As seen in Figure 7.1, inputs and processes that drive positive organizational behavior are positive emotion, mindfulness, psychological capital, and organizational culture and climate.

Which of the following statements about mindfulness is false?

Mindfulness requires less effort than letting our minds wander. Explanation Mindfulness requires effort because our brains work in ways that detract from staying focused.

While waiting to board an airplane, Mario had his earbuds on, listening to music while reading a text message. He missed his boarding group and could not find space to stow his luggage. Mario was being:

Mindless

Which of the following statements is true?

Motivated blindness occurs when we overlook the unethical behavior of another because its in our interest to remain ignorant

___ is the psychological process that arouses our interest in doing something.

Motivation

___ are physiological or psychological deficiencies that arouse behavior

Needs

Leaning forward

Negative nonverbal communication skills include all but one of the following. Which one?

Elaine often volunteers to help others when they run into problems with their projects. She is very positive, always on time, and never calls in sick. Exhibiting ________ is likely to create positive impressions among Elaine's colleagues and manager.

OCB

Elaine often volunteers to help others when they run into problems with their projects. She is very positive, always on time, and never calls in sick. Exhibiting ________ is likely to create positive impressions among Elaine's colleagues and manager.

OCB Explanation Exhibiting OCBs (organizational citizenship behavior) is likely to create positive impressions about you among your colleagues and manager. In turn, these impressions affect your ability to work with others, your manager's evaluation of your performance, and ultimately your promotability.

Phase 1: Anticipatory socialization

Occurs before an individual actually joins an organization Information learned about careers and organizations Learned from: Current employees Social media Internet

Which of the following agencies or institutions is responsible for enforcing the executive orders that cover companies doing business with the federal government?

Office of Federal Contract Compliance Programs

Subcultures

Often not a single homogeneous culture Rather, multiple subcultures that either intensify the existing cultural understanding and practices or diverge from them

Letitia reports to work at Apex Inc. This is her first day. Human resources tells Letitia to report to a conference room so that she can review the company's mission statement and policies, go over her job description, and learn about the technology systems in use. This type of experience is an ___ program.

Onboarding

immediacy.

Open body positions, such as leaning backward or gesturing with palms facing up, communicate:

__________ rely on leveraging technology and structural flexibility to maximize potential value through outsourcing and external collaboration.

Open designs Explanation Open design structures rely on leveraging technology and structural flexibility to maximize potential value through outsourcing and external collaboration.

________ is associated with entrepreneurialism.

Openness to experience

Personality Big Five Traits

Openness, conscientiousness, extraversion, agreeableness and neuroticism

_________ is behavior that is learned when one acts on the environment to produce desired consequences.

Operant behavior Explanation Operant behavior is behavior that is learned when one "operates on" the environment to produce desired consequences.

Recently, the Acme Explosives Company was sold to a new owner, Roadrunner Inc. The operations of the two organizations re going to be merged, with several of the manufacturing locations possibly to be eliminated. ___ is the most likely OB perspective to apply.

Organizational

Mason is highly satisfied with his job. He receives consistently strong evaluations and volunteers to train new employees. Mason exhibits positive job performance and

Organizational Citizenship Behaviors

____________ is defined as employees' perceptions of formal and informal organizational policies, practices, procedures and routines:

Organizational climate

___ is the extent to which an individual identifies with an organization and commits to its goals.

Organizational commitment

______ is the extent to which an individual identifies with an organization and commits to its goals.

Organizational commitment Explanation Organizational commitment reflects the extent to which an individual identifies with an organization and commits to its goals.

________ is the set of shared, taken-for-granted implicit assumptions that a group holds and that determines how it perceives, thinks about, and reacts to its various environments.

Organizational culture

6. Define organizational politics, and explain what triggers it.

Organizational politics is defined as intentional acts of influence to enhance or protect the self-interests of individuals or groups. Uncertainty triggers most politicking in organizations. Political action occurs at individual, coalition, and network levels. Coalitions are informal, temporary, and single-issue alliances.

Which of the following statements about applying organizational socialization research is true?

Organizations can use socialization tactics to promote ethical behavior. Explanation More and more organizations use socialization tactics to reinforce a culture that promotes ethical behavior.

Which of the following statements about organizational culture is true?

Organizations tend to have one dominant type of culture.

___ is when we give a pass to unethical behavior if the outcome is good.

Overvaluing Outcomes

Angela works for a company that does not provide good health benefits or career opportunities. Her _____ is most likely to be negative.

POS Explanation Perceived organizational support (POS) reflects the extent to which employees believe their organization values their contributions and genuinely cares about their well-being.

___ is the popular term for linking monetary incentives with results or accomplishments

Pay for performance

Alana looks at the clouds and sees flowers. Her brother, Sammy, looks at the same clouds and sees jellyfish. Alana and Sammy have different

Perceptions

Barry set himself a goal of bowling at least one game above 250 (out of 300 possible) before the end of the season. This is a ________ goal

Performance

___ is the set of processes and managerial behaviors that involve defining, monitoring, measuring, evaluation, and providing consequences for performance expectations.

Performance Management

You value achievement and your employer rewards people for accomplishing goals, so you are very committed to the company. _____ fit is the consistency between personal and company values.

Person-culture Explanation If you value achievement and your employer rewards people for accomplishing goals, you are more likely to be committed to the company. This consistency between personal and company values is called person-culture fit.

___ is the combination of stable physical, behavioral, and mental characteristics that gives individuals their unique identity.

Personality

Personality Testing in Employee Selection

Personality is a potentially important predictor of work behavior. Matching people to jobs matters, because when people do not fit with their jobs or the company, they are more likely to leave, costing companies as much as a person's annual salary to replace them. One method some companies use to improve this match and detect the people who are potentially good job candidates is personality testing. A key consideration in this debate is the knowledge that how a personality test is used influences its validity. Another problem with personality tests is the uncertain relationship between performance and personality. Research has shown that personality is not a particularly strong indicator of how a person will perform at work. According to one estimate, personality only explains about 10-15% of variation in job performance. In any case, if you decide to use personality tests for selection, you need to be aware of their limitations. Relying only on personality tests for selection of an employee is a bad idea, but if they are used together with other tests such as tests of cognitive abilities, better decisions may be made. the company should ensure that the test fits the job and actually predicts performance. This process is called validating the test.

Which of the following statements about incivility at work is false?

Personality is highly unlikely to be a source of incivility. Explanation Figure 10.3 illustrates some common causes of various forms of incivility.

As the mother of two small children, Jen struggles to make ends meet on her minimum-wage job. After paying rent and child care expenses, there is sometimes not enough money left at the end of the month to pay the heating bill. More than once the family has gone without warmth on cold nights, and she and the kids have gone to bed hungry. Which of the five basic needs is Jen struggling to meet?

Phycological

An aid group is trying to help a village in a remote area with a high rate of malnutrition among children. Most families have children with symptoms of malnutrition but a few families have perfectly healthy children. The aid group investigates and discovers that the healthy families are not particularly wealthy, they just have a different diet from locally available foods that most people don't eat. The aid group decides to develop a program to teach everyone how to cook and eat the foods the healthy families are eating. This is an example of finding and taking advantage of:

Positive devience

__________ are perceptions of whether a stressor is irrelevant, positive, or negative.

Primary appraisals Explanation Primary appraisals are perceptions of whether a stressor is irrelevant, positive, or negative.

A(n) ___ is someone who is relatively unconstrained by situational forces and who effects environmental change.

Proactive Person

Other Personality Traits Pt. 3

Proactive personality refers to a person's inclination to fix what is perceived as wrong, change the status quo and use initiative to solve problems.

____________ is associated with entrepreneurialism.

Proactivity Explanation Proactive personality is linked to intentions to be entrepreneurial.

referent; committed

Professor Jones asks Professor Brown to cover class for her while she is out of town presenting a paper at a conference. They are good friends and work together effectively on several committees. Professor Brown not only covers the class, but develops a special lecture and exercise for the students. Professor Jones is using her _______ power and Professor Brown is ________.

Claire works for Content Consultants. she telecommutes and has great flexibility in her work schedule. In return, Content Consultants expects to have satisfied clients who hire them again. Claire receives consistently high evaluations. Despite that, the company now wants her to work out of the main office. Claire feels that Content Consultants has breached a(n)

Psychological Contract

self-efficacy.

Psychological empowerment is related to:

______ is a lack of concern for others, impulsive behavior, and a lack of remorse or guilt when one's actions harm others.

Psychopathy Explanation Psychopathy is characterized as a lack of concern for others, impulsive behavior, and a lack of remorse or guilt when one's actions harm others.

Feedback

Rebecca e-mailed a wedding invitation to her cousin Shawn. A few days later, she got a reply saying that he can't make it to the wedding because he will not be able to get any vacation. In the communication process, Shawn's reply is an example of

Counterproductive work behavior (CWB) harms other employees, the organization as a whole, and organizational stakeholders such as customers and shareholders. Which of the following is not an example of CWB?

Reporting unethical behavior poor financial results at the quarterly meeting

Which of the following statements about organizational socialization is false?

Research shows that realistic job previews do not help reduce turnover.

Imelda's life is difficult right now. Her husband and she recently decided to divorce and she is now a single parent of two preteenagers. They are good kids, but it is difficult taking care of them and managing her full-time job as a middle manager. Imelda has started taking work home at night and finishing projects at night after her children are in bed. Imelda is likely exhibiting:

Resilience

Discretionary individual behavior that promotes the effective functioning of the organization is called organizational citizenship behavior. ___ is an example of OCB.

Respecting the spirit as well as the letter of housekeeping rules

Discretionary individual behavior that promotes the effective functioning of the organization is called organizational citizenship behavior. ______ is an example of OCB.

Respecting the spirit as well as the letter of housekeeping rules Explanation Organizational citizenship behavior (OCB) is defined as individual behavior that is discretionary, not directly or explicitly recognized by the formal reward system, and that in the aggregate promotes the effective functioning of the organization. Examples include: -Constructive statements about the department. -Expression of personal interest in the work of others. -Suggestions for improvement. -The training of new people. -Respect for the spirit as well as the letter of housekeeping rules. -Care for organizational property. -Punctuality and attendance well beyond standard or enforceable levels.

Which of the following is not a suggestion for managers who want to reduce voluntary turnover?

Reward all employees equally. Explanation Managers can reduce voluntary turnover if they: (1) Hire people who "fit" within the organization's culture; (2) Spend time fostering employee engagement; (3) Provide effective onboarding, which helps employees to integrate, assimilate, and transition to new jobs by making them familiar with corporate policies, procedures, culture, and politics by clarifying work-role expectations and responsibilities; (4) Recognize and reward high performers because they are more likely to quit than average performers.

Ethics is primarily concerned with

Right and Wrong

Detached

Roberto is meeting with Professor Plum in his office during advising hours. As Roberto is describing the problems he is having in his accounting course, Professor Plum is checking his e-mail. Professor Plum is exhibiting the ________ style of listening.

___ theory assumes that competency, autonomy, and relatedness influence our behavior and well-being.

Self-determination

_________ theory assumes that competency, autonomy, and relatedness influence our behavior and well-being.

Self-determination

Sandra has a new assignment. she is confident that she can successfully complete the task. This is an example of

Self-efficacy

A follower is a person who:

Serves as a passive audience

Face-to-face conversation

Sharon is the Director of Human Resources at a company that is undergoing a merger with another organization. As a result of the merger, a portion of the combined workforce will be laid off. Sharon understands that even with a lucrative severance package, the news will adversely affect the lives of these employees. Therefore, she wants to break the news to them in a sensitive manner so as to maintain the company's image. Which of the following is the most appropriate communication medium to use in this situation?

Harriet is an active member of the large Freshman Orientation board. There are five separate teams on this board, and they all want Harriet as a member because she is good at keeping the team on track. Which of the following behaviors does Harriet exhibit?

She helps plan and organize work; monitors progress; and provides constructive feedback. Explanation As someone who keeps the team on track, Harriet helps plan and organize work; monitors progress; and provides constructive feedback.

8. Explain how to manage organizational politics.

Since organizational politics cannot be eliminated, managers need to keep it within reasonable bounds. Measurable objectives for personal accountability are key. Participative management also helps, especially in the form of open-book management. Formal conflict resolution and grievance programs are helpful. Overly political people should not be hired, and employees who get results without playing political games should be publicly recognized and rewarded. The "how-would-it-look-on-TV" ethics test can limit political maneuvering.

Reactive vs Proactive Tactics

Some of the tactics, such as scapegoating, were reactive because the intent was to defend one's self-interest. Other tactics, such as developing a base of support, were proactive because they sought to promote the individual's self-interest.

Which of the following is NOT a group maintenance role?

Someone who clarifies pertinent values

In the context of goal setting, SMART stands for:

Specific, measurable, attainable, results-oriented, time-bound

Values

Stable life goals people have, reflecting what is most important to them. They are established by accumulating life experiences and tend to be relatively stable.

On the first day of class, Professor Simmons explains what the goals of the course are and what he expects of his students. This represents ___ of effective performance management.

Step 1, Defining Performance

On the first day of class, Professor Simmons explains what the goals of the course are and what he expects of his students. This represents ______ of effective performance management.

Step 1, Defining Performance, Explanation As illustrated in Figure 6.2, performance management has multiple steps or components: Step 1: defining performance Step 2: monitoring and evaluating performance Step 3: reviewing performance Step 4: providing consequences Step 1 is to set goals and communicate performance expectations.

A(n) ___is an individual's set of beliefs about the characteristics of a group.

Stereotype

Social Perception

Stereotypes, self fulfilling prophecy and selective perception

John hasn't started working on his term paper. He wants to do well in the course, but he's struggling with how to identify a solid topic and start an outline. Most of the other students appear to be moving ahead. John is procrastination because he is

Stressed

Gerry, one of your subordinates, seems to care so much about being liked that he rarely states strong opinions in meetings of your department. Based on this, Gerry probably has a

Strong need for affiliation

Saying, "You've got to pay your dues" is another way to promote the status quo. This is an example of the ___ option that organizations can use to address any type of diversity issue.

Suppress

___ characteristics are quickly apparent to interactants

Surface-level

______ characteristics are quickly apparent to interactants.

Surface-level Explanation Surface-level characteristics are those that are quickly apparent to interactants, such as race, gender, and age.

rebel.

Tammy participates actively in the weekly team meetings, but when it comes to doing her work, she tends to do it her own way, often disregarding company policies and her boss's wishes. Tammy is probably a(n):

Tanya is the night supervisor for a data processing company. She supervises 26 workers who perform routine jobs that require minimal training. Which of the following statements would indicate that Tanya is following the transformational model of leadership?

Tanya seeks to motivate employees to pursue organizational goals above their own self-interests. Explanation Transformational leaders motivate employees to pursue organizational goals above their own self-interests.

Ellen has worked on a project to market a new medication from beginning to end- advertising, packaging, etc. The design program wins and award. Ellen's ___ is high.

Task identity

_____ are deliberate plans that outline exactly what the team is to do, such as goal setting and defining roles.

Team performance strategies. Explanation Team performance strategies are deliberate plans that outline what exactly the team is to do, such as goal setting and defining particular member roles, tasks, and responsibilities.

Which of the following is not one of the top four skills desired by employers?

Teamwork Explanation According to Table 1.1, the top four skills most desired by employers are critical thinking, problem solving, judgment and decision making, and active listening.

In the TED five-step protocol for effective presentations, TED stands for

Technology, Education, Design. Explanation TED stands for Technology, Education, Design.

___ is one element that allows employees to balance their work lives and family lives.

Telecommuting

Alliance Partners, Strategic Knowledge Workers, Supporting Labor and Core Employees describe

The four contract relationships of the Human Capital Architecture

power distribution.

The highest degree of empowerment is:

leader-member exchange.

The leadership theory that is based on the assumption that leaders develop unique one-to-one relationships with each of the people reporting to them is:

Idea generation

The making of new mental connections regarding the creative task or problem at hand is represented in the _________ stage of the creative performance process.

The idea of perceived organizational support (POS) is that if treated well, employees are motivated by:

The norm of reciprocity.

receivers create meaning in their own minds.

The perceptual model of communication depicts communication as a process in which:

Participate Management

The process whereby employees play a direct role in (1) setting goals, (2) making decisions, (3) solving problems, and (4) making changes in the organization. Participate management includes, but goes beyond, simply asking employees for their ideas or opinions.

Which of the following statements is true of two-way linkages?

The strategic planning function and the human resource management function are interdependent.

helper.

The type of follower who is most compliant with leadership is the:

At Cornet Company, employees and teams had higher performance, greater job satisfaction, and increased organizational commitment when their managers showed ________ behaviors.

Theory Y Explanation Theory Y is a modern and positive set of assumptions about people at work: They are self-engaged, committed, responsible, and creative.

Referent

Therese is a charismatic person who is often able to get other employees to work late on special projects by being very friendly and fun to be around. She is exercising her ______ power.

noise.

Therese is making a presentation in one of her classes. Her PowerPoint slides contain several arithmetic errors that Kathy is finding distracting. This is an example of:

Legitimate Power

This base of power is anchored to one's formal position or authority. Thus, individuals who obtain compliance primarily because of their formal authority to make decisions have legitimate power. Legitimate power may express itself in either a positive or negative manner in managing people. Positive legitimate power focuses constructively on job performance. Negative legitimate power tends to be threatening and demeaning to those being influenced. Its main purpose is to build the power holder's ego. Importantly, there is growing concern today about the limits of managers' legitimate power relative to privacy rights and off-the-job behavior.

Coercive Power

Threats of punishment and actual punishment give an individual coercive power. For instance, consider this heavy-handed tactic by Wolfgang Bernhard, a Volkswagen executive: "A ruthless cost-cutter, Bernard, 46, has a favorite technique: He routinely locks staffers in meeting rooms, then refuses to open the doors until they've stripped $1,500 in costs from a future model."

Which of the following states that it is illegal for an employer to refuse to hire an individual or otherwise discriminate against this individual with respect to compensation, terms, conditions, or privileges of employment because of such individual's race, color, religion, sex, or national origin?

Title VII of the Civil Rights Act of 1964

have two-way problem-solving discussions.

To reduce the chances of another person becoming defensive, a manager should:

liking.

Touching another person communicates:

According to researchers, which of the following is a disadvantage to applying situational theories with members of a team? Answers: Employees need to identify situational factors that affect the leadership style. It advocates using one single leadership style for multiple situations. It can lead to a need for identifying outcomes before determining what goals are to be achieved. Treating group members differently resulted in some employees feeling that they were not among the leader's "in-group." It requires changing the person in the leadership role to increase leadership effectiveness.

Treating group members differently resulted in some employees feeling that they were not among the leader's "in-group." Response Feedback: Study findings revealed that treating group members differently resulted in some employees feeling that they were not among the leader's "in-group."

Collecting performance information from multiple sources helps a person being evaluated to get a broad view of his or her performance.

True

Maslow's need hierarchy and Herzberg's motivator-hygiene theories are both examples of process theories.

True

Negative feedback can have a positive motivational effect

True

The acquired needs identified y McClelland include the need for achievement, the need for affiliation, and he need for esteem.

True

The functions of performance management processes include guiding employee development and making employee-related decisions.

True

Inspirational Appeals (soft)

Trying to build enthusiasm by appealing to others' emotions, ideals, or values

Rational Persuasion (soft)

Trying to convince someone with reason, logic, or facts

Which of the following is not one of the suggestions coming out of the TED protocol?

Use the latest technology available to appear to be up to date. Explanation In Step 4, plan your multimedia, don't feel compelled to use the latest and greatest or any technology at all. But whatever you choose, keep it simple and don't let it distract the audience.

Expert

Valued knowledge or information gives an individual ________ power over those needing that information.

Expert Power

Valued knowledge or information gives an individual expert power over those who need such knowledge or information. The power of supervisors is enhanced because they know about work schedules and assignments before their employees do. Skillful use of expert power played a key role in the effectiveness of team leaders in a study of three physician medical diagnosis teams.33 Knowledge is power in today's high-tech workplaces.

__________ are the most effective schedules of reinforcement.

Variable ratio and variable interval Explanation Variable ratio and variable interval schedules of reinforcement produce the strongest behavior that is most resistant to extinction.

_______ represents what individuals and organizations aspire to be when they are at their very best.

Virtuousness Explanation Virtuousness represents what individuals and organizations aspire to be when they are at their very best.

____ is the need to see one or more alternative paths to achieve a goal, even when faced with adversity.

Waypower Explanation The need to see one or more alternative paths to achieve your goal even when faced with adversity is waypower.

We have the cognitive capacity to process words at a much higher rate than people speak. Which of the following can result from this?

We miss or lose some of what we hear. Explanation Cognitive processes are being underutilized, leading to daydreaming and distractions, which leads us to miss some of what we hear.

Individual differences around the globe

When we refer to a country's culture we are referring to values that distinguish on nation from another.

Include questions on slides.

Which of the following is a tip for more effective PowerPoint presentations?

Economic exchange

Which of the following is not a characteristic of an in-group exchange?

Mutual liking

Which of the following is not a characteristic of an out-group exchange?

People tend to resent and disrespect credible experts.

Which of the following is not a principle of persuasion?

Embrace the most current technology

Which of the following is not a suggestion as to improving communications between generations?

Overestimation of opponents

Which of the following is not a symptom of groupthink?

Less pressure to conform

Which of the following is not an advantage of group decision making?

Stay away from defining what is considered illegal.

Which of the following is not an element of an effective social media policy?

Conveying routine feedback

Which of the following is not an example of a crucial conversation?

Managers possess complete information

Which of the following is one of the assumptions underlying nonrational models of decision making?

Evaluation of leadership effectiveness depends on what the evaluator wants.

Which of the following statements about assessing leadership effectiveness is true?

Impression management is generally damaging to careers.

Which of the following statements about impression management is false?

Demographic characteristics such as gender have no effect on leadership behaviors.

Which of the following statements about leadership effectiveness is false?

It is not useful for understanding differences between generations

Which of the following statements about linguistic style is not true?

Coercive power has a slightly negative effect.

Which of the following statements about the effect of power is true?

It hinders effective teamwork

Which of the following statements about the use of e-mail is false?

Jason is working at Sushow Motors. The position, as originally described, involved conducting research with customers. Instead, Jason is answering phones and filing. He's very unhappy and thinks he should find another job. This thought process is

Withdrawal Cognitions

According to the original path-goal theory, which of the following is NOT a contingency factor in leadership effectiveness? Answers: Employees' need for achievement Employees' need for clarity Employees' task ability Employees' work experience Work-unit performance

Work-unit performance Response Feedback: Five important employee characteristics that act as contingency factors are locus of control, task ability, need for achievement, experience, and need for clarity. Two relevant environmental factors are task structure (independent versus interdependent tasks) and work-group dynamics.

Empowerment is a matter of degree, not an either-or proportion.

You are a management consultant. An executive from Swell Systems asks you to present a workshop on empowerment to her managers. Which of the following ideas should you include?

Expert and referent power had a generally positive impact

You are a management consultant. During a training session you are conducting, a manager from XYZ Energy Corporation asks you to summarize the best research evidence on the impact of the five bases of power on job performance, job satisfaction, and turnover. Which of these would be a correct response?

Exchange

You are late in the preparation of the computer graphics for your final report and presentation. You run into a friend who is great at computer graphics as he heads out of the office at the end of the day and say, "I need your help for my project. If you could come back for two or three hours tonight and help me with these graphics, I will complete those spreadsheets you have been complaining about." Which influence tactic are you using with your friend?

Networks are people-oriented.

You own your own management consulting firm. At a recent seminar, the owner of a local supermarket chain asks you to help him differentiate between a coalition and a network. Which of the following would be a correct response?

Referent

_______ power comes into play when one's personal characteristics and social relationships become the reason for compliance.

Job design theory

__________ is one form of motivation by which a manager can accomplish structural empowerment.

Expectations and norms

___________ about communication represent the key issue when considering communication across generations.

Reward; compliance

____________ power tends to produce __________.

The manager of the Conquest Casino notices that the security employees at the casino are treating customers impersonally. He sets a goal that they should make better eye contact and treat customers in a friendlier manner. This is a(n) _______ goal.

`Behavior

Majority rule

a decision making rule in which each member of the group is given a single vote and the option receiving the greatest number of votes is selected

The Ethics Review Board at XYZ College is composed of students who were elected by the student body. When a student is charged with an ethics violation, the board meets to hear both sides of the dispute in a confidential meeting. Their decisions are not binding on the parties, according to college policies. They are acting as

a peer review team. Explanation They are acting as a peer review team. A peer review team is a panel of trustworthy coworkers (in this case, fellow students), selected for their ability to remain objective, that hears both sides of a dispute in an informal and confidential meeting. Any decision by the review panel may or may not be binding, depending on the company's ADR policy.

Research has shown that

a person's behavior and performance are a function of interdependent person and situation factors. Explanation Many observers believe that some people are by their nature better suited than others to perform well at work. In contrast, others believe that some people are clearly better in a given job or situation. Nobody is the best at everything. This common view is supported by research in psychology and OB. Notably, the interactional perspective states that behavior is a function of interdependent person and situation factors.

1. Which of the following statements about the use of e-mail is false? a. It hinders effective teamwork b. It can contribute to information overload c. It can distract people from their work responsibilities d. It increases employees' flexibility e. It reduces cost of distributing information

a. It hinders effective teamwork

1. Of the following, which is the most closely aligned with employees' perceptions of procedural justice? a. Job performance b. Absenteeism c. Stress d. Health problems e. Mental health

a. Job performance

1. Compassion is: a. a shared value that drives people to help others who are suffering. b. a shared belief in showing restraint and control. c. a shared belief in self-control, humility, and prudence. d. the same as virtuousness. e. a shared value in the importance of resolving conflict multilaterally.

a. a shared value that drives people to help others who are suffering.

1. Forming an overall impression about a person or object and then having that impression bias ratings about that person or object is known as the ________ perceptual error. a. halo b. leniency c. central tendency d. recency e. contrast effects

a. halo

1. People who want to control others and often manipulate people for their own gratification are described as: a. high in personal power. b. high in institutional power. c. high in need for affiliation. d. high in need for achievement. e. high in Theory X.

a. high in personal power.

1. The characteristic of PsyCap that consists of persevering toward goals is: a. hope. b. efficacy. c. optimism. d. health. e. resilience.

a. hope.

1. A head chef of a restaurant always encouraged his subordinates to be creative in the culinary art. He did not believe in using prescribed recipes while cooking. He allowed them to create unique recipes and use different ingredients to prepare dishes. This implied that the head chef established _________ goals. a. learning b. procedural c. performance d. equity e. project

a. learning

1. In regard to differences in how men and women communicate, in general: a. males are expected to hide their emotions. b. males are expected to communicate less aggressively. c. females focus less on rapport. d. females give less confirmation and support. e. differences between males and females are insignificant.

a. males are expected to hide their emotions.

1. The steps of implementing a goal-setting program are, in order: a. set the goal, promote goal commitment, provide support and feedback, and create action plan. b. set the goal, provide support and feedback, promote goal commitment, and create the action plan. c. set the goal, provide support and feedback, create an action plan, and promote goal achievement. d. set the goal, create the action plan, promote goal commitment, and provide support and feedback. e. set the goal, create the action plan, provide support and feedback, and promote goal commitment.

a. set the goal, promote goal commitment, provide support and feedback, and create action plan.

1. A low valence means that: a. the person does not value the outcomes. b. the person does not trust the company. c. the person does not feel confident in his/her ability to do the job. d. the person is not motivated. e. the person is highly motivated.

a. the person does not value the outcomes.

Bounded rationality model

according to this model individuals knowingly limit their options to a manageable set and choose the first acceptable alternative without conducting an exhaustive search for alternatives. Refers to the tendency to satisfice

According to path-goal theory, encouraging employees to perform at their highest level by setting challenging goals, emphasizing excellence, and demonstrating confidence in employee abilities refers to the _____ leadership style. Answers: supportive achievement-oriented directive participative authoritarian

achievement-oriented Response Feedback: According to path-goal theory, encouraging employees to perform at their highest level by setting challenging goals, emphasizing excellence, and demonstrating confidence in employee abilities is achievement-oriented leadership behavior. Refer: Table 14-2

You're Kidding Me, Inc., is a company that develops video games. The company's primary vision is innovation and quick adaptation to the changing environment of the gaming industry, particularly for children. The CEO of the company values creativity and frequently urges her employees to think outside-the-box in order come up with the next big idea. Which of the following best describes You're Kidding Me's organizational culture?

adhocracy Explanation Companies with an adhocracy culture have an external focus and value flexibility. Creation of new products and services is the strategic thrust of this culture, which is accomplished by being adaptable, creative, and fast to respond to changes in the marketplace.

Sally doesn't complain to a man smoking in the nonsmoking section of a restaurant because it doesn't bother her. This lack of irritation reflects the ______ component of her attitude. She is neutral toward people smoking in restaurants.

affective Explanation The affective component of an attitude contains the feelings or emotions one has about a given object or situation. You are unlikely to say anything to someone in this situation if you are not irritated by this behavior (affective).

Manuel wanted friendly and close interpersonal relationships with his colleagues. According to McClelland's need theory, this desire is known as a need for

affiliation. Explanation The need for affiliation is the desire to maintain social relationships, to be liked, and to join groups.

Jennifer wins an art contest. She attributes the outcome to luck. Jennifer is likely to have

an external locus of control. Explanation Those who believe their performance is the product of circumstances beyond their immediate control possess an external locus of control.

People with __________ see themselves as masters of their own fate.

an internal locus of control Explanation People who believe they control the events and consequences that affect their lives are said to possess internal locus of control. They see themselves as masters of their own fate.

Identifying and modeling appropriate ways for people to interact with colleagues is a(n) ________ strategy.

antibullying Explanation Anti-bullying strategies for groups and organizations include: -Develop a workplace bullying policy. -ncourage open and respectful communication. -Develop a clear procedure for handling complaints about bullying. -Identify and model appropriate ways for people to interact with colleagues. -Develop and communicate a system for reporting bullying. -Identify and resolve conflicts quickly and fairly to avoid escalation. -Identify the situations, policies, and behaviors likely to cause bullying or allow it to occur. -Train employees to manage conflict. -Establish and enforce clear consequences for those who engage in bullying. -Monitor and review employee relationships, with particular attention to fairness.

Francis interviews two candidates for an administrative assistant position that will require a fair amount of project management. The first candidate is enthusiastic and states that she can "do anything she sets her mind to." She shows up for the interview without a résumé because she know she will be selected for the position. The second candidate is more reserved, but offers Francis his résumé and a portfolio. He explains that in his last position he managed several projects and kept them on track. Which of the following would be the main reason for Francis to hire the second applicant?

applicant's cognitive abilities Explanation Many effective managers select employees based on positive, job-relevant, but relatively stable individual differences. Cognitive abilities is a relatively fixed individual difference.

Intuitive decision making model

arriving at decisions without conscious reasoning. The model argues that in a given situation experts marking decisions scan the environment for cues to recognize patterns

ABC Airlines hires flight attendants from diverse backgrounds and puts them through an extensive training and orientation program before they are approved to fly. In addition to learning about responding to medical issues and FAA guidelines, they also must learn the company's many rules and processes as contained in an extensive procedures and policies manual. ABC Airlines is adopting a(n) ______ option for managing diversity.

assimilate Explanation The basic premise behind the assimilate option is that all diverse people will learn to fit in or become like the dominant group. It only takes time and reinforcement for people to see the light. Organizations initially assimilate employees through their recruitment practices and the use of company-orientation programs. New hires generally are put through orientation programs that aim to provide employees with the organization's preferred values and a set of standard operating procedures. Employees then are encouraged to refer to the policies and procedures manual when they are confused about what to do in a specific situation. These practices create homogeneity among employees.

Fred always announces his decisions at weekly staff meetings. He doesn't waste time asking for employee input because he already knows what he wants to do. This reflects ________, the lowest level of empowerment.

authoritarian power Explanation According to Figure 12.6, the lowest level of empowerment is authoritarian power.

1. _________ happens when our minds are racing or wandering, resulting in compulsive daydreaming or fantasizing. a. Mindlessness b. Attentional hyperactivity c. Positive deviance d. PsyCap e. Negative emotion

b. Attentional hyperactivity

What does the Myers Briggs type indicator classification of S versus N stand for? a. Social/Perceiving b. Sensing/Intuitive c. Stable/Negligent d. Shrewd/Feeling

b. Sensing/Intuitive

1. CRF schedules are particularly useful when an employee is: a. missing quotas. b. learning a new skill. c. frequently absent from work. d. performing far above expectations for his or her job. e. feeling entitled to receiving a reward.

b. learning a new skill.

Based on the contingency approach, to define performance ___ goals can be used in most jobs

behavioral

internal locus of control

believe that they control their own destiny and what happens to them is their own doing, In fact, one study showed that having internal locus of control at the age of 10 was related to a number of health outcomes, such as lower obesity and lower blood pressure later in life. It is possible that internals take more responsibility for their health and adopt healthier habits,while externals may see less of a connection between how they live and their health

Meg manages a small nonprofit group that assists abused women. Recently Sarah, one of her employees, notices that Meg has been more focused on getting good press for herself than for the women that she helps. Meg wants to position herself to win the Personal Service Award from the mayor of their community. Sarah thinks they should work towards positioning their group for additional funding. In this case, political tactics that counter Meg's interests will likely

benefit the organization. Explanation When destructive leaders create organizational goals and objectives to suit their own interests—their egos or their personal legacies. Political tactics that counter these interests are likely to be positive for the organization.

The existence of implicit cognition leads people to make ________ decisions without realizing they are doing so.

biased Explanation The existence of implicit cognition leads people to make biased decisions without realizing they are doing so. A recent study of job applicants' résumés, for instance, demonstrated that recruiters evaluated women more favorably than men for customer service jobs, probably based on gender-role stereotypes.

Mosco's is a ____________ in which management has largely succeeded in breaking down barriers among internal levels, job functions, and departments, as well as reducing external barriers between the organization and those with whom it does business.

boundaryless organization Explanation A boundaryless organization is one where management has largely succeeded in breaking down barriers among internal levels, job functions, and departments, as well as reducing external barriers between the association and those with whom it does business.

According to Barbara Fredrickson's theory, positive emotions

broaden your perspective and build on themselves. Explanation Barbara Fredrickson's "broaden and build" theory states that positive emotions broaden your perspective about how to overcome challenges in your life. Positive emotions also build on themselves, resulting a spreading of positive emotions within yourself and those around you.

1. Which of the following statements about mindfulness is false? a. Mindfulness pertains to our inner world of thinking and feeling. b. Mindfulness pertains to our outer worlds of other people and events. c. Mindfulness requires less effort than letting our minds wander. d. Mindfulness is about concentrating or focusing. e. Mindfulness represents how much we are aware and attentive to what is going on around us.

c. Mindfulness requires less effort than letting our minds wander.

1. In Herzberg's motivator-hygiene theory, a. Hygiene factors come from the job itself. b. Motivators come from the work context. c. Satisfaction comes from motivating factors, and dissatisfaction comes from hygiene factors. d. Satisfaction comes from hygiene factors, and dissatisfaction comes from motivating factors. e. Job satisfaction and job dissatisfaction are opposites.

c. Satisfaction comes from motivating factors, and dissatisfaction comes from hygiene factors.

1. Which of the following is not a form of social support? a. Providing information that a person is accepted and respected b. Providing financial aid, material resources, or needed services c. Spending time discussing your own vision for yourself d. Spending time with others in leisure and recreational activities e. Providing help in defining, understanding, and coping with problems

c. Spending time discussing your own vision for yourself

1. Brian has a Master's degree in electrical engineering and brings 10 years of experience to his current job. He is known for putting in long hours of hard work on his job. He receives a salary of $75,000, medical and dental coverage, and two weeks paid vacation per year. From Brian's perspective, which of the following is an output of the individual-organization exchange relationship? a. Long hours of work b. High levels of effort c. Two weeks paid vacation d. His Master's degree in electrical engineering e. Ten years of prior experience

c. Two weeks paid vacation

1. ***Positive practices that escalate positive outcomes because of their associations with positive emotions and social capital are having a(n) __________ effect. a. positivity b. buffering c. amplifying d. mindful e. attentional

c. amplifying

1. The idea that a manager should do what the situation requires is captured by the term: a. integrative framework. b. interactional perspective. c. contingency approach. d. performance management. e. performance appraisal.

c. contingency approach.

1. The first step in effective performance management is: a. providing consequences. b. reviewing performance. c. defining performance. d. evaluating performance. e. monitoring performance.

c. defining performance.

1. Expectancy theory contains three elements; they are: a. expectancy, instrumentality, values. b. expectancy, institution, values. c. expectancy, instrumentality, valence. d. expectancy, institution, variance. e. expectancy, instrumentality, variance.

c. expectancy, instrumentality, valence.

1. Which of the following is not one of the suggestions coming out of the TED protocol? a. you should never read your presentation. b. the most important element of stage presence is eye contact. c. use the latest technology available to appear to be up to date. d. prepare far enough in advance. e. conduct a practice session.

c. use the latest technology available to appear to be up to date.

Affirmative action plans

can negatively affect women and minorities. Explanation Affirmative action plans are found to negatively affect the women and minorities expected to benefit from them. Research demonstrates that women and minorities supposedly hired on the basis of affirmative action feel negatively stigmatized as unqualified or incompetent.

Ryan, Jerry, and Mark belong to Sigma Phi Upsilon fraternity. They pride themselves on being a group of solid students with leadership abilities. As a result, they often seem to hijack Fraternity Council Leadership meetings. Clearly, they feel they know how all the fraternities on campus should operate. This very high level of cohesiveness in a group.

causes groupthink. Explanation Too much cohesiveness can breed groupthink because a desire to get along pushes aside critical thinking.

The basic premise behind strategy implementation is that

choice of organizational processes and structural forms makes an economic difference.

What cultural types represent competing values?

clan and market Explanation Certain cultural types reflect opposing core values. These contradicting cultures are found along the two diagonals in Figure 14.5. For example, the clan culture (upper-left quadrant) is represented by values that emphasize an internal focus and flexibility, whereas the market culture (bottom-right quadrant) has an external focus and concern for stability and control.

A(n) ________ is a group of people actively pursuing a particular issue.

coalition Explanation A coalition is an informal group bound together by the active pursuit of a single issue.

Alexa, a manager, explains to Harvey that if he is late to work one more time in the next six weeks he will be suspended without pay for a day. Alexa is exercising _______ power.

coercion Explanation The ability to make threats of punishment and deliver actual punishment gives an individual coercive power.

_________ is the psychological discomfort a person experiences when simultaneously holding two or more conflicting cognitions.

cognitive dissonance Explanation Cognitive dissonance represents the psychological discomfort a person experiences when simultaneously holding two or more conflicting cognitions (ideas, beliefs, values, or emotions).

Tony has a full courseload and a full-time job. He has tried everything he can think of to help him deal with his stress, but nothing has worked. He plans to go see a counselor next week. What method relies on therapeutic intervention?

cognitive restructuring Explanation Cognitive restructuring is a stress-reduction technique where irrational or maladaptive thoughts are identified and replaced with those that are rational or logical. See Table 16.4.

On her first day of work for ABC Company, Justine attended a full-day session with other new employees to learn about company policies and procedures. Each person in that session was introduced to a specific person whom they would all meet with once a week over the next two months to answer their questions about the company. This experience would be described as

collective and serial. Explanation The collective tactic is grouping newcomers and exposing them to a common set of experiences (e.g., all new hires attend an orientation session on the same day). The serial tactic of the newcomer is socialized over time with the help of an experienced member (e.g., a buddy system of orientation). See Table 14.2.

Norman is elected president of the debate club. He is known as a team player. That means that Norman is

committed, collaborative, and competent. Explanation The 3 Cs of team players are committed, collaborative, and competent.

The three key elements of equity theory are outputs, inputs, and a(n)

comparison of the ratio of outputs to inputs. Explanation If the comparison person enjoys greater outcomes for similar inputs, negative inequity will be perceived.

The _________ framework provides a practical way for managers to understand, measure, and change organizational culture.

competing values Explanation The competing values framework (CVF) provides a practical way for managers to understand, measure, and change organizational culture.

Dr. Davis, an accounting professor, is concerned that he was not engaging his students as much as he would like. He asks his boss, the department chair, and two of his colleagues to sit in on one of his classes, to give him comments and suggestions for improvement. He also passes out student evaluation forms. Dr. Davis is

conducting a 360-degree feedback process. Explanation In 360-degree feedback, individuals compare perceptions of their own performance with behaviorally specific performance information from their manager, subordinates, and peers.

Evidence collected after the fact can be used to convince an external audience that the organization is following a sound course of action in a complex and ambiguous decision context. On the positive side, this can lead to

confidence and goodwill about the company. Explanation On the positive side, evidence collected after the fact can be used to convince an external audience that the organization is following a sound course of action in a complex and ambiguous decision context. This can lead to confidence and goodwill about how a company is responding to events.

In ________, one party perceives that its interests are being opposed or negatively affected by another party.

conflict Explanation Conflict occurs when one party perceives that its interests are being opposed or negatively affected by another party.

Alexis, a customer service supervisor at ABC Airlines, typically seeks the full participation of her team members when planning, making important decisions, and implementing changes. Which one of the nine generic influence tactics is she using?

consultation Explanation Consultation is getting others to participate in planning, making decisions, and changes.

There are two key situational characteristics that affect perception: the ___ of the interaction, and the culture and race___ between perceivers

context; consistency

Pop's Ice Cream Store sales were down 15 percent this summer. The owner, Pop, says that in his experience rainy summers mean lower ice cream sales. Jonathan, his assistant, notices the staff at Pop's appear uninterested and slow to serve customers. He would like to replace at least one of them with a younger college student for the rest of the summer. Jonathan is using a(n) ______ perspective.

contingency Explanation Common sense is often based on experience or logic, both of which have limits, and it suffers three major weaknesses you need to be aware of—over-reliance on hindsight, lack of rigor, and lack of objectivity. An important part of your success is your ability to know which tools to use and under what circumstances. This is described as a contingency approach to managing people.

Linda wants to leave her company. No one ever seems to do what they say they will do. In addition, her manager's expectations seem to change from one day to the next. Linda is missing

contractual trust. Explanation Contractual trust is trust of character.

Which of the following is an organizational function of a group?

coordinate interdepartmental efforts Explanation According to Table 8.1, coordinating interdepartmental efforts is one organizational function of a group.

Employees with skills to perform a pre-defined job that are quite valuable to a company, but not particularly unique or difficult to replace. Alliance Partners, Strategic Knowledge Workers, Supporting Labor and Core Employees describe

core employees

The idea behind the job characteristics model is that

core job characteristics lead to critical psychological states. Explanation Hackman and Oldham proposed that intrinsic motivation is determined by three psychological states. These psychological states are fostered by the presence of five core job characteristics. The model is illustrated in Figure 5.10.

Eleanor wants to be sure that the final team project in a senior capstone course is of top quality. Jerome, another team member, just wants to get it done quickly. Using Azjen's model, what can Eleanor do to try and encourage Jerome to work hard?

create a positive attitude

In an out-group exchange, a leader is attempting to: -create more formality in expectations and rewards. -create trust and mutual obligation. -inspire followers. -perform service to others. -initiate structure.

create more formality in expectations and rewards

In an in-group exchange, a leader is attempting to: -perform service to others. -create more formality in expectations and rewards. -inspire followers. -initiate structure. -create trust and mutual obligation.

create trust and mutual obligation

Managers should attempt to raise subordinates' self-efficacy by all of the following except

creating routine jobs that are boring but easy to perform

XYZ Inc. brings together specialists from production, marketing, and finance from around the world, and gives each such team the power to make its own decisions. This implies that the firm is creating a(n) _____ team.

cross-functional Explanation Cross-functionalism occurs when specialists from different areas are put on the same team. XYZ Inc. is creating a cross-functional team as it brings together specialists from production, marketing, and finance from around the world.

JM Video Games uses ________ to help design new products. It reviews proposed designs twice a year, and the winner of the design competition is awarded $2,500.

crowdsourcing Explanation Crowdsourcing occurs when companies invite nonemployees to contribute to achieving particular goals and manage the input process via the Internet.

People who are high on openness to experience are likely to demonstrate

curiosity and broad-mindedness. Explanation According to Table 3.2, someone who scores high on openness to experience would be intellectual, imaginative, curious, and broad-minded.

1. Which of the following is not an outcome likely to result from positive emotions? a. Stronger social relationships b. Lower stress c. Psychological well-being d. More conformity to others' opinions e. Fewer colds

d. More conformity to others' opinions

1. ________ is anything that interferes with the transmission and understanding of a message. a. Feedback b. Decoding c. Encoding d. Noise e. Media richness

d. Noise

______ are indicators of a strong organizational culture. a. High levels of dissension b. High rates of turnover c. Narrowly defined roles d. Widely shared values

d. Widely shared values

1. Allen used to make suggestions to his boss about how the department's performance could be improved. Although his boss always said they were great ideas, they were never acted upon. Allen then stopped making suggestions. This is an example of: a. punishment. b. negative reinforcement. c. positive reinforcement. d. extinction. e. the law of effect.

d. extinction.

Some organizations block employees' access to social media sites. This can

damage employee morale and loyalty. Explanation Blocking social media sites -can alienate employees. -is only fair if employers expect employees to be connected and responsive 24/7 to work-related e-mails, such as those from managers, coworkers, clients, or suppliers. -suggests a lack of trust.

Non-programmed decisions

decisions that requires conscious thinking, information gathering, and careful consideration of alternatives

Chuck is concerned with what he considers to be an unfair situation at work. Although he put in 10 hours of overtime last week, he received the same pay as a co-worker who didn't put in any overtime. This week he decided to take an extra hour for lunch every day. Using the terminology of equity theory, Chuck has decided to ____. increase his inputs decrease his outcomes increase his outcomes increase his efficiency decrease his inputs

decrease his inputs

Marge is managing several new strategies. Her team knows that she is under pressure to bring all these projects in on schedule and within budget. Marge wants to empower Jason, her assistant project manager, to lead for self-determination or choice. She should

delegate important tasks to Jason. Explanation Managers lead for self-determination by delegating meaningful assignments and tasks.

In power distribution, the manager adopts the _____ style.

delegation Explanation In power distribution, the manager adopts the delegation style. See Figure 12.6.

Which of the following statements about leadership effectiveness is false? -Individual differences significantly impact leadership effectiveness. -Demographic characteristics such as gender have no effect on leadership behaviors. -Researchers have estimated that 50 percent of all managers around the world are incompetent or ineffective. -Leadership effectiveness is partly a function of situational factors. -You do not need to have a formal position of authority to be a leader.

demographic characteristics such as gender have no effect on leadership behaviors

Roberto is meeting with Professor Plum in his office during advising hours. As Roberto is describing the problems he is having in his accounting course, Professor Plum is checking his e-mail. Professor Plum is exhibiting the ________ style of listening.

detached Explanation Detached listeners tend to withdraw from the interaction. They appear inattentive, bored, distracted, and uninterested. They may start using mobile devices during the speaking-listening exchange. Body language will reflect lack of interest, such as slumping and avoiding direct eye contact. Professor Plum is exhibiting a detached listening style checking his e-mail while Roberto talks.

The stages of OD are, in order,

diagnosis, intervention, evaluation, and feedback. Explanation Four stages of OD are: diagnosis, intervention, evaluation, and feedback. See Figure 16.7.

Motivation refers to the psychological processes that underlie: extrinsic and intrinsic factors. power, achievement, and affiliation. content and process. satisfaction and dissatisfaction. direction, intensity, and persistence of behavior or thought.

direction, intensity and persistence of behavior or thought

An employee with high task ability and experience is less apt to need additional guidance and thus would respond negatively to _____ leadership. Answers: participative achievement-oriented appreciative performance-oriented directive

directive Response Feedback: An employee with high task ability and experience is less apt to need additional direction and thus would respond negatively to directive leadership.

Harold's boss has just told him that he will be moved to a new office in order to allow space for a new copier for the office. Harold is very unhappy with the prospect of having to pack up all his things and then reorganize them in his new area. He also is having trouble understanding why the new copier has to be in his space, instead of someone else's. How would you describe the most likely cause of Harold's resistance?

dispositional resistance Explanation Individuals with a high dispositional resistance to change are less likely to voluntarily initiate changes and more likely to form negative attitudes toward the changes they encounter.

In some cases, coworkers or friends remain satisfied in situations where others always seem dissatisfied. This is a reflection of the _____ model.

dispositional/genetic Explanation The dispositional/genetic model posits that job satisfaction is a function of both personal traits and genetic factors. The model implies that stable individual differences are at least as powerful as characteristics of the work environment in their impact on satisfaction.

You have an apartment within walking distance of your office. Your company moves the office to the next town. You decide to take a position with another firm close to your apartment. This is an example of _____ the problem.

dissolving Explanation Dissolving problems requires changing or eliminating the situation in which the problem occurs.

Professor Jones learns that faculty members in another college with the same degree, years of teaching experience, and publication record are making significantly more money than he is. He considers this unfair. Which aspect of justice is he reacting to? Organizational Interactional Institutional Distributive Procedural

distributive

Professor Jones learns that faculty members in another college with the same degree, years of teaching experience, and publication record are making significantly more money than he is. He considers this unfair. He is reacting to ______ justice.

distributive Explanation Distributive justice reflects perceived fairness of how resources and rewards are distributed or allocated.

The two basic types of negotiation are

distributive and integrative. Explanation Negotiation experts often distinguish between two fundamental types of negotiation: distributive and integrative.

As HR Director at a large manufacturing firm, it is Henry's responsibility to ensure that employees are properly compensated according to a biweekly time schedule. Many of Henry's employees chose to have their pay checks automatically deposited into their designated bank accounts. However, it is still necessary to inform these workers that their pay has been properly deposited. Which of the following communication media should Henry choose to convey this information?

e-mail Explanation Communication effectiveness is partly based on using the medium that is most appropriate for the situation at hand. Newsletters, computer reports, and general e-mail blasts are lean media because feedback is very slow, the channels involve only limited visual information, and the information provided is generic or impersonal. E-mail is appropriate for Henry's situation.

1. Which of the following is not a part of effective performance management? a. Reviewing performance b. Monitoring performance c. Providing consequences d. Defining performance e. All of these are parts of effective performance management

e. All of these are parts of effective performance management

1. Which of the following is true about balancing positive and negative emotions? a. We should try to eliminate all our negative emotions. b. We should have one negative emotion for each positive emotion we experience. c. Our brains actually look more for positive information over negative during daily activity. d. Positive experiences activate a survival orientation. e. Ideally, we should try to have more positive experiences than negative.

e. Ideally, we should try to have more positive experiences than negative.

1. Which of the following statements about linguistic style is not true? a. It is a set of culturally learned signals b. It affects how we communicate what we mean c. It involves interpreting others' meaning d. It is useful for understanding differences between men and women e. It is not useful for understanding differences between generations

e. It is not useful for understanding differences between generations

1. Well-being is the combined impact of all but one of the following. Which one? a. Engagement b. Relationships c. Achievement d. Meaning e. Productivity

e. Productivity

1. Which of the following is not a factor in media richness? a. How fast it provides feedback b. How many visual and audio cues it provides c. Whether the communication is personal or impersonal d. Language source e. Whether a medium involves electronic methods

e. Whether a medium involves electronic methods

1. Barbara is training her new puppy to sit on command. Every time the puppy responds correctly, he receives a treat. Barbara is using a __________ reinforcement schedule. a. fixed reward b. variable ratio c. fixed interval d. variable interval e. continuous

e. continuous

1. Evaluating people or objects by comparing them with characteristics of other recently observed people or objects is known as the ________ perceptual error. a. halo b. leniency c. central tendency d. recency e. contrast effects

e. contrast effects

1. According to expectancy theory, a person's motivation will be highest when: a. expectancy is high, but instrumentality is low. b. instrumentality is high, but valence is low. c. valence is high, but expectancy is low. d. expectancy, instrumentality, valence are all moderate. e. expectancy, instrumentality, valence are all high.

e. expectancy, instrumentality, valence are all high.

1. A state of reduced attention, expressed in behavior that is rigid, is called: a. mindfulness. b. attention deficit. c. attention delinquency. d. positive deviance. e. mindlessness.

e. mindlessness.

Which of the following is not a characteristic of an in-group exchange? -Mutual respect -Reciprocal influence -Social relationship -Sense of common fates -Economic exchange

economic exchange

The advantage of _________ as a means of overcoming change is that once persuaded, people will often help with the implementation of the change.

education and communication Explanation The advantage of the education and communication approach is that once persuaded, people will often help with the implementation of the change. See Table 16.3

Personality

encompasses the relatively stable feelings, thoughts, and behavioral patterns a person has. . Our personality differentiates us from other people, and understanding someone's personality gives us clues about how that person is likely to act and feel in a variety of situations. . Having this knowledge is also useful for placing people in jobs and organizations. Personality traits are not stable, change with experiences throughout life. Is our behavior in organizations dependent on our personality? Yes and no. While we will discuss the effects of personality for employee behavior, you must remember that the relationships we describe are modest correlations. For example, having a sociable and outgoing personality may encourage people to seek friends and prefer social situations. This does not mean that their personality will immediately affect their work behavior.

In which stage of the organizational socialization process would an onboarding program be provided?

encounter Explanation Many companies use a combination of orientation and training programs to socialize employees during the encounter phase. Onboarding is one such technique.

Which of the following is NOT a task role? -encourager -energizer -recorder -elaborator -information seeker/giver

encourager Explanation According to Table 8.2, Task and Maintenance Roles, encourager is a maintenance role, not a task role.

Meghan is a member of a group that raises money for a homeless shelter. She feels that the group spends too much time arguing the merits of different fundraisers and needs to make a decision. She calls for a vote at today's meeting. Meghan has taken on the role of

energizer. Explanation According to Table 8.2, Task and Maintenance Roles, someone who prods the group to move along or to accomplish more is an energizer.

During the strategic implementation stage of the strategic management process, an organization

ensures that it has skilled employees in place

The millennial generation is described as

entitled, civic-minded, closely involved with parents. Explanation Table 4.3, Generational Differences, lists the broad traits of Millennials as entitled, civic minded, close parental involvement, cyberliteracy, appreciate diversity, multitasking, work/life balance, and technologically savvy.

Which of the following is not an organizational subsystem of an organization?

environmental Explanation Each of the five organizational subsystems—goals and values, technical, psychosocial, structural, and managerial—is dependent on the others.

Luis has just learned that Brian, who has less experience and who does less work, receives a higher salary. Since learning this, Luis has been very unhappy and has started to look for another job. What model of job satisfaction best explains Luis's reactions?

equity

Luis has just learned that Brian, who has less experience and who does less work, receives a higher salary. Since learning this, Luis has been very unhappy and has started to look for another job. What model of job satisfaction best explains Luis's reactions?

equity Explanation Equity theory builds on the notion that satisfaction ties to how fairly an individual is treated at work. Satisfaction results from one's perception that work outcomes, relative to inputs, compare favorably with a significant other's outcomes/inputs.

Waco Company monitors employees' perceptions of fairness by quarterly surveys and occasional focus groups. Management is following the _____ model.

equity Explanation The equity model proposes that management monitor employees' perceptions of fairness and interact with them so they feel fairly treated.

The idea underlying the __________ model is that satisfaction is tied to how fairly an individual is treated at work.

equity Explanation The idea underlying the equity model is that satisfaction is tied to how fairly an individual is treated at work.

The model of motivation that explains how people strive for fairness and justice is: maslow's needs hierarchy theory. value theory. procedural theory. expectancy theory. equity theory.

equity theory

The need for prestige and reputation is part of the need for: esteem. safety. physiological. self-actualization. love.

esteem

The need for self-confidence and strength is part of the need for: esteem. self-actualization. love. safety. physiological.

esteem

Rachel gets her midterm grades and finds that she has a 2.4 in OB. She expected a better grade point average to date. Rachel is _________ her performance.

evaluating

Rachel gets her midterm grades and finds that she has a 2.4 in OB. She expected a better grade point average to date. Rachel is _________ her performance.

evaluating Explanation Evaluating performance is the process of comparing performance at some point in time to a previously established expectation or goal.

Which of the following statements about assessing leadership effectiveness is true? -Leadership effectiveness is simply gaining commitment with our influence attempts. -Evaluation of leadership effectiveness depends on what the evaluator wants. -Leadership can be assessed at the level of the individual leader, but not at group or organizational levels. -Assessing leadership effectiveness is independent of whom is doing the assessment. -The criteria for assessing leadership effectiveness are always clear.

evaluation of leadership effectiveness depends on what the evaluator wants

There are two competing explanations about the origin of linguistic styles between men and women: ______ and ______.

evolutionary psychology; social role theory Explanation There are two competing explanations about the origin of linguistic styles between men and women. Some researchers believe interpersonal differences between women and men are due to inherited biological differences between the sexes. This perspective, also called the evolutionary psychology or Darwinian perspective, attributes gender differences in communication to drives, needs, and conflicts associated with reproductive strategies used by women and men. The second perspective, social role theory, suggests that girls and boys are taught to communicate differently. Here is what these explanations suggest about male and female communication patterns.

Michael wants to be president of his division. Everything he does is geared to that objective. He pushes his employees to to meet higher quotas, criticizes anyone who doesn't put in 10 to 12 hour days, and demands results before the due date. He is arrogant and dominant. Michael is

exhibiting narcissism. Explanation Narcissism is defined as having a self-centered perspective, feelings of superiority, and a drive for personal power and glory.

Ken recently completed his MBA degree. He has received two job offers. Which of the following theories is most useful in predicting the choice that Ken will make? Behavior modification theory Goal setting theory Equity theory Expectancy theory Cognitive dissonance theory

expectancy theory

__________ holds that people are motivated to behave in ways that produce desired combinations of expected outcomes. Goal-setting theory Expectation theory Self-fulfilling prophecy theory Equity theory Expectancy theory

expectancy theory

Expectancy theory contains three elements; they are: expectancy, instrumentality, values. expectancy, institution, values. expectancy, instrumentality, valence. expectancy, instrumentality, variance. expectancy, institution, variance.

expectancy, instrumentality, valence

According to expectancy theory, a person's motivation will be highest when: expectancy is high, but instrumentality is low. expectancy, instrumentality, valence are all high. instrumentality is high, but valence is low. expectancy, instrumentality, valence are all moderate. valence is high, but expectancy is low.

expectancy, instrumentality, valence are all high

Younger employees also are more likely to use the Internet and social media to accomplish their tasks. This is an example of the ___________ that represent the key issue when considering communication across generations.

expectations and norms Explanation Expectations and norms about communication represent the key issue when considering communication across generations.

negative affective people

experience negative moods with greater frequency Negative affective people focus on the "glass half empty" and experience more anxiety and nervousness. When a team comprises mostly negative affective people, there tend to be fewer instances of helping and cooperation. T

Positive affective people

experience positive moods more frequently, Positive affective people tend to be happier at work, [42] and their happiness spreads to the rest of the work environment. . Teams dominated by positive affective people experience lower levels of absenteeism. [43] When people with a lot of power are also high in positive affectivity, the work environment is affected in a positive manner and can lead to greater levels of cooperation and finding mutually agreeable solutions to problems.

Jeff has been a project assistant for XYZ Corporation for five years. Recently he was asked to train Harriet, a new employee. He suggests that she use an Excel worksheet to lay out the projects she is currently working on, and she complies. He is using _______ power.

expert Explanation Valued knowledge or information gives an individual expert power over those who need such knowledge or information.

Seema is married with two children and lives in New York. She has worked at Bubble Zone for 10 years. These are all _______of the four layers of diversity.

external dimensions Explanation External influences are individual differences over which we have more control, such as where we live, our religious affiliation, our marital and parental status, and our work experience. These dimensions also exert a significant influence on our perceptions, behavior, and attitudes.

Organizations with adhocracy cultures are described as

externally focused and valuing flexibility

Allen makes suggestions to his boss about how the department's performance can improve. Although his boss always compliments his ideas, he never acts on them. Allen then stops making suggestions. This is an example of

extinction Explanation Weakening behavior by ignoring it or making sure it is not reinforced is referred to as extinction.

According to research on effective leaders: Answers: masculine-oriented traits and behaviors are more prominent. extraversion is positively related to leadership emergence. intelligence is highly related to leadership effectiveness. intelligence is more important than personality when selecting leaders. changes in leadership prototype will increase biases against women leaders.

extraversion is positively related to leadership emergence. Response Feedback: Extraversion was most consistently and positively related to both leadership emergence and effectiveness. Although past research demonstrated that people were perceived as leaders when they exhibited masculine-oriented traits and behaviors associated with masculinity, and dominance, more recent studies showed an emphasis on more feminine traits and styles that emphasize empowerment, fairness, and supportiveness. This change in prototypes bodes well for reducing bias and discrimination against women in leadership roles.

Marion will receive a promotion and a raise if she completes a difficult assignment. This is an example of

extrinsic motivation. Explanation Extrinsic motivation results from the potential or actual receipt of extrinsic rewards, such as a promotion.

Sharon is the Director of Human Resources at a company that is undergoing a merger with another organization. As a result of the merger, a portion of the combined workforce will be laid off. Sharon understands that even with a lucrative severance package, the news will adversely affect the lives of these employees. Therefore, she wants to break the news to them in a sensitive manner so as to maintain the company's image. Which of the following is the most appropriate communication medium to use in this situation?

face-to-face conversation Explanation Communication effectiveness is partly based on using the medium that is most appropriate for the situation at hand. Face-to-face conversation is the richest form of communication. It provides immediate feedback and allows for the observation of multiple cues such as body language and tone of voice. Face-to-face conversations are useful for communicating about sensitive or important issues that require feedback and intensive interaction.

Roger manages both Bill and Nancy. They are in an argument about who should handle a particular job. Roger encourages them to deal directly with each other in a positive and constructive manner. Roger is acting as a(n)

facilitator. Explanation Roger is acting as a facilitator. A facilitator is a third party, usually a manager, who informally urges disputing parties to deal directly with each other in a positive and constructive manner.

external locus of control

feel that things happen to them because of other people, luck, or a powerful being. Internals feel greater control over their own lives and therefore they act in ways that will increase their chances of success. while being high in external locus is related to a higher rate of depression.

Job satisfaction across the United States has gone up, according to a national survey conducted by the Society for Human Resource Management. The top three facets of satisfaction include

feeling safe at work, having god relationships with coworkers, and having a good relationship with an immediate supervisor

Phil is part of a newly formed work group. He has been introduced to all the group members, but he still feels like he cannot trust them. The group has not yet chosen a leader, and Phil feels unsure about his exact role within the group. According to Tuckman's five-stage model of group development, which of the following stages is Phil's group currently in?

forming Explanation Phil's group is currently in the forming stage of group development. The forming stage of group is known as the ice-breaking stage. During this stage group members tend to be uncertain and anxious about such things as their roles, who is in charge, and the group's goals.

Harry, Marsha, Eve, and Don are discussing ways to increase sales. Harry and Eve want to stick with the current sales promotion plan created four months ago. They say it needs more time to show results. Marsha and Don disagree; they feel sales have continued to decline and will not turn around. As the discussion progresses, various alternatives are proposed. Harry then suggests a modification to the current sales promotion plan that meets with everyone's approval. This is an example of the benefits of ______ conflict.

functional Explanation The distinction between functional conflict and dysfunctional conflict pivots on whether the organization's interests are served. Functional conflict is commonly referred to in management circles as constructive or cooperative conflict and is characterized by consultative interactions, a focus on the issues, mutual respect, and useful give and take. In such situations people often feel comfortable disagreeing and presenting opposing views. Positive outcomes frequently result. Each of these elements is lacking or even opposite in the cases of dysfunctional conflict, which threatens an organization's interests. This highlights the important role of management in determining whether conflict is more or less positive.

During group meetings, Patty feels that everyone should have their say whenever the group makes an important decision. She makes sure that every group member gets the chance to put forth his or her own ideas during the meeting. Which of the following maintenance roles is Patty performing?

gatekeeper Explanation Patty is performing the role of a gatekeeper. Maintenance roles foster supportive and constructive interpersonal relationships. The gatekeeper encourages all group members to participate.

To enhance a worker's expectancy, a manager could: set specific, measurable goals. give feedback and coach the employee. link rewards to desired outcomes. get to know what rewards his/her employees value. explain to the worker why he/she should trust the organization.

give feedback and coach the employee

Which of the following statements about goals and goal setting is not true? Specific goals lead to higher performance Goal achievement leads to job satisfaction Goals should be set at a level just beyond reachable ("stretch" goals) Goals increase persistence Goals direct attention

goals should be set at a level just beyond reachable

Fiona is the daytime supervisor for an online sales company. She supervises 57 people who perform relatively routine jobs that require minimal training. The shop is unionized, so Fiona has little latitude when it comes to dismissing poor performers. However, she does have the authority to transfer employees and has been known to reassign poor performers to less desirable jobs on in the company. Fiona has a reputation for only transferring employees with just cause, and generally is viewed as a fair supervisor by her employees. Although she would like greater autonomy to determine salaries, Fiona generally divides bonuses and raises equally among her employees. To do otherwise would likely create conflict with union members and representatives. According to Fiedler's model, how would Fiona's leader-member relations be described?

good Explanation Leader-member relations reflect the extent to which the leader has the support, loyalty, and trust of the work group. This dimension is the most important component of situational control. Good leader-member relations suggest that the leader can depend on the group, thus ensuring that the work group will try to meet the leader's goals and objectives.

Which of the following is not a benefit of horizontal structures?

greater opportunities for functional specialization Explanation Horizontal designs generally improve coordination and communication in organizations. Cross-functional teams can arrive at creative solutions to problems that arise in a fast-changing environment. Teams can develop new products faster and more efficiently than can functions working independently in a traditional structure. Horizontal designs also encourage knowledge sharing. However, because lines of authority are less clear, managers must be able to share responsibility for the organization's overall performance, build commitment to a shared vision, and influence others even when they lack direct authority. This type of structure is a good fit when specialization is less important than the ability to respond to varied or changing customer needs. It requires employees who can rise to the challenges of empowerment. Table 15.2 lists fewer opportunities for functional specialization as a con, or disadvantage.

________ are powerful forms of social control that influence group and member behavior.

group froles and norsm

0 out of 10 points Claire takes on the role of ______; she tends to mediate conflict through humor.

harmonizer

Pat and Chris are both computer programmers, but Pat earns more money than Chris. However, Chris perceives an equitable relationship because Pat: works less than Chris. takes longer breaks. has more education and experience. works as hard as Chris. needs the money more than Chris.

has more education and experience

The primary intent of Bagley's decision tree model is to

help in making more ethical decisions. Explanation A decision tree can help managers make more ethical decisions.

Joaquin is usually the first one to volunteer at the weekly team meetings to take on additional tasks like taking notes for the meeting. Joaquin would probably be categorized as a(n): -partisan. -diplomat. -helper. -rebel. -independent.

helper

The type of follower who is most compliant with leadership is the: -counselor. -diplomat. -rebel. -helper. -partisan.

helper

A(n) _________ culture develops reliable internal processes, extensive measurement, and a variety of control mechanisms.

hierarchy Explanation A hierarchy culture has an internal focus and values stability and control. This orientation leads to the development of reliable internal processes, extensive measurement, and the implementation of a variety of control mechanisms.

GW, Inc., makes high-quality whizzers according to specifications provided by customers which requires controlled processes and precise measurements. GW needs to maintain high efficiency. This industry is mature and few innovations occur. For GW, Inc. stability is more important than flexibility. Which of the organizational cultures is best suited for GW, Inc.?

hierarchy culture

Jim is the daytime supervisor for an automobile assembly line. He supervises 45 people who perform relatively routine jobs that require minimal training. The shop is unionized, so Jim has little latitude when it comes to dismissing poor performers. However, he does have the authority to transfer employees and has been known to reassign poor performers to less desirable jobs on the assembly line. Jim has a reputation for only transferring employees with just cause, and generally is viewed as a fair supervisor by his employees. Although he would like greater autonomy to determine salaries, Jim generally divides bonuses and raises equally among his employees. To do otherwise would likely create conflict with union members and representatives. According to Fiedler's model, what would the task structure of Jim's employees be considered?

high Explanation Task structure is concerned with the amount of structure contained within tasks performed by the work group. For example, a managerial job contains less structure than that of a bank teller. Because structured tasks have guidelines for how the job should be completed, the leader has more control and influence over employees performing such tasks. This dimension is the second most important component of situational control.

Marianne manages a landscaping company. She supervises 19 people who perform relatively routine jobs that require minimal training. Because she is only the manager, not the owner, Marianne has little latitude when it comes to dismissing poor performers. However, she can move employees to positions where they don't receive tips. Marianne has a reputation for being fair. Although she would like greater autonomy to determine salaries, Marianne generally divides bonuses among her employees. Otherwise, the owner might think she is playing favorites. According to Fiedler's contingency model, how would you characterize the level of situational control in these circumstances?

high Explanation This is a high-control situation because leader-member relations are good, task structure is high, and position power is weak. See Figure 13.3.

John, a mid-level manager, notices that all his subordinates are filling out their monthly reports incorrectly. He concludes that the report forms are too complicated and need to be revised. On which type of attributional information is John's decision based?

high consensus Explanation Consensus compares an individual's behavior with that of his or her peers. There is high consensus when one acts like the rest of the group and low consensus when one acts differently.

David's boss has observed that David took about 12 hours to perform tasks A, B, and C. However, he took 24 hours to complete task D. David's boss concluded that task D must have been tougher than the other tasks. Which of the following attributional information led David's boss to this conclusion?

high distinctiveness Explanation Distinctiveness compares a person's behavior on one task with his or her behavior on other tasks. High distinctiveness means the individual has performed the task in question in a significantly different manner than he or she has performed other tasks.

Marisa is the office manager in a small company. Her employees find that they can often come late to work and leave early, and Marisa will cover their work for them. Marisa is likely to score

high on agreeableness. Explanation According to Table 3.2, someone who is trusting, good-natured, cooperative, and softhearted would score high on agreeableness.

As demonstrated by research, ___________ is positively related to organizational citizenship behavior.

higher performance ratings and lower organizational costs Explanation OCBs are significantly related to both individual level consequences (e.g., performance appraisal ratings, intentions to quit, absenteeism, and turnover) and organizational-level outcomes (e.g., productivity, efficiency, lower costs, customer satisfaction, and unit-level satisfaction and turnover).

People with high levels of positive psychological capital tend to display

hope, efficacy, resilience, and optimism. Explanation Professor Fred Luthans says that those with high levels of positive psychological capital have high levels of hope, efficacy resilience, and optimism (HERO).

Veronica just accepted a job at QuizBiz. She left a large company that offered big bonuses for salespersons who exceeded their quotas. At QuizBiz, the individual bonuses are smaller, but there are team rewards as well. This is known as a _____ reward system.

hybrid Explanation Organizations that foster the greatest collaboration and assemble the most effective teams typically use hybrid reward systems that recognize both individual and team performance.

Ted was dissatisfied with his job. He said that the company policy, supervision, and working conditions were responsible for his dissatisfaction. According to Frederick Herzberg's theory, ________ are the extrinsic factors that create job dissatisfaction.

hygiene factors Explanation Hygiene factors—including company policy and administration, technical supervision, salary, interpersonal relations with one's supervisor, and working conditions—cause a person to move from a state of no satisfaction to dissatisfaction.

Pietro is assigned to a team in his environmental science class. He is not comfortable with Sam, the team leader, but is not completely sure why. Which of the following behaviors would not build trust?

if Sam keeps information to himself Explanation To build trust, a team leader should keep team members informed by explaining policies and decisions and providing accurate feedback.

Leaning backward or gesturing with palms facing up communicate _______, a term used to represent availability for communication.

immediacy Explanation Open body positions, such as leaning backward or gesturing with palms facing up, communicate immediacy, a term used to represent openness, warmth, closeness, and availability for communication.

Carrie goes out to lunch and orders a diet soda. If we consider _________, she is likely to prefer a salad for lunch.

implicit cognition Explanation Implicit cognition represents any thoughts or beliefs that are automatically activated from memory without our conscious awareness.

Deep-level characteristics of diversity

include attitudes, opinions, and values. Explanation Deep-level characteristics are those that take time to emerge in interactions, such as attitudes, opinions, and values. These characteristics are definitely under our control.

Shoney's hired African Americans to fill positions of dining-room supervisors and vice presidents, added more franchises owned by African Americans, and purchased more goods and services from minority-owned companies. This represents the _____ option that organization can use to address diversity issues.

include/exclude Explanation Include/exclude is an outgrowth of affirmative action programs. Its primary goal is to either increase or decrease the number of diverse people at all levels of the organization.

Green Care Landscaping Company minimizes attention to a price increase by selling lawn care treatments to a subsidiary, Keep It Up, Inc. Keep It Up then increases the price of the specific treatments. The manager of Keep It Up wants to notify customers that they are a subsidiary of Green Care Landscaping and that the increase has come from the parent company. This is an example of a remedy for

indirect blindness. Explanation Table 1.2 states that the remedy for indirect blindness when handing off or outsourcing work is to ask whether the assignment might invite unethical behavior and take ownership of these implications.

Alexa quit her job because she felt it was boring. This would be diagnosed as a(n) ______ level problem.

individual Explanation Some people quit because their job just doesn't fulfill what they value, such as challenging and stimulating work (an individual-level input).

Francine manages a team of 20. She pays special attention to the needs of followers and searches for ways to help people develop and grow constantly. She is using _____ leader behavior. Answers: transactional idealized influence individualized consideration intellectual stimulation inspirational motivation

individualized consideration Response Feedback: Individualized consideration entails behaviors associated with providing support, encouragement, empowerment, and coaching to employees. These behaviors necessitate that leaders pay special attention to the needs of their followers and search for ways to help people develop and grow.

John received a poor performance evaluation from his boss. On the weekend, he talks with his neighbor Faisal about his situation. Faisal asks him questions about his work and the evaluation, and as a result John develops a plan of action for improving his productivity. Faisal has provided

informational support. Explanation Informational support is providing help in defining, understanding, and coping with problems.

Carl needs a day off from his work. But he is apprehensive that David, his manager, will not grant the leave. Carl knows that David wants to shed a few pounds and has been hitting the gym for the past month. Before asking for the leave, Carl compliments David on his build and physique. It comes as no surprise to Carl that David grants him the leave! What kind of an influence tactic did Carl use?

ingratiation Explanation Ingratiation is getting someone in a good mood prior to making a request; being friendly, helpful, and using praise, flattery, or humor.

All of the following are leader behaviors exhibited by transformational leaders EXCEPT: Answers: initiating structure. inspirational motivation. idealized influence. intellectual stimulation. individualized consideration.

initiating structure. Response Feedback: The Ohio State researchers concluded there were only two independent dimensions of leader behavior: consideration and initiating structure. Initiating structure is leader behavior that organizes and defines what group members should be doing to maximize output. Refer: Figure 14-3

Janice has transferred to the contract management group at the HMO that employs her. She has been through two months of training and is now working closely with Nancy to better understand the values and expectations of the group. Janice is in the ____ phase of mentoring.

initiation Explanation The four phases of mentoring are initiation, cultivation, separation, and redefinition. The initiation phase lasts 6 to 12 months and starts during the encounter phase of socialization. Mentors socialize new employees about the values, norms, and expectations associated with the organization's culture during this phase.

When Mr. Brown, the new CEO of XYZ Department Stores, took over, his first action was to get rid of the organization's existing marketing strategy, which was heavily based on couponing. He replaced it with a strategy of low prices every day, a policy with which some of its competitors were having much success. This was a(n) ________ change.

innovative Explanation Figure 16.4 identifies innovative change as introducing a practice new to the organization. Innovative change falls midway on the continuum of complexity, cost, and uncertainty.

Frank Menard runs a consulting firm, TalentTrack. He believes in engaging employees by talking about his vision for the company, appealing to their emotions, and is highly enthusiastic about the company at all times. He is using a(n) _____ leader behavior. Answers: idealized influence achievement-oriented inspirational motivation intellectual stimulation individualized consideration

inspirational motivation Response Feedback: Inspirational motivation involves establishing an attractive vision of the future, the use of emotional arguments, and exhibition of optimism and enthusiasm. Refer: Figure 14-3

Jen runs a medium-sized credit processing company. Everyone gets along and does their job, but Jen feels the employees are not really invested in the company. Jen volunteers as an ESOL teacher twice a week, helping immigrants learn English. She is inspired by doing this volunteer work, and she decides to offer all the employees an opportunity to volunteer "on the clock" at whatever they choose. Each employee will meet with Human Resources, create a plan and submit a schedule. Jen is ______ by empowering employees and leading for meaningfulness.

inspiring subordinates Explanation Managers lead for meaningfulness by inspiring their employees and modeling desired behaviors.

Lee and Victor are good friends. Lee's laptop computer has become very slow and is not working correctly, and he goes to Victor and asks for help. Victor downloads some antimalware software onto Lee's computer, and its performance improves. Victor has provided

instrumental support. Explanation Instrumental support is providing financial aid, material resources, or needed services.

How an individual perceives the movement from performance to outcome is called: a value. a valence. an expectancy. an expectation. instrumentality.

instrumentality

A(n) ________ negotiation describes a situation where an agreement can be found that is better for both parties.

integrative Explanation An integrative negotiation describes a situation where an agreement can be found that is better for both parties than what they would have reached through distributive negotiation.

Connie knows that Benton Inc. is not growing. In fact, it's lost a lot of business to new competitors. She asks her managers 1. What is Benton Inc. doing wrong? 2. What can the company do to turn things a round? 3. Should the goals or mission of the company change? She wants them to question the status quo and seek innovative and creative solutions to organizational problems. What kind of behavior is Connie exhibiting?

intellectual stimulation Explanation Intellectual stimulation involves behaviors that encourage employees to question the status quo and to seek innovation and creative solutions to organizational problems.

Proactivity has been shown to be related to all of the following except

intelligence

Johan does well at work because he is able to think constructively, reason, and problem solve. Johan is exhibiting

intelligence. Explanation Intelligence represents an individual's capacity for constructive thinking, reasoning, and problem solving.

______ refers to the amount of effort that we invest in an activity.

intensity

"Different people may perceive similar situations in different ways and similar people may perceive different situations in the same way." This represents the _____ perspective.

interactional Explanation The interactional perspective states that behavior is a function of interdependent person and environmental factors.

Which of the following is used by an organization to identify its strengths and weaknesses?

internal analysis

People with ________ see themselves as masters of their own fate.

internal locus of control

Deliver-IT dispatches over 5,400 trucks from 165 service centers on a daily basis. The company picks up deliveries, consolidates them, and then delivers them to customers. This company: delivers 81 percent of packages on time. has an impressive driver safety record. uses driver mentors and trainers. links pay to safety measure. This reflects the ___________ perspective.

internal processes Explanation The internal processes dimension of effectiveness focuses on "what the organization must excel at" to effectively meet its financial objectives and customers' expectations.

Keyshawn is a player on a professional football team. Because of this, his plays every week are scrutinized by fans and media, as well as his own coaches. Sometimes, their comments are very negative and even personal. Keyshawn will handle this better if he has a high level of _________ intelligence.

intrapersonal Explanation According to Table 3.1, intrapersonal intelligence is the potential to understand and regulate oneself.

Rachel bought a table at a thrift store for $20. She had a feeling it might be valuable. When cleaned up and assessed, the table turned out to be an antique from the 1700s, worth $3,250. What was Rachel using to make her decision?

intuition Explanation Intuition represents judgments, insights, or decisions that come to mind on their own, without explicit awareness of the evoking cues and of course without explicit evaluation of the validity of these cues.

Self-Efficacy

is a belief that one can perform a specific task successfully. Research shows that the belief that we can do something is a good predictor of whether we can actually do it. Self-efficacy is different from other personality traits in that it is job specific. You may have high self-efficacy in being successful academically, but low self-efficacy in relation to your ability to fix your car. . By showing that you believe they can be successful and effectively playing the role of a cheerleader, you may be able to increase self-efficacy. Giving people opportunities to test their skills so that they can see what they are capable of doing (or empowering them) is also a good way of increasing self-efficacy.

The distinction between flexible and fixed individual differences

is that managers have little or no impact on fixed differences. Explanation Managers have little or no impact on fixed individual differences.

Self-Esteem

is the degree to which a person has overall positive feelings about oneself. People with high self-esteem view themselves in a positive light, are confident, and respect themselves. On the other hand, people with low self-esteem experience high levels of self-doubt and question their self-worth. High self-esteem is related to higher levels of satisfaction with one's job and higher levels of performance on the job as well as higher levels of creativity at work. [58] People with low self-esteem are attracted to situations in which they will be relatively invisible, such as large companies.

Agreeableness

is the degree to which a person is affable, tolerant, sensitive, trusting, kind, and warm. In other words, people who are high in agreeableness are likeable people and get along with others.

Openness

is the degree to which a person is curious, original, intellectual, creative, and open to new ideas People high in openness seem to thrive in situations that require being flexible and learning new things.

Extraversion

is the degree to which a person is outgoing, talkative, sociable, and enjoys socializing Interacting with others and being social energizes extraverts, whereas similar levels of stimulation and interactions may be viewed as draining to someone who is an introvert.

By ___, a person can develop self-awareness

knowing his or her own strengths and limits

By ___________, a person can develop self-awareness.

knowing his or her own strengths and limits Explanation According to Table 3.6, a person can develop his or her self-awareness by reading one's own emotions and recognizing their impact; using "gut sense" to guide decisions; knowing one's strengths and limits; and having a sound sense of one's self-worth and capabilities.

The leadership theory that is based on the assumption that leaders develop unique one-to-one relationships with each of the people reporting to them is: -transformational. -transactional. -leader-member exchange. -house's path-goal. -servant leadership.

leader member exchange

Which of the following mechanisms for changing organizational culture addresses only the level of basic underlying assumptions of culture?

leader reactions to critical incidents and organizational crises Explanation Table 14.1 indicates that the organizational activities, processes, or outcomes mechanism addresses only the basic assumption level of organizational culture.

The leadership theory that is based on the assumption that leaders develop unique one-to-one relationships with each of the people reporting to them is

leader-member exchange. Explanation The leader-member exchange (LMX) theory is based on the assumption that leaders develop unique one-to-one relationships with each of the people reporting to them.

In which of the following situations would a leader's situational control be considered low?

leader-member relations poor; task structure low; position power weak Explanation Situation VIII is a low control example where the leader-member relations are poor, the task structure is low, and the position power is weak.

CRF schedules are particularly useful when an employee is

learning a new skill Explanation Continuous reinforcement schedules are especially useful when employees learn a new task or skill.

The ________ perspective focuses on providing employees with the capabilities, resources, and work environment they need to achieve their goals.

learning and growth Explanation The learning and growth perspective focuses on providing employees with the capabilities, resources, and work environment they need to achieve customer, internal business processes, and financial goals. It's the foundation of all other goals in a scorecard.

Dr. Slotsky is a well-liked professor. Last term, no one received less than a B in any of his courses. Dr. Slotsky may be suffering from _________ perceptual error.

leniency Explanation According to Table 6.4, the leniency perceptual error is to consistently evaluate other people or objects in an extremely positive fashion. By evaluating every student in every class no less than a B, he may be demonstrating the leniency perceptual error.

Katherine does especially well in her language courses, but struggles in chemistry. She has ______ intelligence.

linguistic Explanation According to Table 3.1, linguistic intelligence is the potential to learn and use spoken and written languages.

Vivek and Warren are working on an assignment for their marketing class. Warren finishes his part of the assignment; Vivek doesn't. Vivek apologizes because he took an extra shift at work and didn't give the assignment the priority it needed. He promises Warren he will stay home tonight and finish his work. What personality characteristic does this show?

locus of control Explanation Locus of control is the relatively stable personality characteristic that describes how much personal responsibility a person takes for his or her behavior and its consequences.

Which of the following is not an outcome likely to result from positive emotions?

loss of focus Explanation Research provides compelling evidence that positive emotions lead to or foster many desirable behaviors and outcomes such as stronger social relationships, prosocial behaviors, liking of yourself and others, stronger bodies and immune systems, and original thinking.

The business department at WhatsUp University is planning to split into two departments, one for accounting and finance and another for marketing and management. As a result, the present department chair, Professor Bland, will lose his full-time release from teaching and also share his position on the dean's council with whoever the other department chair will be. He is against the change, stating that it will cause many problems. How would you describe the most likely cause of Dr. Bland's resistance?

loss of status Explanation Loss of status and/or job security refers to the fact that administrative and technological changes that threaten to alter power bases or eliminate jobs generally trigger strong resistance.

After 6 weeks in a culinary arts program, Diana continues to struggle to master even the most basic skills. She blames her mother, who could not even boil an egg, for her own lack of talent for cooking. Diana is likely to be experiencing all of the following EXCEPT:

low emotional intelligence

Samantha, a member of a team you are assigned to for a management class, has often been late getting her work to the team, and when it arrives it is often of poor quality. Samantha is likely to rank ________ dimension.

low on the conscientiousness Explanation According to Table 3.2, someone scoring low on conscientiousness would not be very dependable, responsible, achievement oriented, or persistent.

Halfway through the semester, Samuel quit doing assignments for and going to his advanced accounting class. He attributes this to his poor performance on the first two tests, as well as his lack of talent for accounting; he feels he will do much better as a management major. Samuel is likely to be experiencing

low self-efficacy for accounting. Explanation Self-efficacy is a person's belief about his or her chances of successfully accomplishing a specific task.

Low job satisfaction and low employee engagement can lead to

low task performance. Explanation Low job satisfaction and low employee engagement imply lower task performance and higher employee turnover.

Lily graduates from high school and is ready to go to college. However, she also loves sports. She wants to join a basketball academy and become a professional basketball player. Lily finds her uncertainty about her future very stressful and is psychologically uncomfortable about it. This implies that Lily has a(n)

low tolerance for ambiguity. Explanation A person's tolerance for ambiguity indicates the extent to which a person has a high need for structure or control in his or her life. Some people desire a lot of structure in their lives at all times (a low tolerance for ambiguity) and find ambiguous situations very stressful and psychologically uncomfortable.

A company whose employees have high engagement is likely to have ________ compared to a company with low employee engagement.

lower turnover

Decision making

making choices among alternative courses of action, including inaction

In the ________ culture, employees are encouraged to set difficult goals and strive to achieve them. Employee performance is closely monitored and often directly rewarded or punished.

market

Neil is attending a job fair. He knows that there is a lot of competition for these positions and wants to stand out. Neil practices his greeting and "elevator" pitch in front of a mirror. He brings copies of his résumé and business cards. On each card, he notes his blog site address. On the blog, he has comments on several recent articles about new technologies in this field. Neil is maximizing

media richness. Explanation Media richness involves the capacity of a given communication medium to convey information and promote understanding.

A neutral and trained third party can guide others to find innovative solutions to conflict. To ensure neutrality, most organizations hired ADR qualified outsiders. In this case, a(n) _________ is one who does not render a decision. Rather, it is up to the parties to the conflict to reach a mutually acceptable decision.

mediator Explanation A neutral and trained third party guides the others to find innovative solutions to the conflict. To ensure neutrality, most organizations hire ADR qualified outsiders. Unlike an arbitrator, a mediator does not render a decision. It is up to the disputants to reach a mutually acceptable decision. The Equal Employment Opportunity Commission (EEOC) implemented mediation in the 1990s and cut the average time to resolution by 80 percent!

Big Brothers Big Sisters is a volunteer _________ network that pairs adults with children. A survey of adults who participated in the program when they were children revealed that the majority obtained values and principles that influenced them throughout their lives.

mentoring Explanation Big Brothers Big Sisters is the largest volunteer mentoring network in the United States. The organization has paired adults with children for over 100 years. A survey of adults who participated in the program as children revealed that 83 percent obtained values and principles that influenced them throughout their lives.

Troy starts a job right out of college. He feels lucky to have found a position in the financial industry. Troy knows that he needs help to form and maintain the lasting developmental relationships that will further his career. Troy should first establish a(n)

mentoring plan. Explanation Develop a mentoring plan. Experts suggest your plan should include the following steps: -Base your mentoring goals on what you want to learn, and then prioritize them. -Identify people who are skilled or experienced in areas where you want to improve. Don't overlook your peers; they are a good source of functional, technical, and organizational knowledge. -Decide how best to build a relationship with your targeted individuals. -Figure out how you can provide value to your mentor. Because mentoring is a two-way street, others are more likely to help you if they see some value in assisting you to pursue your career goals. -Recognize when it is time to move on. Mentors are not forever. If you believe your mentor is ineffective or harming more than helping, find a new mentor. It's easy to become stuck with one mentor. Expanding your horizons will not only benefit you, but it can help the mentor develop his or her mentoring skills as well.

Carlos works at Tyner Construction Company as an assistant project manager. His manager assured him that if his projects came in on time and under budget, he would be promoted within a year. At nine months, Carlo is promoted to project manager. This is an example of

met expectiations

Reyna is meeting with her supervisor to discuss a project. Someone knocks on the door and walks in, saying, "I have a quick question." The supervisor keeps looking at Reyna and puts her hand up to stop the person who interrupted them. This behavior reflects:

mindfulness

Which of the following is an open organizational design?

modular Explanation Organizations defined by an open approach tend to have hollow, modular, or virtual structures.

In empowering employees, to lead for progress, managers should

monitor and reward employees.. Explanation Managers lead for progress by monitoring and rewarding others.

More on Artifacts

most visible level of culture and include such things as acronyms, manner of dress, awards, myths and stories told about the organization, published lists of values, observable rituals and ceremonies, special parking spaces, decorations, and so on. Easier to change than the less visible aspects of organizational culture.

Job enrichment is based on the ___ theory of job satisfaction.

motivator-hygiene

Barbara is managing two franchises and is a single mother of three children. Which stress-reduction technique would Barbara find inexpensive and easy to use?

muscle relaxation Explanation Muscle relaxation is a stress-reduction technique that is inexpensive and easy to use. See Table 16.4.

Claire is working 24/7 to meet a deadline. She often uses _________, a method for stress reduction that involves slow, deep breathing.

muscle relaxation Explanation Muscle relaxation is a stress-reduction technique that uses slow, deep breathing and systematic muscle tension relaxation. See Table 16.4.

Which of the following is not a characteristic of an out-group exchange? -Mutual liking -Economic exchange -Negotiating the relationship between performance and rewards -Fewer common interests -Formality

mutual liking

Wayne is on the production line for Medical Services Inc. He has been there for five years and has won Employee of the Month more than anyone else. Zander, his coworker, who has only been at Medical Services three years and hasn't even been nominated for Employee of the Month, just received a promotion. Wayne perceives a

negative inequity

Amy receives a note from Carrie, asking her to pick up some materials for a project from Staples. Amy does the shopping, but misses a few items because she can't decipher Carrie's writing. This is an example of

noise. Explanation Noise represents anything that interferes with the transmission and understanding of the message.

Jennie and Lacey are having an argument. As Lacey speaks, Jennie crosses her arms and looks at the ground. Lacey gets the message even thought Jennie isn't saying anything. Jennie is using ________ in the conversation with Lacey.

nonverbal communication Explanation Nonverbal communication is any message sent or received independent of the written or spoken word.

In which stage of the group development process do group members resolve their power struggles so that something can be accomplished?

norming Explanation Groups that make it through storming and into the norming stage generally do so because a respected member, other than the leader, challenges the group to resolve its power struggles so something can be accomplished.

Carl's team is evaluation his effectiveness as a leader. Which of the following behaviors does not reflect and effective leader?

not letting other group members make any decisions

The sales department of a mutual fund firm sets goals for the next fiscal year to sell a certain number of mutual funds each month. The firm only sets targets, but provides no other details. These goals are _____ goals.

objective Explanation Table 6.3 lists examples of objective goals as sales quotes, production rates, error rates. By setting a goal to sell a certain number of mutual funds, the firm is setting objective goals.

An example of a(n) _____ goal is when Melissa's manager sets a goal for her to produce 16 widgets per hour.

objective Explanation Table 6.3 uses production rates to illustrates an objective goal.

Mottollo Pasta uses their website to explain that five traits make their culture distinctive—creativity, sustainability, family, community, and diversity. What level of organizational culture does this represent?

observable artifacts Explanation Observable artifacts represent the more visible level of culture. This includes acronyms, manner of dress, awards, myths and stories told about the organization, published lists of values, observable rituals and ceremonies, special parking spaces, decorations, and so on. We can look to a company's website as an observable artifact to better understand their culture.

The levels of organizational culture are

observable artifacts, espoused values, and basic underlying assumptions. Explanation Organizational culture operates on three levels: (1) observable artifacts, (2) espoused values, and (3) basic underlying assumptions.

Because the auditors of XYX Company accepted a client firm's questionable financial statements when the infractions have occurred over time, James believes that they are

on a slippery slope. Explanation Table 1.2 states the slippery slope occurs when we are less able to see others' unethical behavior when it develops gradually (e.g., auditors may be more likely to accept a client firm's questionable financial statements if the infractions have accrued over time).

A(n) ________ helps new recruits integrate, assimilate, and transition to new jobs by making them familiar with corporate policies, procedures, culture, and politics and by clarifying work-role expectations and responsibilities.

onboarding program Explanation Onboarding programs help employees to integrate, assimilate, and transition to new jobs by making them familiar with corporate policies, procedures, culture, and politics and by clarifying work-role expectations and responsibilities.

When a business doesn't have control of all the environmental forces, it is called __________ design. For example, Acme Medical relies on predictions and contingencies to cope with unexpected input. Last year, an influenza epidemic affected suppliers, personnel, and even customers, causing lost production and lost profit.

open Explanation Organizations defined by an open approach tend to have hollow, modular, or virtual structures. Each of these structures relies on leveraging technology and structural flexibility to maximize potential value through outsourcing and external collaboration.

Nola graduated from her MBA program last year and took on a high-paying position as a consultant-in-training at McCrabree Consulting. She often works 60-80 hours per week, and mostly her assignments are in cities across the country; she rarely stays home for more than two days at a time. This is all part of the way consulting operates, as she was told by the people who ran the firm's orientation session. In fact, at that session, the facilitator brought in consultants who had worked at the firm for several years who stressed that the firm believes in pushing employees to the limit. Nola is definitely feeling the stress! What would you characterize as the level of her stressors?

organizational Explanation Figure 16.9 identifies potential stressors at the organizational level as culture, structure, technology, introduction of change in work conditions.

Level 3: Basic Underlying Assumptions

organizational values that have become so taken for granted over time that they become assumptions that guide organizational behavior.

The ______ keeps the group headed toward its stated goals.

orienter

Our self-efficacy is an example of a factor that influences: our valences. our outcomes. our perceptions of instrumentality. our expectancies. our values.

our expectancies

"How much do I value the rewards I receive?" This question addresses

our valance. Explanation Valence describes the positive or negative value people place on outcomes.

Strategic plan

outlines an organization's long-term goals and the actions necessary to achieve those goals.

Josephine is about to graduate from college and is excited about joining her two roommates to start a small coffee shop near campus. She has never started a business before, but she is sure that her business degree has taught her everything she needs to know. The three of them have enough money for their initial costs and figure that they can cover their expenses with the cash flow from the business. Which heuristic is probably involved?

overconfidence Explanation The overconfidence bias is our tendency to be overconfident about estimates or forecasts. This bias grows in strength when people are asked moderate to extremely difficult questions rather than easy ones. Entrepreneurs especially fall prey to this bias when deciding to start and sustain new ventures.

In showing class-action ________ lawsuits, the plaintiffs show some statistical disparities between the composition of some group within the company compared to some other relevant group.

pattern and practice

Alana looks at the clouds and sees flowers. Her brother, Sammy, looks at the same clouds and sees jellyfish. Alana and Sammy have different

perceptions. Explanation Perception is a cognitive process that enables us to interpret and understand our surroundings.

Barry set himself a goal of bowling at least one game above 250 (out of 300 possible) before the end of the season. This is a ________ goal.

performance Explanation A performance goal targets a specific end result. Barry's goal of bowling at least one game above 250 before the end of the season is an example of a performance goal.

The two basic types of goals are

performance and learning

Vanessa receives her evaluation. Her supervisor gives her high marks in __________, noting that Vanessa is especially strong in self-awareness and self management.

personal competence Explanation Self-awareness and self-management constitute personal competence.

Victor seems to want to control others and often manipulates people for his own gratification. He exhibits the need for

personal power. Explanation The negative face of power is called the need for personal power. People with this need want to control others, and they often manipulate people for their own gratification.

Otto is convinced that social media offers many benefits for employees and wants to institute a plan at Compass Inc. Which of the following is not a potential benefit for employees?

personal stability Explanation Evidence is mounting that social media confers a host of benefits for employees, such as -Increased job satisfaction and better work-life balance. -Performance and retention. -More creativity and collaboration.

____________ is the combination of stable physical, behavioral, and mental characteristics that gives individuals their unique identity.

personality

As the mother of two small children, Jen struggles to make ends meet on her minimum-wage job. After paying rent and child care expenses, there is sometimes not enough money left at the end of the month to pay the heating bill. More than once the family has gone without warmth on cold nights, and she and kids have gone to bed hungry. Which of the five basic needs is Jen struggling to meet? Esteem Physiological Love Self-actualization Safety

physiological

As the mother of two small children, Jen struggles to make ends meet on her minimum-wage job. After paying rent and child care expenses, there is sometimes not enough money left at the end of the month to pay the heating bill. More than once the family has gone without warmth on cold nights, and she and the kids have gone to bed hungry. Which of the five basic needs is Jen struggling to meet?

physiological Explanation According to Figure 5.2, physiological need is the most basic need. This entails having enough food, air, and water to survive.

Bettina will experience _______ if her outcome to input ratio is greater than that of a relevant comparison person.

positive inequity Explanation Bettina will experience positive inequity when her outcome to input ratio is greater than that of a relevant comparison person.

What type of power tends to produce commitment?

positive legitimate, expert, and referent Explanation Positive legitimate power, expert power, and referent power tend to produce commitment.

After a series of train incidents, NS&Q offered employee bonuses based on safety for the first quarter of this year. This is an example of

positive reinforcement

The _______ effect is the attraction of all living systems toward that which is life-giving and away from that which is life-depleting.

positivity Explanation The positivity effect is the attraction of all living systems toward positive energy and away from negative energy, or toward that which is life giving and away from that which is life depleting.

Organizational ______ is the host of procedures, policies, routines, and rules that organizations use to get things done.

practices Explanation Organizational practices refer to a host of procedures, policies, practices, routines, and rules that organizations use to get things done.

The idea that creativity starts from a base of knowledge is represented in the _______ stage of the creative performance process.

preparation/information gathering Explanation The preparation stage reflects the notion that creativity starts from a base of knowledge.

Of the following, ________ is the least frequently used influence tactic.

pressure Explanation The following tactics are listed in rank order, beginning with most frequently used: 1.rational persuasion 2.inspirational appeals 3.consultation 4.ingratiation 5.personal appeals 6.exchange 7.coalition tactics 8.pressure 9.legitimating tactics

Core self-evaluations represent a broad personality trait comprised of all of the following except

proactivity

Bettina wants to make the Dean's List this semester. She is doing well in all of her courses except Accounting. This presents a(n) _____ for Bettina.

problem Explanation A problem is a difference or gap between an actual and a desired situation.

The perceived fairness of the processes used to make allocation decisions is called ___ justice

procedural

The perceived fairness of the processes used to make allocation decisions is called: institutional justice. procedural justice. interpersonal justice. distributive justice. interactional justice.

procedural justice

As part of a work team in his office, it is Larry's job to make photocopies of relevant materials and hand them out to the team members during meetings. Which of the following task roles is Larry performing in his work team?

procedural technician Explanation Larry is performing the role of a procedural technician. The procedural technician performs routine duties (e.g., handing out materials or rearranging seats).

Sunshine Crackers introduces Wheat Flats,a cracker with more fiber, fewer calories, and great flavor. Wheat Flats is a great success. This is an example of __________ innovation.

product Explanation A product innovation is a change in the appearance or functionalilty/performance of a product or a service or the creation of a new one.

When a receiver expresses a reaction to the sender's message, he or she is

providing feedback. Explanation Feedback is when the receiver expresses a reaction to the sender's message.

Joe creates advertising slogans for an advertising firm. He works with beliefs, perceptions, and informal obligations about what he is entitled to receive in return for what he provides to the organization. A(n) ________ represents Joe's beliefs.

psychological

Tammy participates actively in the weekly team meetings, but when it comes to doing her work, she tends to do it her own way, often disregarding company policies and her boss's wishes. Tammy is probably a(n): -diplomat. -rebel. -helper. -independent. -partisan.

rebel

Tammy participates actively in the weekly team meetings, but when it comes to doing her work, she tends to do it her own way, often disregarding company policies and her boss's wishes. Tammy is probably a(n)

rebel. Explanation Rebels show divergence from the leader and are least compliant.

The recommended way to look at resistance to change is to view it as caused by

recipient characteristics, change agent characteristics, and the relationship between the change agent and the recipient. Explanation Figure 16.8 illustrates that resistance is a dynamic interaction among three sources: recipient characteristics, change agent characteristics, and the change agent-recipient relationship.

Which of the following is not an example of a hygiene factor? Recognition Company policies Supervision Working conditions Salary

recognition

Which of the following is not an example of a hygiene factor?

recognition Explanation Hygiene factors—including company policy and administration, technical supervision, salary, interpersonal relations with one's supervisor, and working conditions—cause a person to move from a state of no dissatisfaction to dissatisfaction.

Terminal values

refer to end states people desire in life, such as leading a prosperous life and a world at peace

Values

refer to stable life goals that people have, reflecting what is most important to them Values are established throughout one's life as a result of the accumulating life experiences and tend to be relatively stable. Value attainment is one reason why people stay in a company, and when an organization does not help them attain their values, they are more likely to decide to leave if they are dissatisfied with the job itself. [3] The values a person holds will affect his or her employment.

Therese is a charismatic person who is often able to get other employees to work late on special projects by being very friendly and fun to be around. She is exercising her ______ power.

referent Explanation Referent power is when one's personal characteristics and social relationships become the reason for compliance. Charisma is commonly associated with referent power.

Proactive Personality

refers to a person's inclination to fix what is perceived as wrong, change the status quo, and use initiative to solve problems. Instead of waiting to be told what to do, proactive people take action to initiate meaningful change and remove the obstacles they face along the way. In general, having a proactive personality has a number of advantages for these people. For example, they tend to be more successful in their job searches. [52] Proactive employees are also more successful over the course of their careers, because they use initiative and acquire greater understanding of the politics within the organization. Proactive people are eager to learn and engage in many developmental activities to improve their skills

Neuroticism .

refers to the degree to which a person is anxious, irritable, temperamental, and moody. people very high in neuroticism experience a number of problems at work. Most of their problems are due to difficulties in handling stress. Neurotic people have a tendency to experience a lot of anger when they are confronted with daily hassles of their work, including time pressure or red tape, and they are more inclined to respond to their daily stress by drinking after hours.

Conscientiousness

refers to the degree to which a person is organized, systematic, punctual, achievement-oriented, and dependable Conscientiousness is the one personality trait that uniformly predicts how high a person's performance will be across a variety of occupations and jobs.

Self-Monitoring

refers to the extent to which a person is capable of altering his or her actions and appearance in social situations. In other words, people who are self-monitors are social chameleons who understand what the situation demands and act accordingly, while low self-monitors tend to act the way they feel

Anchoring

refers to the tendency for individuals to rely to heavily on a single piece of information.

Leader-member exchange theory focuses on the leader's

relationships with followers. Explanation The leader-member exchange (LMX) theory is based on the assumption that leaders develop unique one-to-one relationships with each of the people reporting to them.

It's snowing and Alice runs outside to the mailbox without her coat. She immediately shivers and gets goose bumps. This is an example of _____ behavior.

respondent Explanation Unlearned reflexes or stimulus-response connections are respondent behavior.

Anna blinks her eyes whenever she looks up at the sun. This is an example of

respondent behavior. Explanation Unlearned reflexes are respondent behavior.

Pete and Dana are working on a project together. They disagree on how to present the key concept of equality. Dana complains that Pete is dismissing her "feminist" position without consideration. Pete claims that Dana considers him a chauvinist. Wyatt, their supervisor, knows that both Pete and Dana are committed employees who support equality. He sits both of them down to work through the issues. This reflects _____ justice.

restorative Explanation Restorative justice reflects a shared belief in the importance of resolving conflict multilaterally through the inclusion of victims, offenders, and all other stakeholders.

John leads a team of 10 salespersons. He informs the team that the first member to achieve the year's target will be sent on an all-expenses-paid holiday to the Grand Canyon National Park. John is demonstrating __________ power.

reward Explanation Individuals or organizations have reward power if they can obtain compliance by promising or granting rewards.

Professor Smith promises Professor Davis that he will buy him dinner if Professor Davis covers class for him one day while Professor Smith is presenting a paper at a conference. Professor Davis agrees, although he is merely going to hand out an assignment to students and be available for questions if they have any. Professor Smith is using his ________ power and Professor Davis is ________.

reward; complying Explanation See Figure 12.4. Reward power tends to produce compliance.

Jerry is in charge of the telesales department in Recreation Paradise. He insists that all the callers on the floor greet their customers with strict adherence to the script endorsed by the company. Any changes from routine are marked down on quality irrespective of their impact. Jerry's style of leadership is ineffective due to his _____ behavior. Answers: insular incompetent rigid callous intemperate

rigid Response Feedback: Jeremy is ineffective as a leader as he is rigid, stiff, and unyielding in his beliefs.

Groups transform individuals into functioning organizational members communicating and enforcing ______ expectations.

role

Groups transform individuals into functioning organizational members communicating and enforcing ______ expectations.

role Explanation Groups transform individuals into functioning organizational members through subtle yet powerful social forces. These social forces, in effect, turn "I" into "we" and "me" into "us." Group influence weaves individuals into the organization's social fabric by communicating and enforcing both role expectations and norms. That is, group members positively reinforce those who adhere to roles and norms with friendship and acceptance.

During a class lecture, Mike finds himself thinking about an argument he had last night with his girlfriend. Mike is displaying

rumination. Explanation Rumination is the uncontrollable repetitive dwelling on causes, meanings, and implications of negative feelings or events in the past.

Which of the following is not a motivating factor? Responsibility Recognition Salary Advancement Stimulating work

salary

In Herzberg's motivator-hygiene theory, Satisfaction comes from motivating factors, and dissatisfaction comes from hygiene factors. Job satisfaction and job dissatisfaction are opposites. Satisfaction comes from hygiene factors, and dissatisfaction comes from motivating factors. Hygiene factors come from the job itself. Motivators come from the work context.

satisfaction comes from motivating factors, and dissatisfaction comes from hygiene factors

Which of the following is an individual function of a group?

satisfy the person's need for affiliation

Meghan is a little stressed over an assignment. She just can't seem to identify an easy way to get this work done, but she's not one to give up. Finally, she says: "If this way fails, I can always try another method." This indicates a positive

secondary appraisal. Explanation Secondary appraisals are perceptions of how able you are to deal or cope with a given demand.

In solving a problem, a manager

selects the optimal response. Explanation Solving problems is the optimal or ideal response.

At ABC Inc., each employee is required to evaluate himself or herself as part of preparation to meet with his or her manager for the annual employee performance review. As part of this, Daneesha prepared a thoughtful and comprehensive list of her strengths and weaknesses. What competence does this display?

self-awareness Explanation According to Table 3.6, self-awareness is displayed by knowing one's strengths and limits.

The part of CSE that can best be enhanced because it is most flexible is

self-efficacy. Explanation Self-efficacy is relatively more flexible than the other three components and thus can be enhanced.

Psychological empowerment is related to

self-efficacy. Explanation While structural empowerment draws on job design and characteristics, psychological empowerment is related to self-efficacy.

The critical aspect of organizational politics is the emphasis on

self-interest. Explanation The critical aspect of organizational politics is the emphasis on self-interest, as this distinguishes politics from other forms of influence.

Josephine is on a four-person project team in her management class. The team produces four reports during the term, and each person leads one of these projects. The project Josephine leads earned an A, while the team receives Cs on the other three reports. Josephine believes that her project was successful was because of her leadership abilities, while the other projects were not as good because of the inadequate skills of the other team members. She demonstrates ______ in her beliefs.

self-serving bias Explanation The self-serving bias represents one's tendency to take more personal responsibility for success than for failure.

When monitoring performance, Frank watches all of the following except

setting goals. Explanation Goals are set so that the manager has something to measure against. Monitoring includes all forms of measuring, tracking, or otherwise verifying progress and ultimate outcomes.

Which of the following is not an unethical tactic to use in negotiations?

showing emotions Explanation Many people believe that good negotiators show no emotion. Emotions are indeed an integral part of the human experience and part of most everything we do. Negotiation experts and researchers acknowledge this and provide guidance on how to use emotions to your advantage.

Person factors and ______ are the two broad categories of OB concepts and theories.

situation factors Explanation OB concepts and theories can be classified into two broad categories: person factors and situation factors.

Able, Baker, and Charlie, Ltd., is an accounting partnership. The Internal Revenue Service has implemented new procedures for filing taxes beginning next year. What kind of change does this represent?

social and political pressures Explanation Social and political pressures are created by social and political events.

Mantle Inc. is noted for its philosophy of having fun, holding parties, and celebrating. For example, staff in each city in which the firm operates are given a budget for parties. The company also uses a variety of performance-based awards and service awards to reinforce employees' efforts. Its positive and enriching environment is supported by the very low employee turnover. What function of organizational culture does this represent?

social system stability Explanation Social system stability reflects the extent to which the work environment is perceived as positive and reinforcing, and the extent to which conflict and change are effectively managed.

Problem solving and critical thinking are ______ because they use logic and reasoning to develop and evaluate options.

soft skills Explanation Soft skills relate to human interactions and include both interpersonal skills and personal attributes. They include problem solving, critical thinking, active listening and judgment and decision making.

Justin and Edwardo were hired by the same network support firm. One year later, Edwardo receives a promotion to team leader. Justin believes he has stronger technical skills and asks his supervisor why he did not get the promotion. The supervisor says that Edwardo has better people skills. Having ____ can enable a person's promotion.

soft skills. Explanation For most jobs you are selected for your technical skills, your ability to do the given job (i.e., hard skills). What it takes to get promoted is your perceived ability to get things done through others and manage people. Figure 1.1 illustrates how technical or job-specific skills decline in importance as you move to levels of higher responsibility, while the need for personal skills increases.

Halim hires a new employee who best meets the characteristics he is looking for in the ideal employee. He is

solving the problem. Explanation Solving problems is the optimal or ideal response.

Joyce finds that the members of the project team to which she has been assigned in her management class are all athletes on the college's football and basketball teams. She immediately considers dropping the class because she thinks her experience with that team will be negative. Joyce is likely to be reacting to a

stereotype. Explanation A stereotype is an individual's set of beliefs about the characteristics or attributes of a group.

Sharon observes her older co-worker, Robert, being rude to a new trainee. She has seen this behavior in Robert whenever the company brings in new trainees. Sharon believes that Robert resents the younger management trainees, who he refers to as "know-it-alls." She talks to him about _______ to help him understand.

stereotyping Explanation A stereotype is an individual's set of beliefs about the characteristics or attributes of a group.

The general strategy for the practical application of situational theories includes all of the following except

stick to one leadership style. Explanation The general strategy contains five steps (1) Identify important outcomes. (2) Identify relevant leadership behaviors. (3) Identify situational conditions. (4) Match leadership to the conditions at hand. (5) Determine how to make the match.

Carla will graduate from her business program next month. She is happiest in jobs where she doesn't know what will be waiting for her when she gets to work, and she excels at problem solving. Carla needs a career that involvesv:

stimulation. Explanation Carla enjoys a varied routine and a challenge. According to Figure 2.2, Carla values stimulation. Stimulation is excitement, novelty, and challenge in life (daring, a varied life, an exciting life).

As a manager, Laurel has established a new work group to tackle a particular project. She has introduced all the members to each other and the group has elected a leader. However, lately Laurel has noticed that some group members are procrastinating on their duties. Two cliques have formed within the group and they are frequently arguing with and challenging the group leader's opinions. Which of the following stages of group development process is Laurel observing?

storming Explanation Laurel is observing the storming stage of group development here. The storming stage of the group development process is a time of testing. Individuals test the leader's policies and assumptions as they try to determine how they fit into the power structure. Subgroups take shape, and subtle forms of rebellion, such as procrastination, occur.

A(n) ________ outlines the organization's long-term direction and the actions necessary to achieve planned results.

strategic plan Explanation A strategic plan outlines an organization's long-term direction and the actions necessary to achieve planned results.

During ________, the strategic planning groups decide on a strategic direction by defining the company's mission and goals, its external opportunities and threats, and its internal strengths and weaknesses.

strategy formulation

TNT, Inc. manufacturers desktops and laptops. TNT has decided to enhance revenues by expanding its product line to the tablet market and has identified ACME, Inc. as a potential competitor. TNT Inc. is currently in the ________ stage of the strategic management process.

strategy implementation

According to Kaplan and Norton, a(n) ________ is a "visual representation of a company's critical objectives and the crucial relationships among them that drive organizational performance."

strategy map. Explanation A strategy map is a "visual representation of a company's critical objectives and the crucial relationships among them that drive organizational performance." Maps show relationships among a company's strategic goals. This helps employees understand how their work contributes to their employer's overall success.

Negative reinforcement is

strengthening a behavior by contingently withdrawing something displeasing

John hasn't started working on his term paper. He wants to do well in the course, but he's struggling with how to identify a solid topic and start an outline. Most of the other students appear to be moving ahead. John is procrastinating because he is

stressed. Explanation In Malcolm Gladwell's book, Outliers: The Story of Success, he concludes, "Ten thousand hours is the magic number of greatness" for practice.

Which of the following is not a benefit of positive emotions?

survival orientation Explanation Negative experiences activate a survival orientation, which leads us to be more responsive to negative information.

The __________ includes inputs, strategic plans, target elements of change, and outputs. It is a very practical approach that can diagnose what to change and how to evaluate the success of a change effort.

systems model of change Explanation Figure 16.6 shows the four targeted elements of change: (1) organizational arrangements, (2) social factors, (3) methods, and (4) people.

To make a good first impression in an interaction with another, a person should do all of the following except

talk a lot about yourself. Explanation Five recommendations to make a good first impression are: (1) Set an intention—When you're preparing for an event, think of whom you'd like to meet and what you'd like to achieve from that introduction. Then, plan your energy, intro, and comments accordingly. (2) Consider your ornaments—Be sure your ornaments match your personality and your intended message. (3) Remember that your body speaks—Pay attention to the nonverbals. (4) Bust bad moods and bad days—If you cannot shake a bad mood or prevent your bad day from spilling over into your event, then stay home. (5) Be interested to be interesting—People tend to like people that like them.

Jenna and Martin complete their part of Project Zenith. Their project is best measured against ______ goals

task

Lana is frustrated with her OB project group. She frequently asks: "What's the problem with our group? We don't seem to be getting anywhere." This is a(n) ____ function.

task Explanation Task roles enable the work group to define, clarify, and pursue a common purpose

Ellen has worked on a project to market a new medication from beginning to end—advertising, packaging, etc. The design program wins an award. Ellen's _____ is high.

task identity Explanation Task identity is the extent to which the job requires an individual to perform a whole or completely identifiable piece of work.

In a study of executives from 300 companies, the most desirable soft skill named was

teamwork. Explanation Executives from 300 companies indicated that teamwork was the single most desirable soft skill (64 percent of executives).

Framing bias

tendency to be influenced by the way that a situation or problem is presented

Javier asked Maria for general advice on homework. Then he asked her to let him see specific problems. Now he wants to copy her assignment, claiming he has no time and the content is too difficult. This is an example of

the Slippery Slope

The contingency approach suggests that

the best answer depends on the situation

The contingency approach suggests that

the best answer depends on the situation. Explanation A contingency approach calls for using OB concepts and tools as situationally appropriate, instead of trying to rely on "one best way." This means there is no single best way to manage people, teams, and organizations.

Extrinsic rewards come from ___ and intrinsic rewards come from ___

the environment; oneself

Extrinsic rewards come from ________ and intrinsic rewards come from ________.

the environment; oneself Explanation Extrinsic rewards come from the environment. Intrinsic rewards are self-granted.

Leader-member exchange theory focuses on: -the leader's behavior. -the leader's relationships with followers. -the leader's traits. -the leader's position in the organization. -the leader's vision.

the leaders relationships with followers

Hindsight bias

the opposite of overconfidence bias as it occurs when looking backward in time and mistakes seem obvious after they have already occurred

Papa Bill's, a local pizza chain, institutes a new compensation policy to increase the on-time delivery of its pizzas. Drivers are paid an extra $1.00 for each on-time delivery, payable at the next pay period. However, a review of the policy showed that traffic tickets for reckless driving among delivery drivers increased by 50 percent since the new policy was introduced. Which of the following best explains why this reward system did not perform as expected?

the orgnanizationl

A low instrumentality means that: the person is not motivated. the person is highly motivated. the person does not feel confident in his/her ability to do the job. the person does not believe putting forth effort will result in gaining a reward. the person does not value the outcomes.

the person does not believe putting forth effort will result in gaining a reward

An expectancy of zero means that: the person does not value the outcomes. the person is highly motivated. the person does not trust the company. the person is not motivated. the person does not feel confident in his/her ability to do the job.

the person does not feel confident in his/her ability to do the job

A low valence means that: the person is not motivated. the person is highly motivated. the person does not feel confident in his/her ability to do the job. the person does not trust the company. the person does not value the outcomes.

the person does not value the outcomes

Naturalist intelligence is

the potential to live in harmony with one's environment

Interactional justice is defined as the perceived fairness of

the quality of interpersonal treatment received

Interactional justice is defined as the perceived fairness of

the quality of interpersonal treatment received. Explanation Interactional justice relates to the "quality of the interpersonal treatment people receive when procedures are implemented."

Perceptions of equity are based on comparing: my total output to another's total input. my total input to another's total output. my total output to another's total output. the ratio of my output to my input, compared to another's total output to input. my total input to another's total input.

the ratio of my output to my input, compared to another's total output to input

Nita is the office manager at a service station. Two applicants, Max and Tim, arrive to interview for the open mechanic's position. Max'a clothes are dirty, his hair is unwashed, and he doesn't make eye contact when he gives Nita his name. Tim dresses in clean jeans and a nice shirt, is shaved, and shakes Nita's hand when he introduces himself. After the interview, Keith the head mechanic, asks Nita what she thought. Both men have the skill set needed for the position. Nita recommends Tim. She is responding to her perception of

the target

Nita is the office manager at a service station. Two applicants, Max and Tim, arrive to interview for the open mechanic's position. Max's clothes are dirty, his hair is unwashed, and he doesn't make eye contact when he gives Nita his name. Tim dresses in clean jeans and a nice shirt, is shaved, and shakes Nita's hand when he introduces himself. After the interview, Keith the head mechanic, asks Nita what she thought. Both men have the skill set needed for the position. Nita recommends Tim. She is responding to her perception of

the target. Explanation The characteristics of the target—the person being observed—include direction of gaze, facial features and body shape, nonverbal cues, appearance or dress, and physical attractiveness.

Organizational culture influences

the type of organizational structure adopted by a company and a host of internal processes implemented in pursuit of organizational goals

Enacted values

the values and norms that actually are exhibited or converted into employee behavior

The theory of motivation that focuses on two sharply contrasting sets of assumptions about human nature is: herzberg's motivator-hygiene. extrinsic and intrinsic motivation. theory X and Theory Y. acquired needs. maslow's need hierarchy.

theory x and theory y

________ is a positive set of assumptions about people at work. Herzberg's motivator-hygiene theory Acquired needs theory Theory X Self-determination theory Theory Y

theory y

Why do recruiters use virtual interviews?

to reduce problems associated with implicit cognition Explanation More companies now are using virtual interviews as a tool for reducing problems associated with implicit cognition. Virtual interviews can improve the accuracy of job interviews and reduce costs: -Consistency. Video-enabled interviews standardize the process, which leads to more reliable evaluations. For example, Walmart uses video interviews to obtain a better idea of how candidate pharmacists will interact with customers. T.G.I. Friday's restaurant selects restaurant managers through video interviews for the same reason. -Collaboration. Whether they are recorded or live, video interviews can encourage collaboration among those making hiring decisions. And experts suggest more input leads to better candidate selection. -Saving time and money.

The remedy for motivated blindness is

to root out conflicts of interest. Explanation Table 1.2 states that the remedy for motivated blindness is to root out conflicts of interest. Simply being aware of them doesn't necessarily reduce their negative effect on decision making.

Dr. Beswick enjoys teaching college courses. She likes the salary as well as the ability to travel in summers. However, she also finds the job challenging and enjoyable. This would imply that she is working for

total rewards. Explanation Total rewards encompass not only compensation and benefits, but also personal and professional growth opportunities and a motivating work environment that includes recognition, job design, and work-life balance. Dr. Beswick enjoys the total rewards (i.e., teaching, salary, benefits, and a challenging, enjoyable job).

Which of the following is not a key workplace attitude that managers should track?

turnover

The self-transcendence part of Schwartz's model is composed of

universalism and benevolence. Explanation According to Figure 2.2, self-transcendence is composed of universalism and benevolence.

Timmy is really nervous about his presentation to the board of directors. He tends to stand straight up, move from one foot to the other and hold on to the podium. A better strategy might be to

walk around or stand in particular spots for different lengths of time. Explanation Considerations about how you move and about your eye contact with your audience are part of the develop your stage presence step of the TED protocol for effective presentations.

Over confidence bias

what occurs when individuals overestimate their ability to predict future events

Hope consists of

willpower and waypower. Explanation The two components of hope are willpower and waypower.

Which of the following is not a driver or cause of organizational culture?

work attitudes and behaviors

Joan is providing guidance to employees about what should be done and how to do it, scheduling work, and empowering employees. According to path-goal theory, she is exhibiting the _____ leadership style. Answers: achievement-oriented work-facilitation value-based interaction-facilitation representation and networking behaviors

work-facilitation Response Feedback: Work-facilitation leader behavior includes planning, scheduling, organizing, and coordinating work; providing mentoring, coaching, counseling, feedback to assist employees in developing their skills; eliminating roadblocks; providing resources; and empowering employees to take actions and make decisions. Refer: Table 14-2

Ellen and George work for the same company. Ellen, a Gen Xer, really appreciates the flextime opportunities, while George, a baby boomer, takes advantage of the free computer training offered at the company. These policies are examples of

work-life balance. Explanation A study of work values across 16,000 people from different generational groups suggests that organizations should consider implementing work policies that are targeted toward different generational groups.

Barriers to Delegation

• Belief in the fallacy, "If you want it done right, do it yourself." • Lack of confidence and trust in lower-level employees. • Low self-confidence. • Fear of being called lazy. • Vague job definition. • Fear of competition from those below. • Reluctance to take the risks involved in depending on others. • Lack of controls that provide early warning of problems with delegated duties. • Poor example set by bosses who do not delegate.

Four Functions of Organizational Culture

1. Establishes organizational identity. 2. Encourages collective commitment. 3. Ensures social system stability, which reflects the extent to which the work environment is perceived as positive and reinforcing, and the extent to which conflict and change are effectively managed. 4. Acts as sense-making device, by helping members make sense of their surroundings by helping employees understand why the organization does what it does and how it intends to accomplish its long-term goals.

__________ rely on leveraging technology and structural flexibility to maximize potential value through outsourcing and external collaboration. A. Traditional structures B. Horizontal designs C. Open designs D. Matrix organizations E. Divisional organizations

C

What is the major problem with the Myers-Briggs Type indicator as a measure of the personality? a. An excess of classifications which tends to confuse person b. It is not reliable over time c. It tends to overemphasize intuitive traits a over analytical traits d. Indicates results related exclusively to job performance

b. It is not reliable over time

______ are embellished heroic acts of accomplishments, especially regarding the founding of an organization a. Artifacts b. Stories or Sagas c. Rites d. Rituals

b. Stories or Sagas

Which of the following statements about organizational socialization is false? a. Socialization begins before an employee actually joins an organization b. The entire socialization process may range from a few weeks to a year to complete c. Socialization can take longer for one person than another, depending on individual differences d. Research shows that realistic job previews do not help reduce turnover e. There is no set way to "onboard" new employees

d. Research shows that realistic job previews do not help reduce turnover

Marcel hates the annual review process he has to do for the employees in his department. Although he is fully aware of some of his employees' faults, he is reluctant to discuss them in person, and is only comfortable writing them in an unsigned report that his supervisor will review. Marcel's behavior demonstrates _______. a. Provision of due process b. Inflated assessment of employee performance c. Organizational citizenship behavior d. Managerial sincerity e. Fear of confrontation

e. Fear of confrontation

Which of the following statements is most likely to be true regarding behavioral structured interviews? a. They are most useful when interviewing knowledge workers. b. They increase an interviewer's reliance on his or her "gut feelings." c. They are conducted in a similar manner as audition-type interviews. d. They are useful only for interviewing non-skilled workers. e. They increase the effectiveness of the interview technique.

e. They increase the effectiveness of the interview technique.

Level 1: Observable Artifacts Artifacts:

physical manifestations of an organization's culture

Competing values framework (CVF):

provides a practical way for managers to understand, measure, and change organizational culture.

The CVF indicates

that organizations vary along two fundamental dimensions or axes: The first dimension is the extent to which an organization focuses its attention and efforts on internal dynamics and employees or outward toward its external environment and its customers and shareholders. The second dimension is the organization's preference for flexibility and discretion or control and stability.


Related study sets

How Marketing Channels and Supply Chains Create Value Quiz Questions

View Set

Exam 4 endocrine and reproductive proctored assessment

View Set

Life Insurance Underwriting and Policy Issue

View Set

Human Growth and Development Middle Age

View Set